Sei sulla pagina 1di 137

c 


  c  
 c 

1. The nurse In-charge in labor and delivery unit administered a dose of terbutaline to a client without checking the
client¶s pulse. The standard that would be used to determine if the nurse was negligent is:

a. The physician¶s orders.


b. The action of a clinical nurse specialist who is recognized expert in the field.
c. The statement in the drug literature about administration of terbutaline.
d. The actions of a reasonably prudent nurse with similar education and experience.

2. Nurse Trish is caring for a female client with a history of GI bleeding, sickle cell disease, and a platelet count of
22,000/ȝl. The female client is dehydrated and receiving dextrose 5% in half-normal saline solution at 150 ml/hr.
The client complains of severe bone pain and is scheduled to receive a dose of morphine sulfate. In administering
the medication, Nurse Trish should avoid which route?

a. I.V c. Oral
b. I.M d. S.C

3. Dr. Garcia writes the following order for the client who has been recently admitted ³Digoxin .125 mg P.O. once
daily.´ To prevent a dosage error, how should the nurse document this order onto the medication administration
record?

a. ³Digoxin .1250 mg P.O. once daily´ c. ³Digoxin 0.125 mg P.O. once daily´
b. ³Digoxin 0.1250 mg P.O. once daily´ d. ³Digoxin .125 mg P.O. once daily´

4. A newly admitted female client was diagnosed with deep vein thrombosis. Which nursing diagnosis should
receive the highest priority?

a. Ineffective peripheral tissue perfusion related to venous congestion.


b. Risk for injury related to edema.
c. Excess fluid volume related to peripheral vascular disease.
d. Impaired gas exchange related to increased blood flow.

5. Nurse Betty is assigned to the following clients. The client that the nurse would see first after endorsement?

a. A 34 year-old post operative appendectomy client of five hours who is complaining of pain.
b. A 44 year-old myocardial infarction (MI) client who is complaining of nausea.
c. A 26 year-old client admitted for dehydration whose intravenous (IV) has infiltrated.
d. A 63 year-old post operative¶s abdominal hysterectomy client of three days whose incisional dressing is saturated
with serosanguinous fluid.

6. Nurse Gail places a client in a four-point restraint following orders from the physician. The client care plan should
include:

a. Assess temperature frequently. c. Check circulation every 15-30 minutes.


b. Provide diversional activities. d. Socialize with other patients once a shift.

7. A male client who has severe burns is receiving H2 receptor antagonist therapy. The nurse In-charge knows the
purpose of this therapy is to:

a. Prevent stress ulcer c. Facilitate protein synthesis.


b. Block prostaglandin synthesis d. Enhance gas exchange
V. The doctor orders hourly urine output measurement for a postoperative male client. The nurse Trish records the
following amounts of output for 2 consecutive hours: V a.m.: 50 ml; 9 a.m.: 60 ml. Based on these amounts, which
action should the nurse take?

a. Increase the I.V. fluid infusion rate c. Notify the physician


b. Irrigate the indwelling urinary catheter d. Continue to monitor and record hourly urine output

9. Tony, a basketball player twist his right ankle while playing on the court and seeks care for ankle pain and
swelling. After the nurse applies ice to the ankle for 30 minutes, which statement by Tony suggests that ice
application has been effective?

a. ³My ankle looks less swollen now´. c. ³My ankle appears redder now´.
b. ³My ankle feels warm´. d. ³I need something stronger for pain relief´

10. The physician prescribes a loop diuretic for a client. When administering this drug, the nurse anticipates that the
client may develop which electrolyte imbalance?

a. Hypernatremia c. Hypokalemia
b. Hyperkalemia d. Hypervolemia

11.She finds out that some managers have benevolent-authoritative style of management. Which of the following
behaviors will she exhibit most likely?

a. Have condescending trust and confidence in their subordinates.


b. Gives economic and ego awards.
c. Communicates downward to staffs.
d. Allows decision making among subordinates.

12. Nurse Amy is aware that the following is true about functional nursing

a. Provides continuous, coordinated and comprehensive nursing services.


b. One-to-one nurse patient ratio.
c. Emphasize the use of group collaboration.
d. Concentrates on tasks and activities.

13.Which type of medication order might read "Vitamin K 10 mg I.M. daily × 3 days?"

a. Single order c. Standing order


b. Standard written order d. Stat order

14.A female client with a fecal impaction frequently exhibits which clinical manifestation?

a. Increased appetite
b. Loss of urge to defecate
c. Hard, brown, formed stools
d. Liquid or semi-liquid stools

15.Nurse Linda prepares to perform an otoscopic examination on a female client. For proper visualization, the nurse
should position the client's ear by:

a. Pulling the lobule down and back


b. Pulling the helix up and forward
c. Pulling the helix up and back
d. Pulling the lobule down and forward
16. Which instruction should nurse Tom give to a male client who is having external radiation therapy:

a. Protect the irritated skin from sunlight.


b. Eat 3 to 4 hours before treatment.
c. Wash the skin over regularly.
d. Apply lotion or oil to the radiated area when it is red or sore.

17.In assisting a female client for immediate surgery, the nurse In-charge is aware that she should:

a. Encourage the client to void following preoperative medication.


b. Explore the client¶s fears and anxieties about the surgery.
c. Assist the client in removing dentures and nail polish.
d. Encourage the client to drink water prior to surgery.

1V. A male client is admitted and diagnosed with acute pancreatitis after a holiday celebration of excessive food and
alcohol. Which assessment finding reflects this diagnosis?

a. Blood pressure above normal range.


b. Presence of crackles in both lung fields.
c. Hyperactive bowel sounds
d. Sudden onset of continuous epigastric and back pain.

19. Which dietary guidelines are important for nurse Oliver to implement in caring for the client with burns?

a. Provide high-fiber, high-fat diet c. Monitor intake to prevent weight gain.


b. Provide high-protein, high-carbohydrate diet. d. Provide ice chips or water intake.

20.Nurse Hazel will administer a unit of whole blood, which priority information should the nurse have about the
client?

a. Blood pressure and pulse rate. c. Calcium and potassium levels


b. Height and weight. d. Hgb and Hct levels.

21. Nurse Michelle witnesses a female client sustain a fall and suspects that the leg may be broken. The nurse takes
which priority action?

a. Takes a set of vital signs.


b. Call the radiology department for X-ray.
c. Reassure the client that everything will be alright.
d. Immobilize the leg before moving the client.

22.A male client is being transferred to the nursing unit for admission after receiving a radium implant for bladder
cancer. The nurse in-charge would take which priority action in the care of this client?

a. Place client on reverse isolation.


b. Admit the client into a private room.
c. Encourage the client to take frequent rest periods.
d. Encourage family and friends to visit.

23.A newly admitted female client was diagnosed with agranulocytosis. The nurse formulates which priority nursing
diagnosis?

a. Constipation c. Risk for infection


b. Diarrhea d. Deficient knowledge
24.A male client is receiving total parenteral nutrition suddenly demonstrates signs and symptoms of an air
embolism. What is the priority action by the nurse?

a. Notify the physician.


b. Place the client on the left side in the Trendelenburg position.
c. Place the client in high-Fowlers position.
d. Stop the total parenteral nutrition.

25.Nurse May attends an educational conference on leadership styles. The nurse is sitting with a nurse employed at a
large trauma center who states that the leadership style at the trauma center is task-oriented and directive. The nurse
determines that the leadership style used at the trauma center is:

a. Autocratic. c. Democratic.
b. Laissez-faire. d. Situational

26.The physician orders DS 500 cc with KCl 10 mEq/liter at 30 cc/hr. The nurse in-charge is going to hang a 500 cc
bag. KCl is supplied 20 mEq/10 cc. How many cc¶s of KCl will be added to the IV solution?

a. .5 cc c. 1.5 cc
b. 5 cc d. 2.5 cc

27.A child of 10 years old is to receive 400 cc of IV fluid in an V hour shift. The IV drip factor is 60. The IV rate
that will deliver this amount is:

a. 50 cc/ hour c. 24 cc/ hour


b. 55 cc/ hour d. 66 cc/ hour

2V.The nurse is aware that the most important nursing action when a client returns from surgery is:

a. Assess the IV for type of fluid and rate of flow.


b. Assess the client for presence of pain.
c. Assess the Foley catheter for patency and urine output
d. Assess the dressing for drainage.

29. Which of the following vital sign assessments that may indicate cardiogenic shock after myocardial infarction?

a. BP ± V0/60, Pulse ± 110 irregular


b. BP ± 90/50, Pulse ± 50 regular
c. BP ± 130/V0, Pulse ± 100 regular
d. BP ± 1V0/100, Pulse ± 90 irregular

30.Which is the most appropriate nursing action in obtaining a blood pressure measurement?

a. Take the proper equipment, place the client in a comfortable position, and record the appropriate information in
the client¶s chart.
b. Measure the client¶s arm, if you are not sure of the size of cuff to use.
c. Have the client recline or sit comfortably in a chair with the forearm at the level of the heart.
d. Document the measurement, which extremity was used, and the position that the client was in during the
measurement.

31.Asking the questions to determine if the person understands the health teaching provided by the nurse would be
included during which step of the nursing process?

a. Assessmen t c. Implementation
b. Evaluation d. Planning and goals
32.Which of the following item is considered the single most important factor in assisting the health professional in
arriving at a diagnosis or determining the person¶s needs?

a. Diagnostic test results c. History of present illness


b. Biographical date d. Physical examination

33.In preventing the development of an external rotation deformity of the hip in a client who must remain in bed for
any period of time, the most appropriate nursing action would be to use:

a. Trochanter roll extending from the crest of the ileum to the midthigh.
b. Pillows under the lower legs.
c. Footboard
d. Hip-abductor pillow

34.Which stage of pressure ulcer development does the ulcer extend into the subcutaneous tissue?

a. Stage I c. Stage III


b. Stage II d. Stage IV

35.When the method of wound healing is one in which wound edges are not surgically approximated and
integumentary continuity is restored by granulations, the wound healing is termed

a. Second intention healing c. Third intention healing


b. Primary intention healing d. First intention healing

36.An V0-year-old male client is admitted to the hospital with a diagnosis of pneumonia. Nurse Oliver learns that the
client lives alone and hasn¶t been eating or drinking. When assessing him for dehydration, nurse Oliver would
expect to find:

a. Hypothermia c. Distended neck veins


b. Hypertension d. Tachycardia

37.The physician prescribes meperidine (Demerol), 75 mg I.M. every 4 hours as needed, to control a client¶s
postoperative pain. The package insert is ³Meperidine, 100 mg/ml.´ How many milliliters of meperidine should the
client receive?

a. 0.75 c. 0.5
b. 0.6 d. 0.25

3V. A male client with diabetes mellitus is receiving insulin. Which statement correctly describes an insulin unit?

a. It¶s a common measurement in the metric system.


b. It¶s the basis for solids in the avoirdupois system.
c. It¶s the smallest measurement in the apothecary system.
d. It¶s a measure of effect, not a standard measure of weight or quantity.

39.Nurse Oliver measures a client¶s temperature at 102° F. What is the equivalent Centigrade temperature?

a. 40.1 °C c. 4V °C
b. 3V.9 °C d. 3V °C
40.The nurse is assessing a 4V-year-old client who has come to the physician¶s office for his annual physical exam.
One of the first physical signs of aging is:

a. Accepting limitations while developing assets. c. Failing eyesight, especially close vision.
b. Increasing loss of muscle tone. d. Having more frequent aches and pains.

41.The physician inserts a chest tube into a female client to treat a pneumothorax. The tube is connected to water-
seal drainage. The nurse in-charge can prevent chest tube air leaks by:

a. Checking and taping all connections.


b. Checking patency of the chest tube.
c. Keeping the head of the bed slightly elevated.
d. Keeping the chest drainage system below the level of the chest.

42.Nurse Trish must verify the client¶s identity before administering medication. She is aware that the safest way to
verify identity is to:

a. Check the client¶s identification band.


b. Ask the client to state his name.
c. State the client¶s name out loud and wait a client to repeat it.
d. Check the room number and the client¶s name on the bed.

43.The physician orders dextrose 5 % in water, 1,000 ml to be infused over V hours. The I.V. tubing delivers 15
drops/ml. Nurse John should run the I.V. infusion at a rate of:

a. 30 drops/minute c. 20 drops/minute
b. 32 drops/minute d. 1V drops/minute

44.If a central venous catheter becomes disconnected accidentally, what should the nurse in-charge do immediately?

a. Clamp the catheter c. Call the physician


b. Call another nurse d. Apply a dry sterile dressing to the site.

45.A female client was recently admitted. She has fever, weight loss, and watery diarrhea is being admitted to the
facility. While assessing the client, Nurse Hazel inspects the client¶s abdomen and notice that it is slightly concave.
Additional assessment should proceed in which order:

a. Palpation, auscultation, and percussion. c. Palpation, percussion, and auscultation.


b. Percussion, palpation, and auscultation. d. Auscultation, percussion, and palpation.

46. Nurse Betty is assessing tactile fremitus in a client with pneumonia. For this examination, nurse Betty should use
the:

a. Fingertips c. Dorsal surface of the hand


b. Finger pads d. Ulnar surface of the hand

47. Which type of evaluation occurs continuously throughout the teaching and learning process?

a. Summative c. Formative
b. Informative d. Retrospective

4V.A 45 year old client, has no family history of breast cancer or other risk factors for this disease. Nurse John
should instruct her to have mammogram how often?

a. Twice per year c. Every 2 years


b. Once per year d. Once, to establish baseline
49.A male client has the following arterial blood gas values: pH 7.30; Pao2 V9 mmHg; Paco2 50 mmHg; and HCO3
26mEq/L. Based on these values, Nurse Patricia should expect which condition?

a. Respiratory acidosis c. Metabolic acidosis


b. Respiratory alkalosis d. Metabolic alkalosis

50.Nurse Len refers a female client with terminal cancer to a local hospice. What is the goal of this referral?

a. To help the client find appropriate treatment options.


b. To provide support for the client and family in coping with terminal illness.
c. To ensure that the client gets counseling regarding health care costs.
d. To teach the client and family about cancer and its treatment.

51.When caring for a male client with a 3-cm stage I pressure ulcer on the coccyx, which of the following actions
can the nurse institute independently?

a. Massaging the area with an astringent every 2 hours.


b. Applying an antibiotic cream to the area three times per day.
c. Using normal saline solution to clean the ulcer and applying a protective dressing as necessary.
d. Using a povidone-iodine wash on the ulceration three times per day.

52.Nurse Oliver must apply an elastic bandage to a client¶s ankle and calf. He should apply the bandage beginning at
the client¶s:

a. Knee c. Lower thigh


b. Ankle d. Foot

53.A 10 year old child with type 1 diabetes develops diabetic ketoacidosis and receives a continuous insulin
infusion. Which condition represents the greatest risk to this child?

a. Hypernatremia c. Hyperphosphatemia
b. Hypokalemia d. Hypercalcemia

54.Nurse Len is administering sublingual nitrglycerin (Nitrostat) to the newly admitted client. Immediately
afterward, the client may experience:

a. Throbbing headache or dizziness c. Drowsiness or blurred vision.


b. Nervousness or paresthesia. d. Tinnitus or diplopia.

55.Nurse Michelle hears the alarm sound on the telemetry monitor. The nurse quickly looks at the monitor and notes
that a client is in a ventricular tachycardia. The nurse rushes to the client¶s room. Upon reaching the client¶s bedside,
the nurse would take which action first?

a. Prepare for cardioversion c. Call a code


b. Prepare to defibrillate the client d. Check the client¶s level of consciousness

56.Nurse Hazel is preparing to ambulate a female client. The best and the safest position for the nurse in assisting
the client is to stand:

a. On the unaffected side of the client. c. In front of the client.


b. On the affected side of the client. d. Behind the client.
57.Nurse Janah is monitoring the ongoing care given to the potential organ donor who has been diagnosed with
brain death. The nurse determines that the standard of care had been maintained if which of the following data is
observed?

a. Urine output: 45 ml/hr c. Serum pH: 7.32


b. Capillary refill: 5 seconds d. Blood pressure: 90/4V mmHg

5V. Nurse Amy has an order to obtain a urinalysis from a male client with an indwelling urinary catheter. The nurse
avoids which of the following, which contaminate the specimen?

a. Wiping the port with an alcohol swab before inserting the syringe.
b. Aspirating a sample from the port on the drainage bag.
c. Clamping the tubing of the drainage bag.
d. Obtaining the specimen from the urinary drainage bag.

59.Nurse Meredith is in the process of giving a client a bed bath. In the middle of the procedure, the unit secretary
calls the nurse on the intercom to tell the nurse that there is an emergency phone call. The appropriate nursing action
is to:

a. Immediately walk out of the client¶s room and answer the phone call.
b. Cover the client, place the call light within reach, and answer the phone call.
c. Finish the bed bath before answering the phone call.
d. Leave the client¶s door open so the client can be monitored and the nurse can answer the phone call.

60. Nurse Janah is collecting a sputum specimen for culture and sensitivity testing from a client who has a
productive cough. Nurse Janah plans to implement which intervention to obtain the specimen?

a. Ask the client to expectorate a small amount of sputum into the emesis basin.
b. Ask the client to obtain the specimen after breakfast.
c. Use a sterile plastic container for obtaining the specimen.
d. Provide tissues for expectoration and obtaining the specimen.

61. Nurse Ron is observing a male client using a walker. The nurse determines that the client is using the walker
correctly if the client:

a. Puts all the four points of the walker flat on the floor, puts weight on the hand pieces, and then walks into it.
b. Puts weight on the hand pieces, moves the walker forward, and then walks into it.
c. Puts weight on the hand pieces, slides the walker forward, and then walks into it.
d. Walks into the walker, puts weight on the hand pieces, and then puts all four points of the walker flat on the floor.

62.Nurse Amy has documented an entry regarding client care in the client¶s medical record. When checking the
entry, the nurse realizes that incorrect information was documented. How does the nurse correct this error?

a. Erases the error and writes in the correct information.


b. Uses correction fluid to cover up the incorrect information and writes in the correct information.
c. Draws one line to cross out the incorrect information and then initials the change.
d. Covers up the incorrect information completely using a black pen and writes in the correct information

63.Nurse Ron is assisting with transferring a client from the operating room table to a stretcher. To provide safety to
the client, the nurse should:

a. Moves the client rapidly from the table to the stretcher.


b. Uncovers the client completely before transferring to the stretcher.
c. Secures the client safety belts after transferring to the stretcher.
d. Instructs the client to move self from the table to the stretcher.
64.Nurse Myrna is providing instructions to a nursing assistant assigned to give a bed bath to a client who is on
contact precautions. Nurse Myrna instructs the nursing assistant to use which of the following protective items when
giving bed bath?

a. Gown and goggles c. Gloves and shoe protectors


b. Gown and gloves d. Gloves and goggles

65. Nurse Oliver is caring for a client with impaired mobility that occurred as a result of a stroke. The client has
right sided arm and leg weakness. The nurse would suggest that the client use which of the following assistive
devices that would provide the best stability for ambulating?

a. Crutches c. Quad cane


b. Single straight-legged cane d. Walker

66.A male client with a right pleural effusion noted on a chest X-ray is being prepared for thoracentesis. The client
experiences severe dizziness when sitting upright. To provide a safe environment, the nurse assists the client to
which position for the procedure?

a. Prone with head turned toward the side supported by a pillow.


b. Sims¶ position with the head of the bed flat.
c. Right side-lying with the head of the bed elevated 45 degrees.
d. Left side-lying with the head of the bed elevated 45 degrees.

67.Nurse John develops methods for data gathering. Which of the following criteria of a good instrument refers to
the ability of the instrument to yield the same results upon its repeated administration?

a. Validity c. Sensitivity
b. Specificity d. Reliability

6V.Harry knows that he has to protect the rights of human research subjects. Which of the following actions of Harry
ensures anonymity?

a. Keep the identities of the subject secret


b. Obtain informed consent
c. Provide equal treatment to all the subjects of the study.
d. Release findings only to the participants of the study

69.Patient¶s refusal to divulge information is a limitation because it is beyond the control of Tifanny´. What type of
research is appropriate for this study?

a. Descriptive- correlational c. Quasi-experiment


b. Experiment d. Historical

70.Nurse Ronald is aware that the best tool for data gathering is?

a. Interview schedule c. Use of laboratory data


b. Questionnaire d. Observation

71.Monica is aware that there are times when only manipulation of study variables is possible and the elements of
control or randomization are not attendant. Which type of research is referred to this?

a. Field study c. Solomon-Four group design


b. Quasi-experiment d. Post-test only design
72.Cherry notes down ideas that were derived from the description of an investigation written by the person who
conducted it. Which type of reference source refers to this?

a. Footnote c. Primary source


b. Bibliography d. Endnotes

73.When Nurse Trish is providing care to his patient, she must remember that her duty is bound not to do doing any
action that will cause the patient harm. This is the meaning of the bioethical principle:

a. Non-maleficence c. Justice
b. Beneficence d. Solidarity

74.When a nurse in-charge causes an injury to a female patient and the injury caused becomes the proof of the
negligent act, the presence of the injury is said to exemplify the principle of:

a. Force majeure c. Res ipsa loquitor


b. Respondeat superior d. Holdover doctrine

75.Nurse Myrna is aware that the Board of Nursing has quasi-judicial power. An example of this power is:

a. The Board can issue rules and regulations that will govern the practice of nursing
b. The Board can investigate violations of the nursing law and code of ethics
c. The Board can visit a school applying for a permit in collaboration with CHED
d. The Board prepares the board examinations

76. When the license of nurse Krina is revoked, it means that she:

a. Is no longer allowed to practice the profession for the rest of her life
b. Will never have her/his license re-issued since it has been revoked
c. May apply for re-issuance of his/her license based on certain conditions stipulated in RA 9173
d. Will remain unable to practice professional nursing

77.Ronald plans to conduct a research on the use of a new method of pain assessment scale. Which of the following
is the second step in the conceptualizing phase of the research process?

a. Formulating the research hypothesis c. Formulating and delimiting the research problem
b. Review related literature d. Design the theoretical and conceptual framework

7V. The leader of the study knows that certain patients who are in a specialized research setting tend to respond
psychologically to the conditions of the study. This referred to as :

a. Cause and effect c. Halo effect


b. Hawthorne effect d. Horns effect

79.Mary finally decides to use judgment sampling on her research. Which of the following actions of is correct?

a. Plans to include whoever is there during his study.


b. Determines the different nationality of patients frequently admitted and decides to get representations samples
from each.
c. Assigns numbers for each of the patients, place these in a fishbowl and draw 10 from it.
d. Decides to get 20 samples from the admitted patients

V0. The nursing theorist who developed transcultural nursing theory is:

a. Florence Nightingale c. Albert Moore


b. Madeleine Leininger d. Sr. Callista Roy
V1.Marion is aware that the sampling method that gives equal chance to all units in the population to get picked is:

a. Random c. Quota
b. Accidental d. Judgment

V2.John plans to use a Likert Scale to his study to determine the:

a. Degree of agreement and disagreement c. Level of satisfaction


b. Compliance to expected standards d. Degree of acceptance

V3.Which of the following theory addresses the four modes of adaptation?

a. Madeleine Leininger c. Florence Nightingale


b. Sr. Callista Roy d. Jean Watson

V4.Ms. Garcia is responsible to the number of personnel reporting to her. This principle refers to:

a. Span of control c. Downward communication


b. Unity of command d. Leader

V5.Ensuring that there is an informed consent on the part of the patient before a surgery is done, illustrates the bioethical principle
of:

a. Beneficence c. Veracity
b. Autonomy d. Non-maleficence

V6.Nurse Reese is teaching a female client with peripheral vascular disease about foot care; Nurse Reese should include which
instruction?

a. Avoid wearing cotton socks. c. Avoid wearing canvas shoes.


b. Avoid using a nail clipper to cut toenails. d. Avoid using cornstarch on feet.

V7.A client is admitted with multiple pressure ulcers. When developing the client's diet plan, the nurse should include:

a. Fresh orange slices c. Ice cream


b. Steamed broccoli d. Ground beef patties

VV.The nurse prepares to administer a cleansing enema. What is the most common client position used for this procedure?

a. Lithotomy c. Prone
b. Supine d. Sims¶ left lateral

V9.Nurse Marian is preparing to administer a blood transfusion. Which action should the nurse take first?

a. Arrange for typing and cross matching of the client¶s blood.


b. Compare the client¶s identification wristband with the tag on the unit of blood.
c. Start an I.V. infusion of normal saline solution.
d. Measure the client¶s vital signs.

90.A 65 years old male client requests his medication at 9 p.m. instead of 10 p.m. so that he can go to sleep earlier. Which type of
nursing intervention is required?

a. Independent c. Interdependent
b. Dependent d. Intradependent
91.A female client is to be discharged from an acute care facility after treatment for right leg thrombophlebitis. The Nurse Betty
notes that the client's leg is pain-free, without redness or edema. The nurse's actions reflect which step of the nursing process?

a. Assessment c. Implementation
b. Diagnosis d. Evaluation
92.Nursing care for a female client includes removing elastic stockings once per day. The Nurse Betty is aware that the rationale
for this intervention?

a. To increase blood flow to the heart c. To allow the leg muscles to stretch and relax
b. To observe the lower extremities d. To permit veins in the legs to fill with blood.

93.Which nursing intervention takes highest priority when caring for a newly admitted client who's receiving a blood transfusion?

a. Instructing the client to report any itching, swelling, or dyspnea.


b. Informing the client that the transfusion usually take 1 . to 2 hours.
c. Documenting blood administration in the client care record.
d. Assessing the client¶s vital signs when the transfusion ends.

94.A male client complains of abdominal discomfort and nausea while receiving tube feedings. Which intervention is most
appropriate for this problem?

a. Give the feedings at room temperature.


b. Decrease the rate of feedings and the concentration of the formula.
c. Place the client in semi-Fowler's position while feeding.
d. Change the feeding container every 12 hours.

95.Nurse Patricia is reconstituting a powdered medication in a vial. After adding the solution to the powder, she nurse should:

a. Do nothing. c. Shake the vial vigorously.


b. Invert the vial and let it stand for 3 to 5 minutes. d. Roll the vial gently between the palms.

96.Which intervention should the nurse Trish use when administering oxygen by face mask to a female client?

a. Secure the elastic band tightly around the client's head.


b. Assist the client to the semi-Fowler position if possible.
c. Apply the face mask from the client's chin up over the nose.
d. Loosen the connectors between the oxygen equipment and humidifier.

97.The maximum transfusion time for a unit of packed red blood cells (RBCs) is:

a. 6 hours c. 3 hours
b. 4 hours d. 2 hours

9V.Nurse Monique is monitoring the effectiveness of a client's drug therapy. When should the nurse Monique obtain a blood
sample to measure the trough drug level?

a. 1 hour before administering the next dose.


b. Immediately before administering the next dose.
c. Immediately after administering the next dose.
d. 30 minutes after administering the next dose.

99.Nurse May is aware that the main advantage of using a floor stock system is:

a. The nurse can implement medication orders quickly.


b. The nurse receives input from the pharmacist.
c. The system minimizes transcription errors.
d. The system reinforces accurate calculations.

100. Nurse Oliver is assessing a client's abdomen. Which finding should the nurse report as abnormal?

a. Dullness over the liver.


b. Bowel sounds occurring every 10 seconds.
c. Shifting dullness over the abdomen.
d. Vascular sounds heard over the renal arteries.


 
 
       
1. May arrives at the health care clinic and tells the nurse that her last menstrual period was 9 weeks ago. She also
tells the nurse that a home pregnancy test was positive but she began to have mild cramps and is now having
moderate vaginal bleeding. During the physical examination of the client, the nurse notes that May has a dilated
cervix. The nurse determines that May is experiencing which type of abortion?

a. Inevitable c. Threatened
b. Incomplete d. Septic

2. Nurse Reese is reviewing the record of a pregnant client for her first prenatal visit. Which of the following data, if
noted on the client¶s record, would alert the nurse that the client is at risk for a spontaneous abortion?

a. Age 36 years c. History of genital herpes


b. History of syphilis d. History of diabetes mellitus

3. Nurse Hazel is preparing to care for a client who is newly admitted to the hospital with a possible diagnosis of
ectopic pregnancy. Nurse Hazel develops a plan of care for the client and determines that which of the following
nursing actions is the priority?

a. Monitoring weight c. Monitoring apical pulse


b. Assessing for edema d. Monitoring temperature

4. Nurse Oliver is teaching a diabetic pregnant client about nutrition and insulin needs during pregnancy. The nurse
determines that the client understands dietary and insulin needs if the client states that the second half of pregnancy
require:

a. Decreased caloric intake c. Decreased Insulin


b. Increased caloric intake d. Increase Insulin

5. Nurse Michelle is assessing a 24 year old client with a diagnosis of hydatidiform mole. She is aware that one of
the following is unassociated with this condition?

a. Excessive fetal activity. c. Vaginal bleeding


b. Larger than normal uterus for gestational age. d. Elevated levels of human chorionic gonadotropin.

6. A pregnant client is receiving magnesium sulfate for severe pregnancy induced hypertension (PIH). The clinical
findings that would warrant use of the antidote , calcium gluconate is:

a. Urinary output 90 cc in 2 hours. c. Rapid respiratory rate above 40/min.


b. Absent patellar reflexes. d. Rapid rise in blood pressure.

7. During vaginal examination of Janah who is in labor, the presenting part is at station plus two. Nurse, correctly
interprets it as:

a. Presenting part is 2 cm above the plane of the ischial spines.


b. Biparietal diameter is at the level of the ischial spines.
c. Presenting part in 2 cm below the plane of the ischial spines.
d. Biparietal diameter is 2 cm above the ischial spines.

V. A pregnant client is receiving oxytocin (Pitocin) for induction of labor. A condition that warrant the nurse in-
charge to discontinue I.V. infusion of Pitocin is:

a. Contractions every 1 . minutes lasting 70-V0 seconds.


b. Maternal temperature 101.2
c. Early decelerations in the fetal heart rate.
d. Fetal heart rate baseline 140-160 bpm.
9. Calcium gluconate is being administered to a client with pregnancy induced hypertension (PIH). A nursing action
that must be initiated as the plan of care throughout injection of the drug is:

a. Ventilator assistance c. EKG tracings


b. CVP readings d. Continuous CPR

10. A trial for vaginal delivery after an earlier caesareans, would likely to be given to a gravida, who had:

a. First low transverse cesarean was for active herpes type 2 infections; vaginal culture at 39 weeks pregnancy was
positive.
b. First and second caesareans were for cephalopelvic disproportion.
c. First caesarean through a classic incision as a result of severe fetal distress.
d. First low transverse caesarean was for breech position. Fetus in this pregnancy is in a vertex presentation.

11.Nurse Ryan is aware that the best initial approach when trying to take a crying toddler¶s temperature is:

a. Talk to the mother first and then to the toddler.


b. Bring extra help so it can be done quickly.
c. Encourage the mother to hold the child.
d. Ignore the crying and screaming.

12.Baby Tina a 3 month old infant just had a cleft lip and palate repair. What should the nurse do to prevent trauma
to operative site?

a. Avoid touching the suture line, even when cleaning.


b. Place the baby in prone position.
c. Give the baby a pacifier.
d. Place the infant¶s arms in soft elbow restraints.

13. Which action should nurse Marian include in the care plan for a 2 month old with heart failure?

a. Feed the infant when he cries.


b. Allow the infant to rest before feeding.
c. Bathe the infant and administer medications before feeding.
d. Weigh and bathe the infant before feeding.

14.Nurse Hazel is teaching a mother who plans to discontinue breast feeding after 5 months. The nurse should
advise her to include which foods in her infant¶s diet?

a. Skim milk and baby food. c. Iron-rich formula only.


b. Whole milk and baby food. d. Iron-rich formula and baby food.

15.Mommy Linda is playing with her infant, who is sitting securely alone on the floor of the clinic. The mother
hides a toy behind her back and the infant looks for it. The nurse is aware that estimated age of the infant would be:

a. 6 months c. V months
b. 4 months d. 10 months

16.Which of the following is the most prominent feature of public health nursing?

a. It involves providing home care to sick people who are not confined in the hospital.
b. Services are provided free of charge to people within the catchments area.
c. The public health nurse functions as part of a team providing a public health nursing services.
d. Public health nursing focuses on preventive, not curative, services.
17.When the nurse determines whether resources were maximized in implementing Ligtas Tigdas, she is evaluating

a. Effectiveness c. Adequacy
b. Efficiency d. Appropriateness

1V.Vangie is a new B.S.N. graduate. She wants to become a Public Health Nurse. Where should she apply?

a. Department of Health c. Regional Health Office


b. Provincial Health Office d. Rural Health Unit

19.Tony is aware the Chairman of the Municipal Health Board is:

a. Mayor c. Public Health Nurse


b. Municipal Health Officer d. Any qualified physician

20.Myra is the public health nurse in a municipality with a total population of about 20,000. There are 3 rural health
midwives among the RHU personnel. How many more midwife items will the RHU need?

a. 1 c. 3
b. 2 d. The RHU does not need any more midwife item.

21.According to Freeman and Heinrich, community health nursing is a developmental service. Which of the
following best illustrates this statement?

a. The community health nurse continuously develops himself personally and professionally.
b. Health education and community organizing are necessary in providing community health services.
c. Community health nursing is intended primarily for health promotion and prevention and treatment of disease.
d. The goal of community health nursing is to provide nursing services to people in their own places of residence.

22.Nurse Tina is aware that the disease declared through Presidential Proclamation No. 4 as a target for eradication
in the Philippines is?

a. Poliomyelitis c. Rabies
b. Measles d. Neonatal tetanus

23.May knows that the step in community organizing that involves training of potential leaders in the community is:

a. Integration c. Community study


b. Community organization d. Core group formation

24.Beth a public health nurse takes an active role in community participation. What is the primary goal of
community organizing?

a. To educate the people regarding community health problems


b. To mobilize the people to resolve community health problems
c. To maximize the community¶s resources in dealing with health problems.
d. To maximize the community¶s resources in dealing with health problems.

25.Tertiary prevention is needed in which stage of the natural history of disease?


a. Pre-pathogenesis c. Prodromal
b. Pathogenesis d. Terminal
26.The nurse is caring for a primigravid client in the labor and delivery area. Which condition would place the client
at risk for disseminated intravascular coagulation (DIC)?

a. Intrauterine fetal death. c. Dysfunctional labor.


b. Placenta accreta. d. Premature rupture of the membranes.

27.A fullterm client is in labor. Nurse Betty is aware that the fetal heart rate would be:

a. V0 to 100 beats/minute c. 120 to 160 beats/minute


b. 100 to 120 beats/minute d. 160 to 1V0 beats/minute

2V.The skin in the diaper area of a 7 month old infant is excoriated and red. Nurse Hazel should instruct the mother
to:

a. Change the diaper more often.


b. Apply talc powder with diaper changes.
c. Wash the area vigorously with each diaper change.
d. Decrease the infant¶s fluid intake to decrease saturating diapers.

29.Nurse Carla knows that the common cardiac anomalies in children with Down Syndrome (tri-somy 21) is:

a. Atrial septal defect c. Ventricular septal defect


b. Pulmonic stenosis d. Endocardial cushion defect

30.Malou was diagnosed with severe preeclampsia is now receiving I.V. magnesium sulfate. The adverse effects
associated with magnesium sulfate is:

a. Anemia c. Hyperreflexia
b. Decreased urine output d. Increased respiratory rate

31.A 23 year old client is having her menstrual period every 2 weeks that last for 1 week. This type of menstrual
pattern is bets defined by:

a. Menorrhagia c. Dyspareunia
b. Metrorrhagia d. Amenorrhea

32.Jannah is admitted to the labor and delivery unit. The critical laboratory result for this client would be:

a. Oxygen saturation c. Blood typing


b. Iron binding capacity d. Serum Calcium

33.Nurse Gina is aware that the most common condition found during the second-trimester of pregnancy is:

a. Metabolic alkalosis c. Mastitis


b. Respiratory acidosis d. Physiologic anemia

34.Nurse Lynette is working in the triage area of an emergency department. Who needs to be treated first is:

a. A crying 5 year old child with a laceration on his scalp.


b. A 4 year old child with a barking coughs and flushed appearance.
c. A 3 year old child with Down syndrome who is pale and asleep in his mother¶s arms.
d. A 2 year old infant with stridorous breath sounds, sitting up in his mother¶s arms and drooling.
35.Maureen in her third trimester arrives at the emergency room with painless vaginal bleeding. Which of the
following conditions is suspected?

a. Placenta previa c. Premature labor


b. Abruptio placentae d. Sexually transmitted disease

36.A young child named Richard is suspected of having pinworms. The community nurse collects a stool specimen
to confirm the diagnosis. The nurse should schedule the collection of this specimen for:

a. Just before bedtime c. Any time during the day


b. After the child has been bathe d. Early in the morning

37.In doing a child¶s admission assessment, Nurse Betty should be alert to note which signs or symptoms of chronic
lead poisoning?

a. Irritability and seizures c. Bradycardia and hypotension


b. Dehydration and diarrhea d. Petechiae and hematuria

3V.To evaluate a woman¶s understanding about the use of diaphragm for family planning, Nurse Trish asks her to
explain how she will use the appliance. Which response indicates a need for further health teaching?

a. ³I should check the diaphragm carefully for holes every time I use it´
b. ³I may need a different size of diaphragm if I gain or lose weight more than 20 pounds´
c. ³The diaphragm must be left in place for atleast 6 hours after intercourse´
d. ³I really need to use the diaphragm and jelly most during the middle of my menstrual cycle´.

39.Hypoxia is a common complication of laryngotracheobronchitis. Nurse Oliver should frequently assess a child
with laryngotracheobronchitis for:

a. Drooling c. Restlessness
b. Muffled voice d. Low-grade fever

40.How should Nurse Michelle guide a child who is blind to walk to the playroom?

a. Without touching the child, talk continuously as the child walks down the hall.
b. Walk one step ahead, with the child¶s hand on the nurse¶s elbow.
c. Walk slightly behind, gently guiding the child forward.
d. Walk next to the child, holding the child¶s hand.

41.When assessing a newborn diagnosed with ductus arteriosus, Nurse Olivia should expect that the child most
likely would have an:

a. Loud, machinery-like murmur. c. Decreased BP reading in the upper extremities


b. Bluish color to the lips. d. Increased BP reading in the upper extremities.

42.The reason nurse May keeps the neonate in a neutral thermal environment is that when a newborn becomes too
cool, the neonate requires:

a. Less oxygen, and the newborn¶s metabolic rate increases.


b. More oxygen, and the newborn¶s metabolic rate decreases.
c. More oxygen, and the newborn¶s metabolic rate increases.
d. Less oxygen, and the newborn¶s metabolic rate decreases.
43.Before adding potassium to an infant¶s I.V. line, Nurse Ron must be sure to assess whether this infant has:

a. Stable blood pressure c. Moro¶s reflex


b. Patant fontanelles d. Voided

44.Nurse Carla should know that the most common causative factor of dermatitis in infants and younger children is:

a. Baby oil c. Laundry detergent


b. Baby lotion d. Powder with cornstarch

45.During tube feeding, how far above an infant¶s stomach should the nurse hold the syringe with formula?

a. 6 inches c. 1V inches
b. 12 inches d. 24 inches

46. In a mothers¶ class, Nurse Lhynnete discussed childhood diseases such as chicken pox. Which of the following
statements about chicken pox is correct?

a. The older one gets, the more susceptible he becomes to the complications of chicken pox.
b. A single attack of chicken pox will prevent future episodes, including conditions such as shingles.
c. To prevent an outbreak in the community, quarantine may be imposed by health authorities.
d. Chicken pox vaccine is best given when there is an impending outbreak in the community.

47.Barangay Pinoy had an outbreak of German measles. To prevent congenital rubella, what is the BEST advice that
you can give to women in the first trimester of pregnancy in the barangay Pinoy?

a. Advice them on the signs of German measles.


b. Avoid crowded places, such as markets and movie houses.
c. Consult at the health center where rubella vaccine may be given.
d. Consult a physician who may give them rubella immunoglobulin.

4V.Myrna a public health nurse knows that to determine possible sources of sexually transmitted infections, the
BEST method that may be undertaken is:

a. Contact tracing c. Mass screening tests


b. Community survey d. Interview of suspects

49.A 33-year old female client came for consultation at the health center with the chief complaint of fever for a
week. Accompanying symptoms were muscle pains and body malaise. A week after the start of fever, the client
noted yellowish discoloration of his sclera. History showed that he waded in flood waters about 2 weeks before the
onset of symptoms. Based on her history, which disease condition will you suspect?

a. Hepatitis A c. Tetanus
b. Hepatitis B d. Leptospirosis

50.Mickey a 3-year old client was brought to the health center with the chief complaint of severe diarrhea and the
passage of ³rice water´ stools. The client is most probably suffering from which condition?

a. Giardiasis c. Amebiasis
b. Cholera d. Dysentery

51.The most prevalent form of meningitis among children aged 2 months to 3 years is caused by which
microorganism?

a. Hemophilus influenzae c. Steptococcus pneumoniae


b. Morbillivirus d. Neisseria meningitidis
52.The student nurse is aware that the pathognomonic sign of measles is Koplik¶s spot and you may see Koplik¶s
spot by inspecting the:

a. Nasal mucosa c. Skin on the abdomen


b. Buccal mucosa d. Skin on neck

53.Angel was diagnosed as having Dengue fever. You will say that there is slow capillary refill when the color of
the nailbed that you pressed does not return within how many seconds?

a. 3 seconds c. 9 seconds
b. 6 seconds d. 10 seconds

54.In Integrated Management of Childhood Illness, the nurse is aware that the severe conditions generally require
urgent referral to a hospital. Which of the following severe conditions DOES NOT always require urgent referral to
a hospital?

a. Mastoiditis c. Severe pneumonia


b. Severe dehydration d. Severe febrile disease

55.Myrna a public health nurse will conduct outreach immunization in a barangay Masay with a population of about
1500. The estimated number of infants in the barangay would be:

a. 45 infants c. 55 infants
b. 50 infants d. 65 infants

56.The community nurse is aware that the biological used in Expanded Program on Immunization (EPI) should
NOT be stored in the freezer?

a. DPT c. Measles vaccine


b. Oral polio vaccine d. MMR

57.It is the most effective way of controlling schistosomiasis in an endemic area?

a. Use of molluscicides c. Proper use of sanitary toilets


b. Building of foot bridges d. Use of protective footwear, such as rubber boots

5V.Several clients is newly admitted and diagnosed with leprosy. Which of the following clients should be classified
as a case of multibacillary leprosy?

a. 3 skin lesions, negative slit skin smear c. 5 skin lesions, negative slit skin smear
b. 3 skin lesions, positive slit skin smear d. 5 skin lesions, positive slit skin smear

59.Nurses are aware that diagnosis of leprosy is highly dependent on recognition of symptoms. Which of the
following is an early sign of leprosy?

a. Macular lesions c. Thickened painful nerves


b. Inability to close eyelids d. Sinking of the nosebridge

60.Marie brought her 10 month old infant for consultation because of fever, started 4 days prior to consultation. In
determining malaria risk, what will you do?

a. Perform a tourniquet test. c. Get a specimen for blood smear.


b. Ask where the family resides. d. Ask if the fever is present everyday.
61.Susie brought her 4 years old daughter to the RHU because of cough and colds. Following the IMCI assessment
guide, which of the following is a danger sign that indicates the need for urgent referral to a hospital?

a. Inability to drink c. Signs of severe dehydration


b. High grade fever d. Cough for more than 30 days

62.Jimmy a 2-year old child revealed ³baggy pants´. As a nurse, using the IMCI guidelines, how will you manage
Jimmy?

a. Refer the child urgently to a hospital for confinement.


b. Coordinate with the social worker to enroll the child in a feeding program.
c. Make a teaching plan for the mother, focusing on menu planning for her child.
d. Assess and treat the child for health problems like infections and intestinal parasitism.

63.Gina is using Oresol in the management of diarrhea of her 3-year old child. She asked you what to do if her child
vomits. As a nurse you will tell her to:

a. Bring the child to the nearest hospital for further assessment.


b. Bring the child to the health center for intravenous fluid therapy.
c. Bring the child to the health center for assessment by the physician.
d. Let the child rest for 10 minutes then continue giving Oresol more slowly.

64.Nikki a 5-month old infant was brought by his mother to the health center because of diarrhea for 4 to 5 times a
day. Her skin goes back slowly after a skin pinch and her eyes are sunken. Using the IMCI guidelines, you will
classify this infant in which category?

a. No signs of dehydration c. Severe dehydration


b. Some dehydration d. The data is insufficient.

65.Chris a 4-month old infant was brought by her mother to the health center because of cough. His respiratory rate
is 42/minute. Using the Integrated Management of Child Illness (IMCI) guidelines of assessment, his breathing is
considered as:

a. Fast c. Normal
b. Slow d. Insignificant

66.Maylene had just received her 4th dose of tetanus toxoid. She is aware that her baby will have protection against
tetanus for

a. 1 year c. 5 years
b. 3 years d. Lifetime

67.Nurse Ron is aware that unused BCG should be discarded after how many hours of reconstitution?

a. 2 hours c. V hours
b. 4 hours d. At the end of the day

6V.The nurse explains to a breastfeeding mother that breast milk is sufficient for all of the baby¶s nutrient needs only
up to:

a. 5 months c. 1 year
b. 6 months d. 2 years
69.Nurse Ron is aware that the gestational age of a conceptus that is considered viable (able to live outside the
womb) is:

a. V weeks c. 24 weeks
b. 12 weeks d. 32 weeks

70.When teaching parents of a neonate the proper position for the neonate¶s sleep, the nurse Patricia stresses the
importance of placing the neonate on his back to reduce the risk of which of the following?

a. Aspiration c. Suffocation
b. Sudden infant death syndrome (SIDS) d. Gastroesophageal reflux (GER)

71.Which finding might be seen in baby James a neonate suspected of having an infection?

a. Flushed cheeks c. Decreased temperature


b. Increased temperature d. Increased activity level

72.Baby Jenny who is small-for-gestation is at increased risk during the transitional period for which complication?

a. Anemia probably due to chronic fetal hyposia


b. Hyperthermia due to decreased glycogen stores
c. Hyperglycemia due to decreased glycogen stores
d. Polycythemia probably due to chronic fetal hypoxia

73.Marjorie has just given birth at 42 weeks¶ gestation. When the nurse assessing the neonate, which physical
finding is expected?

a. A sleepy, lethargic baby c. Desquamation of the epidermis


b. Lanugo covering the body d. Vernix caseosa covering the body

74.After reviewing the Myrna¶s maternal history of magnesium sulfate during labor, which condition would nurse
Richard anticipate as a potential problem in the neonate?

a. Hypoglycemia c. Respiratory depression


b. Jitteriness d. Tachycardia

75.Which symptom would indicate the Baby Alexandra was adapting appropriately to extra-uterine life without
difficulty?

a. Nasal flaring c. Respiratory rate 40 to 60 breaths/minute


b. Light audible grunting d. Respiratory rate 60 to V0 breaths/minute

76. When teaching umbilical cord care for Jennifer a new mother, the nurse Jenny would include which
information?

a. Apply peroxide to the cord with each diaper change


b. Cover the cord with petroleum jelly after bathing
c. Keep the cord dry and open to air
d. Wash the cord with soap and water each day during a tub bath.

77.Nurse John is performing an assessment on a neonate. Which of the following findings is considered common in
the healthy neonate?

a. Simian crease c. Cystic hygroma


b. Conjunctival hemorrhage d. Bulging fontanelle
7V.Dr. Esteves decides to artificially rupture the membranes of a mother who is on labor. Following this procedure,
the nurse Hazel checks the fetal heart tones for which the following reasons?

a. To determine fetal well-being. c. To assess fetal position


b. To assess for prolapsed cord d. To prepare for an imminent delivery.

79.Which of the following would be least likely to indicate anticipated bonding behaviors by new parents?

a. The parents¶ willingness to touch and hold the new born.


b. The parent¶s expression of interest about the size of the new born.
c. The parents¶ indication that they want to see the newborn.
d. The parents¶ interactions with each other.

V0.Following a precipitous delivery, examination of the client's vagina reveals a fourth-degree laceration. Which of
the following would be contraindicated when caring for this client?

a. Applying cold to limit edema during the first 12 to 24 hours.


b. Instructing the client to use two or more peripads to cushion the area.
c. Instructing the client on the use of sitz baths if ordered.
d. Instructing the client about the importance of perineal (kegel) exercises.

V1. A pregnant woman accompanied by her husband, seeks admission to the labor and delivery area. She states that
she's in labor and says she attended the facility clinic for prenatal care. Which question should the nurse Oliver ask
her first?

a. ³Do you have any chronic illnesses?´ c. ³What is your expected due date?´
b. ³Do you have any allergies?´ d. ³Who will be with you during labor?´

V2.A neonate begins to gag and turns a dusky color. What should the nurse do first?

a. Calm the neonate.


b. Notify the physician.
c. Provide oxygen via face mask as ordered
d. Aspirate the neonate¶s nose and mouth with a bulb syringe.

V3. When a client states that her "water broke," which of the following actions would be inappropriate for the nurse
to do?

a. Observing the pooling of straw-colored fluid.


b. Checking vaginal discharge with nitrazine paper.
c. Conducting a bedside ultrasound for an amniotic fluid index.
d. Observing for flakes of vernix in the vaginal discharge.

V4. A baby girl is born V weeks premature. At birth, she has no spontaneous respirations but is successfully
resuscitated. Within several hours she develops respiratory grunting, cyanosis, tachypnea, nasal flaring, and
retractions. She's diagnosed with respiratory distress syndrome, intubated, and placed on a ventilator. Which nursing
action should be included in the baby's plan of care to prevent retinopathy of prematurity?

a. Cover his eyes while receiving oxygen. c. Monitor partial pressure of oxygen (Pao2) levels.
b. Keep her body temperature low. d. Humidify the oxygen.

V5. Which of the following is normal newborn calorie intake?

a. 110 to 130 calories per kg. c. At least 2 ml per feeding


b. 30 to 40 calories per lb of body weight. d. 90 to 100 calories per kg
V6. Nurse John is knowledgeable that usually individual twins will grow appropriately and at the same rate as
singletons until how many weeks?

a. 16 to 1V weeks c. 30 to 32 weeks
b. 1V to 22 weeks d. 3V to 40 weeks

V7. Which of the following classifications applies to monozygotic twins for whom the cleavage of the fertilized
ovum occurs more than 13 days after fertilization?

a. conjoined twins c. diamniotic monochorionic twin


b. diamniotic dichorionic twins d. monoamniotic monochorionic twins

VV. Tyra experienced painless vaginal bleeding has just been diagnosed as having a placenta previa. Which of the
following procedures is usually performed to diagnose placenta previa?

a. Amniocentesis c. External fetal monitoring


b. Digital or speculum examination d. Ultrasound

V9. Nurse Arnold knows that the following changes in respiratory functioning during pregnancy is considered
normal:

a. Increased tidal volume c. Decreased inspiratory capacity


b. Increased expiratory volume d. Decreased oxygen consumption

90. Emily has gestational diabetes and it is usually managed by which of the following therapy?

a. Diet c. Oral hypoglycemic


b. Long-acting insulin d. Oral hypoglycemic drug and insulin

91. Magnesium sulfate is given to Jemma with preeclampsia to prevent which of the following condition?

a. Hemorrhage c. Hypomagnesemia
b. Hypertension d. Seizure

92. Cammile with sickle cell anemia has an increased risk for having a sickle cell crisis during pregnancy.
Aggressive management of a sickle cell crisis includes which of the following measures?

a. Antihypertensive agents c. I.V. fluids


b. Diuretic agents d. Acetaminophen (Tylenol) for pain

93. Which of the following drugs is the antidote for magnesium toxicity?

a. Calcium gluconate (Kalcinate) c. Naloxone (Narcan)


b. Hydralazine (Apresoline) d. Rho (D) immune globulin (RhoGAM)

94. Marlyn is screened for tuberculosis during her first prenatal visit. An intradermal injection of purified protein
derivative (PPD) of the tuberculin bacilli is given. She is considered to have a positive test for which of the
following results?

a. An indurated wheal under 10 mm in diameter appears in 6 to 12 hours.


b. An indurated wheal over 10 mm in diameter appears in 4V to 72 hours.
c. A flat circumcised area under 10 mm in diameter appears in 6 to 12 hours.
d. A flat circumcised area over 10 mm in diameter appears in 4V to 72 hours.
95. Dianne, 24 year-old is 27 weeks¶ pregnant arrives at her physician¶s office with complaints of fever, nausea,
vomiting, malaise, unilateral flank pain, and costovertebral angle tenderness. Which of the following diagnoses is
most likely?

a. Asymptomatic bacteriuria c. Pyelonephritis


b. Bacterial vaginosis d. Urinary tract infection (UTI)

96. Rh isoimmunization in a pregnant client develops during which of the following conditions?

a. Rh-positive maternal blood crosses into fetal blood, stimulating fetal antibodies.
b. Rh-positive fetal blood crosses into maternal blood, stimulating maternal antibodies.
c. Rh-negative fetal blood crosses into maternal blood, stimulating maternal antibodies.
d. Rh-negative maternal blood crosses into fetal blood, stimulating fetal antibodies.

97. To promote comfort during labor, the nurse John advises a client to assume certain positions and avoid others.
Which position may cause maternal hypotension and fetal hypoxia?

a. Lateral position c. Supine position


b. Squatting position d. Standing position

9V. Celeste who used heroin during her pregnancy delivers a neonate. When assessing the neonate, the nurse
Lhynnette expects to find:

a. Lethargy 2 days after birth. c. A flattened nose, small eyes, and thin lips.
b. Irritability and poor sucking. d. Congenital defects such as limb anomalies.

99. The uterus returns to the pelvic cavity in which of the following time frames?

a. 7th to 9th day postpartum. c. End of 6th week postpartum.


b. 2 weeks postpartum. d. When the lochia changes to alba.

100. Maureen, a primigravida client, age 20, has just completed a difficult, forceps-assisted delivery of twins. Her
labor was unusually long and required oxytocin (Pitocin) augmentation. The nurse who's caring for her should stay
alert for:

a. Uterine inversion c. Uterine involution


b. Uterine atony d. Uterine discomfort
   c   c      
1. Nurse Michelle should know that the drainage is normal 4 days after a sigmoid colostomy when the stool is:

a. Green liquid c. Loose, bloody


b. Solid formed d. Semiformed

2. Where would nurse Kristine place the call light for a male client with a right-sided brain attack and left
homonymous hemianopsia?

a. On the client¶s right side c. Directly in front of the client


b. On the client¶s left side d. Where the client like

3. A male client is admitted to the emergency department following an accident. What are the first nursing actions of
the nurse?

a. Check respiration, circulation, neurological response.


b. Align the spine, check pupils, and check for hemorrhage.
c. Check respirations, stabilize spine, and check circulation.
d. Assess level of consciousness and circulation.

4. In evaluating the effect of nitroglycerin, Nurse Arthur should know that it reduces preload and relieves angina by:

a. Increasing contractility and slowing heart rate. c. Decreasing contractility and oxygen consumption.
b. Increasing AV conduction and heart rate. d. Decreasing venous return through vasodilation.

5. Nurse Patricia finds a female client who is post-myocardial infarction (MI) slumped on the side rails of the bed
and unresponsive to shaking or shouting. Which is the nurse next action?

a. Call for help and note the time.


b. Clear the airway
c. Give two sharp thumps to the precordium, and check the pulse.
d. Administer two quick blows.

6. Nurse Monett is caring for a client recovering from gastro-intestinal bleeding. The nurse should:

a. Plan care so the client can receive V hours of uninterrupted sleep each night.
b. Monitor vital signs every 2 hours.
c. Make sure that the client takes food and medications at prescribed intervals.
d. Provide milk every 2 to 3 hours.

7. A male client was on warfarin (Coumadin) before admission, and has been receiving heparin I.V. for 2 days. The
partial thromboplastin time (PTT) is 6V seconds. What should Nurse Carla do?

a. Stop the I.V. infusion of heparin and notify the physician.


b. Continue treatment as ordered.
c. Expect the warfarin to increase the PTT.
d. Increase the dosage, because the level is lower than normal.

V. A client undergone ileostomy, when should the drainage appliance be applied to the stoma?

a. 24 hours later, when edema has subsided.


b. In the operating room.
c. After the ileostomy begin to function.
d. When the client is able to begin self-care procedures.
9. A client undergone spinal anesthetic, it will be important that the nurse immediately position the client in:

a. On the side, to prevent obstruction of airway by tongue.


b. Flat on back.
c. On the back, with knees flexed 15 degrees.
d. Flat on the stomach, with the head turned to the side.

10.While monitoring a male client several hours after a motor vehicle accident, which assessment data suggest
increasing intracranial pressure?

a. Blood pressure is decreased from 160/90 to 110/70.


b. Pulse is increased from V7 to 95, with an occasional skipped beat.
c. The client is oriented when aroused from sleep, and goes back to sleep immediately.
d. The client refuses dinner because of anorexia.

11.Mrs. Cruz, V0 years old is diagnosed with pneumonia. Which of the following symptoms may appear first?

a. Altered mental status and dehydration c. Hemoptysis and Dyspnea


b. Fever and chills d. Pleuritic chest pain and cough

12. A male client has active tuberculosis (TB). Which of the following symptoms will be exhibit?

a. Chest and lower back pain c. Fever of more than 104°F (40°C) and nausea
b. Chills, fever, night sweats, and hemoptysis d. Headache and photophobia

13. Mark, a 7-year-old client is brought to the emergency department. He¶s tachypneic and afebrile and has a
respiratory rate of 36 breaths/minute and has a nonproductive cough. He recently had a cold. Form this history; the
client may have which of the following conditions?

a. Acute asthma c. Chronic obstructive pulmonary disease (COPD)


b. Bronchial pneumonia d. Emphysema

14. Marichu was given morphine sulfate for pain. She is sleeping and her respiratory rate is 4 breaths/minute. If
action isn¶t taken quickly, she might have which of the following reactions?

a. Asthma attack c. Seizure


b. Respiratory arrest d. Wake up on his own

15. A 77-year-old male client is admitted for elective knee surgery. Physical examination reveals shallow
respirations but no sign of respiratory distress. Which of the following is a normal physiologic change related to
aging?

a. Increased elastic recoil of the lungs


b. Increased number of functional capillaries in the alveoli
c. Decreased residual volume
d. Decreased vital capacity

16. Nurse John is caring for a male client receiving lidocaine I.V. Which factor is the most relevant to administration
of this medication?

a. Decrease in arterial oxygen saturation (SaO2) when measured with a pulse oximeter.
b. Increase in systemic blood pressure.
c. Presence of premature ventricular contractions (PVCs) on a cardiac monitor.
d. Increase in intracranial pressure (ICP).
17. Nurse Ron is caring for a male client taking an anticoagulant. The nurse should teach the client to:

a. Report incidents of diarrhea. c. Use a straight razor when shaving.


b. Avoid foods high in vitamin K d. Take aspirin to pain relief.

1V. Nurse Lhynnette is preparing a site for the insertion of an I.V. catheter. The nurse should treat excess hair at the
site by:

a. Leaving the hair intact c. Clipping the hair in the area


b. Shaving the area d. Removing the hair with a depilatory.

19. Nurse Michelle is caring for an elderly female with osteoporosis. When teaching the client, the nurse should
include information about which major complication:

a. Bone fracture c. Negative calcium balance


b. Loss of estrogen d. Dowager¶s hump

20. Nurse Len is teaching a group of women to perform BSE. The nurse should explain that the purpose of
performing the examination is to discover:

a. Cancerous lumps c. Changes from previous examinations.


b. Areas of thickness or fullness d. Fibrocystic masses

21. When caring for a female client who is being treated for hyperthyroidism, it is important to:

a. Provide extra blankets and clothing to keep the client warm.


b. Monitor the client for signs of restlessness, sweating, and excessive weight loss during thyroid replacement
therapy.
c. Balance the client¶s periods of activity and rest.
d. Encourage the client to be active to prevent constipation.

22. Nurse Kris is teaching a client with history of atherosclerosis. To decrease the risk of atherosclerosis, the nurse
should encourage the client to:

a. Avoid focusing on his weight. c. Follow a regular diet.


b. Increase his activity level. d. Continue leading a high-stress lifestyle.

23. Nurse Greta is working on a surgical floor. Nurse Greta must logroll a client following a:

a. Laminectomy c. Hemorrhoidectomy
b. Thoracotomy d. Cystectomy.

24. A 55-year old client underwent cataract removal with intraocular lens implant. Nurse Oliver is giving the client
discharge instructions. These instructions should include which of the following?

a. Avoid lifting objects weighing more than 5 lb (2.25 kg).


b. Lie on your abdomen when in bed
c. Keep rooms brightly lit.
d. Avoiding straining during bowel movement or bending at the waist.

25. George should be taught about testicular examinations during:

a. when sexual activity starts c. After age 40


b. After age 69 d. Before age 20.
26. A male client undergone a colon resection. While turning him, wound dehiscence with evisceration occurs.
Nurse Trish first response is to:

a. Call the physician


b. Place a saline-soaked sterile dressing on the wound.
c. Take a blood pressure and pulse.
d. Pull the dehiscence closed.
27. Nurse Audrey is caring for a client who has suffered a severe cerebrovascular accident. During routine
assessment, the nurse notices Cheyne- Strokes respirations. Cheyne-strokes respirations are:

a. A progressively deeper breaths followed by shallower breaths with apneic periods.


b. Rapid, deep breathing with abrupt pauses between each breath.
c. Rapid, deep breathing and irregular breathing without pauses.
d. Shallow breathing with an increased respiratory rate.

2V. Nurse Bea is assessing a male client with heart failure. The breath sounds commonly auscultated in clients with
heart failure are:

a. Tracheal c. Coarse crackles


b. Fine crackles d. Friction rubs

29. The nurse is caring for Kenneth experiencing an acute asthma attack. The client stops wheezing and breath
sounds aren¶t audible. The reason for this change is that:

a. The attack is over.


b. The airways are so swollen that no air cannot get through.
c. The swelling has decreased.
d. Crackles have replaced wheezes.

30. Mike with epilepsy is having a seizure. During the active seizure phase, the nurse should:

a. Place the client on his back remove dangerous objects, and insert a bite block.
b. Place the client on his side, remove dangerous objects, and insert a bite block.
c. Place the client o his back, remove dangerous objects, and hold down his arms.
d. Place the client on his side, remove dangerous objects, and protect his head.

31. After insertion of a cheat tube for a pneumothorax, a client becomes hypotensive with neck vein distention,
tracheal shift, absent breath sounds, and diaphoresis. Nurse Amanda suspects a tension pneumothorax has occurred.
What cause of tension pneumothorax should the nurse check for?

a. Infection of the lung. c. Excessive water in the water-seal chamber


b. Kinked or obstructed chest tube d. Excessive chest tube drainage

32. Nurse Maureen is talking to a male client, the client begins choking on his lunch. He¶s coughing forcefully. The
nurse should:

a. Stand him up and perform the abdominal thrust maneuver from behind.
b. Lay him down, straddle him, and perform the abdominal thrust maneuver.
c. Leave him to get assistance
d. Stay with him but not intervene at this time.

33. Nurse Ron is taking a health history of an V4 year old client. Which information will be most useful to the nurse
for planning care?

a. General health for the last 10 years. c. Family history of diseases.


b. Current health promotion activities. d. Marital status.
34. When performing oral care on a comatose client, Nurse Krina should:

a. Apply lemon glycerin to the client¶s lips at least every 2 hours.


b. Brush the teeth with client lying supine.
c. Place the client in a side lying position, with the head of the bed lowered.
d. Clean the client¶s mouth with hydrogen peroxide.

35. A 77-year-old male client is admitted with a diagnosis of dehydration and change in mental status. He¶s being
hydrated with L.V. fluids. When the nurse
takes his vital signs, she notes he has a fever of 103°F (39.4°C) a cough producing yellow sputum and pleuritic chest
pain. The nurse suspects this clien may have which of the following conditions?

a. Adult respiratory distress syndrome (ARDS) c. Pneumonia


b. Myocardial infarction (MI) d. Tuberculosis

36. Nurse Oliver is working in a out patient clinic. He has been alerted that there is an outbreak of tuberculosis (TB).
Which of the following clients entering the clinic today most likely to have TB?

a. A 16-year-old female high school student


b. A 33-year-old day-care worker
c. A 43-yesr-old homeless man with a history of alcoholism
d. A 54-year-old businessman

37. Virgie with a positive Mantoux test result will be sent for a chest X-ray. The nurse is aware that which of the
following reasons this is done?

a. To confirm the diagnosis


b. To determine if a repeat skin test is needed
c. To determine the extent of lesions
d. To determine if this is a primary or secondary infection

3V. Kennedy with acute asthma showing inspiratory and expiratory wheezes and a decreased forced expiratory
volume should be treated with which of the following classes of medication right away?

a. Beta-adrenergic blockers c. Inhaled steroids


b. Bronchodilators d. Oral steroids

39. Mr. Vasquez 56-year-old client with a 40-year history of smoking one to two packs of cigarettes per day has a
chronic cough producing thick sputum, peripheral edema and cyanotic nail beds. Based on this information, he most
likely has which of the following conditions?

a. Adult respiratory distress syndrome (ARDS) c. Chronic obstructive bronchitis


b. Asthma d. Emphysema


    !"    #      
$# $
% &

40. The treatment for patients with leukemia is bone marrow transplantation. Which statement about bone marrow
transplantation is not correct?

a. The patient is under local anesthesia during the procedure


b. The aspirated bone marrow is mixed with heparin.
c. The aspiration site is the posterior or anterior iliac crest.
d. The recipient receives cyclophosphamide (Cytoxan) for 4 consecutive days before the procedure.
41. After several days of admission, Francis becomes disoriented and complains of frequent headaches. The nurse
in-charge first action would be:

a. Call the physician c. Prepare oxygen treatment


b. Document the patient¶s status in his charts. d. Raise the side rails

42. During routine care, Francis asks the nurse, ³How can I be anemic if this disease causes increased my white
blood cell production?´ The nurse in-charge best response would be that the increased number of white blood cells
(WBC) is:

a. Crowd red blood cells c. Uses nutrients from other cells


b. Are not responsible for the anemia. d. Have an abnormally short life span of cells.

43. Diagnostic assessment of Francis would probably not reveal:

a. Predominance of lymhoblasts c. Abnormal blast cells in the bone marrow


b. Leukocytosis d. Elevated thrombocyte counts

44. Robert, a 57-year-old client with acute arterial occlusion of the left leg undergoes an emergency embolectomy.
Six hours later, the nurse isn¶t able to obtain pulses in his left foot using Doppler ultrasound. The nurse immediately
notifies the physician, and asks her to prepare the client for surgery. As the nurse enters the client¶s room to prepare
him, he states that he won¶t have any more surgery. Which of the following is the best initial response by the nurse?

a. Explain the risks of not having the surgery c. Notifying the nursing supervisor
b. Notifying the physician immediately d. Recording the client¶s refusal in the nurses¶ notes

45. During the endorsement, which of the following clients should the on-duty nurse assess first?

a. The 5V-year-old client who was admitted 2 days ago with heart failure, blood pressure of 126/76 mm Hg, and a
respiratory rate of 22 breaths/ minute.
b. The V9-year-old client with end-stage right-sided heart failure, blood pressure of 7V/50 mm Hg, and a ³do not
resuscitate´ order
c. The 62-year-old client who was admitted 1 day ago with thrombophlebitis and is receiving L.V. heparin
d. The 75-year-old client who was admitted 1 hour ago with new-onset atrial fibrillation and is receiving L.V.
dilitiazem (Cardizem)

46. Honey, a 23-year old client complains of substernal chest pain and states that her heart feels like ³it¶s racing out
of the chest´. She reports no history of cardiac disorders. The nurse attaches her to a cardiac monitor and notes sinus
tachycardia with a rate of 136beats/minutes. Breath sounds are clear and the respiratory rate is 26 breaths/minutes.
Which of the following drugs should the nurse question the client about using?

a. Barbiturates c. Cocaine
b. Opioids d. Benzodiazepines

47. A 51-year-old female client tells the nurse in-charge that she has found a painless lump in her right breast during
her monthly self-examination. Which assessment finding would strongly suggest that this client's lump is cancerous?

a. Eversion of the right nipple and mobile mass c. Mobile mass that is soft and easily delineated
b. Nonmobile mass with irregular edges d. Nonpalpable right axillary lymph nodes

4V. A 35-year-old client with vaginal cancer asks the nurse, "What is the usual treatment for this type of cancer?"
Which treatment should the nurse name?

a. Surgery c. Radiation
b. Chemotherapy d. Immunotherapy
49. Cristina undergoes a biopsy of a suspicious lesion. The biopsy report classifies the lesion according to the TNM
staging system as follows: TIS, N0, M0. What does this classification mean?

a. No evidence of primary tumor, no abnormal regional lymph nodes, and no evidence of distant metastasis
b. Carcinoma in situ, no abnormal regional lymph nodes, and no evidence of distant metastasis
c. Can't assess tumor or regional lymph nodes and no evidence of metastasis
d. Carcinoma in situ, no demonstrable metastasis of the regional lymph nodes, and ascending degrees of distant
metastasis

50. Lydia undergoes a laryngectomy to treat laryngeal cancer. When teaching the client how to care for the neck
stoma, the nurse should include which instruction?

a. "Keep the stoma uncovered."


b. "Keep the stoma dry."
c. "Have a family member perform stoma care initially until you get used to the procedure."
d. "Keep the stoma moist."

51. A 37-year-old client with uterine cancer asks the nurse, "Which is the most common type of cancer in women?"
The nurse replies that it's breast cancer. Which type of cancer causes the most deaths in women?

a. Breast cancer c. Brain cancer


b. Lung cancer d. Colon and rectal cancer

52. Antonio with lung cancer develops Horner's syndrome when the tumor invades the ribs and affects the
sympathetic nerve ganglia. When assessing for signs and symptoms of this syndrome, the nurse should note:

a. miosis, partial eyelid ptosis, and anhidrosis on the affected side of the face.
b. chest pain, dyspnea, cough, weight loss, and fever.
c. arm and shoulder pain and atrophy of arm and hand muscles, both on the affected side.
d. hoarseness and dysphagia.

53. Vic asks the nurse what PSA is. The nurse should reply that it stands for:

a. prostate-specific antigen, which is used to screen for prostate cancer.


b. protein serum antigen, which is used to determine protein levels.
c. pneumococcal strep antigen, which is a bacteria that causes pneumonia.
d. Papanicolaou-specific antigen, which is used to screen for cervical cancer.

54. What is the most important postoperative instruction that nurse Kate must give a client who has just returned
from the operating room after receiving a subarachnoid block?

a. "Avoid drinking liquids until the gag reflex returns."


b. "Avoid eating milk products for 24 hours."
c. "Notify a nurse if you experience blood in your urine."
d. "Remain supine for the time specified by the physician."

55. A male client suspected of having colorectal cancer will require which diagnostic study to confirm the
diagnosis?

a. Stool Hematest c. Sigmoidoscopy


b. Carcinoembryonic antigen (CEA) d. Abdominal computed tomography (CT) scan
56. During a breast examination, which finding most strongly suggests that the
Luz has breast cancer?

a. Slight asymmetry of the breasts.


b. A fixed nodular mass with dimpling of the overlying skin
c. Bloody discharge from the nipple
d. Multiple firm, round, freely movable masses that change with the menstrual cycle

57. A female client with cancer is being evaluated for possible metastasis. Which of the following is one of the most
common metastasis sites for cancer cells?

a. Liver c. Reproductive tract


b. Colon d. White blood cells (WBCs)

5V. Nurse Mandy is preparing a client for magnetic resonance imaging (MRI) to confirm or rule out a spinal cord
lesion. During the MRI scan, which of the following would pose a threat to the client?

a. The client lies still. c. The client hears thumping sounds.


b. The client asks questions. d. The client wears a watch and wedding band.

59. Nurse Cecile is teaching a female client about preventing osteoporosis. Which of the following teaching points is
correct?

a. Obtaining an X-ray of the bones every 3 years is recommended to detect bone loss.
b. To avoid fractures, the client should avoid strenuous exercise.
c. The recommended daily allowance of calcium may be found in a wide variety of foods.
d. Obtaining the recommended daily allowance of calcium requires taking a calcium supplement.

60. Before Jacob undergoes arthroscopy, the nurse reviews the assessment findings for contraindications for this
procedure. Which finding is a contraindication?

a. Joint pain c. Joint flexion of less than 50%


b. Joint deformity d. Joint stiffness

61. Mr. Rodriguez is admitted with severe pain in the knees. Which form of arthritis is characterized by urate
deposits and joint pain, usually in the feet and legs, and occurs primarily in men over age 30?

a. Septic arthritis c. Intermittent arthritis


b. Traumatic arthritis d. Gouty arthritis

62. A heparin infusion at 1,500 unit/hour is ordered for a 64-year-old client with stroke in evolution. The infusion
contains 25,000 units of heparin in 500 ml of saline solution. How many milliliters per hour should be given?

a. 15 ml/hour c. 45 ml/hour
b. 30 ml/hour d. 50 ml/hour

63. A 76-year-old male client had a thromboembolic right stroke; his left arm is swollen. Which of the following
conditions may cause swelling after a stroke?

a. Elbow contracture secondary to spasticity


b. Loss of muscle contraction decreasing venous return
c. Deep vein thrombosis (DVT) due to immobility of the ipsilateral side
d. Hypoalbuminemia due to protein escaping from an inflamed glomerulus
64. Heberden¶s nodes are a common sign of osteoarthritis. Which of the following statement is correct about this
deformity?

a. It appears only in men


b. It appears on the distal interphalangeal joint
c. It appears on the proximal interphalangeal joint
d. It appears on the dorsolateral aspect of the interphalangeal joint.

65. Which of the following statements explains the main difference between rheumatoid arthritis and osteoarthritis?

a. Osteoarthritis is gender-specific, rheumatoid arthritis isn¶t


b. Osteoarthritis is a localized disease rheumatoid arthritis is systemic
c. Osteoarthritis is a systemic disease, rheumatoid arthritis is localized
d. Osteoarthritis has dislocations and subluxations, rheumatoid arthritis doesn¶t

66. Mrs. Cruz uses a cane for assistance in walking. Which of the following statements is true about a cane or other
assistive devices?

a. A walker is a better choice than a cane.


b. The cane should be used on the affected side
c. The cane should be used on the unaffected side
d. A client with osteoarthritis should be encouraged to ambulate without the cane

67. A male client with type 1 diabetes is scheduled to receive 30 U of 70/30 insulin. There is no 70/30 insulin
available. As a substitution, the nurse may give the client:

a. 9 U regular insulin and 21 U neutral protamine Hagedorn (NPH).


b. 21 U regular insulin and 9 U NPH.
c. 10 U regular insulin and 20 U NPH.
d. 20 U regular insulin and 10 U NPH.

6V. Nurse Len should expect to administer which medication to a client with gout?

a. aspirin c. colchicines
b. furosemide (Lasix) d. calcium gluconate (Kalcinate)

69. Mr. Domingo with a history of hypertension is diagnosed with primary hyperaldosteronism. This diagnosis
indicates that the client's hypertension is caused by excessive hormone secretion from which of the following
glands?

a. Adrenal cortex c. Adrenal medulla


b. Pancreas d. Parathyroid

70. For a diabetic male client with a foot ulcer, the doctor orders bed rest, a wetto- dry dressing change every shift,
and blood glucose monitoring before meals and bedtime. Why are wet-to-dry dressings used for this client?

a. They contain exudate and provide a moist wound environment.


b. They protect the wound from mechanical trauma and promote healing.
c. They debride the wound and promote healing by secondary intention.
d. They prevent the entrance of microorganisms and minimize wound discomfort.

71. Nurse Zeny is caring for a client in acute addisonian crisis. Which laboratory data would the nurse expect to
find?

a. Hyperkalemia c. Hypernatremia
b. Reduced blood urea nitrogen (BUN) d. Hyperglycemia
72. A client is admitted for treatment of the syndrome of inappropriate antidiuretic hormone (SIADH). Which
nursing intervention is appropriate?

a. Infusing I.V. fluids rapidly as ordered


b. Encouraging increased oral intake
c. Restricting fluids
d. Administering glucose-containing I.V. fluids as ordered

73. A female client tells nurse Nikki that she has been working hard for the last 3 months to control her type 2
diabetes mellitus with diet and exercise. To determine the effectiveness of the client's efforts, the nurse should
check:

a. urine glucose level. c. serum fructosamine level.


b. fasting blood glucose level. d. glycosylated hemoglobin level.

74. Nurse Trinity administered neutral protamine Hagedorn (NPH) insulin to a diabetic client at 7 a.m. At what time
would the nurse expect the client to be most at risk for a hypoglycemic reaction?

a. 10:00 am c. 4:00 pm
b. Noon d. 10:00 pm

75. The adrenal cortex is responsible for producing which substances?

a. Glucocorticoids and androgens c. Mineralocorticoids and catecholamines


b. Catecholamines and epinephrine d. Norepinephrine and epinephrine

76. On the third day after a partial thyroidectomy, Proserfina exhibits muscle twitching and hyperirritability of the
nervous system. When questioned, the client reports numbness and tingling of the mouth and fingertips. Suspecting
a lifethreatening electrolyte disturbance, the nurse notifies the surgeon immediately. Which electrolyte disturbance
most commonly follows thyroid surgery?

a. Hypocalcemia c. Hyperkalemia
b. Hyponatremia d. Hypermagnesemia

77. Which laboratory test value is elevated in clients who smoke and can't be used as a general indicator of cancer?

a. Acid phosphatase level c. Alkaline phosphatase level


b. Serum calcitonin level d. Carcinoembryonic antigen level

7V. Francis with anemia has been admitted to the medical-surgical unit. Which assessment findings are characteristic
of iron-deficiency anemia?

a. Nights sweats, weight loss, and diarrhea c. Nausea, vomiting, and anorexia
b. Dyspnea, tachycardia, and pallor d. Itching, rash, and jaundice

79. In teaching a female client who is HIV-positive about pregnancy, the nurse would know more teaching is
necessary when the client says:

a. The baby can get the virus from my placenta."


b. "I'm planning on starting on birth control pills."
c. "Not everyone who has the virus gives birth to a baby who has the virus."
d. "I'll need to have a C-section if I become pregnant and have a baby."
V0. When preparing Judy with acquired immunodeficiency syndrome (AIDS) for discharge to the home, the nurse
should be sure to include which instruction?

a. "Put on disposable gloves before bathing."


b. "Sterilize all plates and utensils in boiling water."
c. "Avoid sharing such articles as toothbrushes and razors."
d. "Avoid eating foods from serving dishes shared by other family members."

V1. Nurse Marie is caring for a 32-year-old client admitted with pernicious anemia. Which set of findings should the
nurse expect when assessing the client?

a. Pallor, bradycardia, and reduced pulse pressure c. Sore tongue, dyspnea, and weight gain
b. Pallor, tachycardia, and a sore tongue d. Angina, double vision, and anorexia

V2. After receiving a dose of penicillin, a client develops dyspnea and hypotension. Nurse Celestina suspects the
client is experiencing anaphylactic shock. What should the nurse do first?

a. Page an anesthesiologist immediately and prepare to intubate the client.


b. Administer epinephrine, as prescribed, and prepare to intubate the client if necessary.
c. Administer the antidote for penicillin, as prescribed, and continue to monitor the client's vital signs.
d. Insert an indwelling urinary catheter and begin to infuse I.V. fluids as ordered.

V3. Mr. Marquez with rheumatoid arthritis is about to begin aspirin therapy to reduce inflammation. When teaching
the client about aspirin, the nurse discusses adverse reactions to prolonged aspirin therapy. These include:

a. weight gain. c. respiratory acidosis.


b. fine motor tremors. d. bilateral hearing loss.

V4. A 23-year-old client is diagnosed with human immunodeficiency virus (HIV). After recovering from the initial
shock of the diagnosis, the client expresses a desire to learn as much as possible about HIV and acquired
immunodeficiency syndrome (AIDS). When teaching the client about the immune system, the nurse states that
adaptive immunity is provided by which type of white blood cell?

a. Neutrophil c. Monocyte
b. Basophil d. Lymphocyte

V5. In an individual with Sjogren's syndrome, nursing care should focus on:

a. moisture replacement. c. nutritional supplementation.


b. electrolyte balance. d. arrhythmia management.

V6. During chemotherapy for lymphocytic leukemia, Mathew develops abdominal pain, fever, and "horse barn"
smelling diarrhea. It would be most important for the nurse to advise the physician to order:

a. enzyme-linked immunosuppressant assay (ELISA) test.


b. electrolyte panel and hemogram.
c. stool for V  

  test.
d. flat plate X-ray of the abdomen.

V7. A male client seeks medical evaluation for fatigue, night sweats, and a 20-lb weight loss in 6 weeks. To confirm
that the client has been infected with the human immunodeficiency virus (HIV), the nurse expects the physician to
order:

a. E-rosette immunofluorescence. c. enzyme-linked immunosorbent assay (ELISA).


b. quantification of T-lymphocytes. d. Western blot test with ELISA.
VV. A complete blood count is commonly performed before a Joe goes into surgery. What does this test seek to
identify?

a. Potential hepatic dysfunction indicated by decreased blood urea nitrogen (BUN) and creatinine levels
b. Low levels of urine constituents normally excreted in the urine
c. Abnormally low hematocrit (HCT) and hemoglobin (Hb) levels
d. Electrolyte imbalance that could affect the blood's ability to coagulate properly

V9. While monitoring a client for the development of disseminated intravascular coagulation (DIC), the nurse should
take note of what assessment parameters?

a. Platelet count, prothrombin time, and partial thromboplastin time


b. Platelet count, blood glucose levels, and white blood cell (WBC) count
c. Thrombin time, calcium levels, and potassium levels
d. Fibrinogen level, WBC, and platelet count

90. When taking a dietary history from a newly admitted female client, Nurse Len should remember that which of
the following foods is a common allergen?

a. Bread c. Orange
b. Carrots d. Strawberries

91. Nurse John is caring for clients in the outpatient clinic. Which of the following phone calls should the nurse
return first?

a. A client with hepatitis A who states, ³My arms and legs are itching.´
b. A client with cast on the right leg who states, ³I have a funny feeling in my right leg.´
c. A client with osteomyelitis of the spine who states, ³I am so nauseous that I can¶t eat.´
d. A client with rheumatoid arthritis who states, ³I am having trouble sleeping.´

92. Nurse Sarah is caring for clients on the surgical floor and has just received report from the previous shift. Which
of the following clients should the nurse see first?

a. A 35-year-old admitted three hours ago with a gunshot wound; 1.5 cm area of dark drainage noted on the
dressing.
b. A 43-year-old who had a mastectomy two days ago; 23 ml of serosanguinous fluid noted in the Jackson-Pratt
drain.
c. A 59-year-old with a collapsed lung due to an accident; no drainage noted in the previous eight hours.
d. A 62-year-old who had an abdominal-perineal resection three days ago; client complaints of chills.

93. Nurse Eve is caring for a client who had a thyroidectomy 12 hours ago for treatment of Grave¶s disease. The
nurse would be most concerned if which of the following was observed?

a. Blood pressure 13V/V2, respirations 16, oral temperature 99 degrees Fahrenheit.


b. The client supports his head and neck when turning his head to the right.
c. The client spontaneously flexes his wrist when the blood pressure is obtained.
d. The client is drowsy and complains of sore throat.

94. Julius is admitted with complaints of severe pain in the lower right quadrant of the abdomen. To assist with pain
relief, the nurse should take which of the following actions?

a. Encourage the client to change positions frequently in bed.


b. Administer Demerol 50 mg IM q 4 hours and PRN.
c. Apply warmth to the abdomen with a heating pad.
d. Use comfort measures and pillows to position the client.
95. Nurse Tina prepares a client for peritoneal dialysis. Which of the following actions should the nurse take first?

a. Assess for a bruit and a thrill. c. Position the client on the left side.
b. Warm the dialysate solution. d. Insert a Foley catheter

96. Nurse Jannah teaches an elderly client with right-sided weakness how to use cane. Which of the following
behaviors, if demonstrated by the client to the nurse, indicates that the teaching was effective?

a. The client holds the cane with his right hand, moves the can forward followed by the right leg, and then moves the
left leg.
b. The client holds the cane with his right hand, moves the cane forward followed by his left leg, and then moves the
right leg.
c. The client holds the cane with his left hand, moves the cane forward followed by the right leg, and then moves the
left leg.
d. The client holds the cane with his left hand, moves the cane forward followed by his left leg, and then moves the
right leg.

97. An elderly client is admitted to the nursing home setting. The client is occasionally confused and her gait is often
unsteady. Which of the following actions, if taken by the nurse, is most appropriate?

a. Ask the woman¶s family to provide personal items such as photos or mementos.
b. Select a room with a bed by the door so the woman can look down the hall.
c. Suggest the woman eat her meals in the room with her roommate.
d. Encourage the woman to ambulate in the halls twice a day.

9V. Nurse Evangeline teaches an elderly client how to use a standard aluminum walker. Which of the following
behaviors, if demonstrated by the client, indicates that the nurse¶s teaching was effective?

a. The client slowly pushes the walker forward 12 inches, then takes small steps forward while leaning on the
walker.
b. The client lifts the walker, moves it forward 10 inches, and then takes several small steps forward.
c. The client supports his weight on the walker while advancing it forward, then takes small steps while balancing on
the walker.
d. The client slides the walker 1V inches forward, then takes small steps while holding onto the walker for balance.

99. Nurse Deric is supervising a group of elderly clients in a residential home setting. The nurse knows that the
elderly are at greater risk of developing sensory deprivation for what reason?

a. Increased sensitivity to the side effects of medications.


b. Decreased visual, auditory, and gustatory abilities.
c. Isolation from their families and familiar surroundings.
d. Decrease musculoskeletal function and mobility.

100. A male client with emphysema becomes restless and confused. What step should nurse Jasmine take next?
a. Encourage the client to perform pursed lip breathing.
b. Check the client¶s temperature.
c. Assess the client¶s potassium level.
d. Increase the client¶s oxygen flow rate.
'   c   c      
1. Randy has undergone kidney transplant, what assessment would prompt Nurse Katrina to suspect organ rejection?

a. Sudden weight loss c. Hypertension


b. Polyuria d. Shock

2. The immediate objective of nursing care for an overweight, mildly hypertensive male client with ureteral colic
and hematuria is to decrease:

a. Pain c. Hematuria
b. Weight d. Hypertension

3. Matilda, with hyperthyroidism is to receive Lugol¶s iodine solution before a subtotal thyroidectomy is performed.
The nurse is aware that this medication is given to:

a. Decrease the total basal metabolic rate.


b. Maintain the function of the parathyroid glands.
c. Block the formation of thyroxine by the thyroid gland.
d. Decrease the size and vascularity of the thyroid gland.

4. Ricardo, was diagnosed with type I diabetes. The nurse is aware that acute hypoglycemia also can develop in the
client who is diagnosed with:

a. Liver disease c. Type 2 diabetes


b. Hypertension d. Hyperthyroidism

5. Tracy is receiving combination chemotherapy for treatment of metastatic carcinoma. Nurse Ruby should monitor
the client for the systemic side effect of:

a. Ascites c. Leukopenia
b. Nystagmus d. Polycythemia

6. Norma, with recent colostomy expresses concern about the inability to control the passage of gas. Nurse Oliver
should suggest that the client plan to:

a. Eliminate foods high in cellulose.


b. Decrease fluid intake at meal times.
c. Avoid foods that in the past caused flatus.
d. Adhere to a bland diet prior to social events.

7. Nurse Ron begins to teach a male client how to perform colostomy irrigations. The nurse would evaluate that the
instructions were understood when the client states, ³I should:

a. Lie on my left side while instilling the irrigating solution.´


b. Keep the irrigating container less than 1V inches above the stoma.´
c. Instill a minimum of 1200 ml of irrigating solution to stimulate evacuation of the bowel.´
d. Insert the irrigating catheter deeper into the stoma if cramping occurs during the procedure.´

V. Patrick is in the oliguric phase of acute tubular necrosis and is experiencing fluid and electrolyte imbalances. The
client is somewhat confused and complains of nausea and muscle weakness. As part of the prescribed therapy to
correct this electrolyte imbalance, the nurse would expect to:

a. Administer Kayexalate
b. Restrict foods high in protein
c. Increase oral intake of cheese and milk.
d. Administer large amounts of normal saline via I.V.
9. Mario has burn injury. After Forty4V hours, the physician orders for Mario 2 liters of IV fluid to be administered
q12 h. The drop factor of the tubing is 10 gtt/ml. The nurse should set the flow to provide:

a. 1V gtt/min c. 32 gtt/min
b. 2V gtt/min d. 36 gtt/min

10.Terence suffered form burn injury. Using the rule of nines, which has the largest percent of burns?

a. Face and neck c. Right thigh and penis


b. Right upper arm and penis d. Upper trunk

11. Herbert, a 45 year old construction engineer is brought to the hospital unconscious after falling from a 2-story
building. When assessing the client, the nurse would be most concerned if the assessment revealed:

a. Reactive pupils c. Bleeding from ears


b. A depressed fontanel d. An elevated temperature

12. Nurse Sherry is teaching male client regarding his permanent artificial pacemaker. Which information given by
the nurse shows her knowledge deficit about the artificial cardiac pacemaker?

a. take the pulse rate once a day, in the morning upon awakening
b. May be allowed to use electrical appliances
c. Have regular follow up care
d. May engage in contact sports

13.The nurse is ware that the most relevant knowledge about oxygen administration to a male client with COPD is

a. Oxygen at 1-2L/min is given to maintain the hypoxic stimulus for breathing.


b. Hypoxia stimulates the central chemoreceptors in the medulla that makes the client breath.
c. Oxygen is administered best using a non-rebreathing mask
d. Blood gases are monitored using a pulse oximeter.

14.Tonny has undergoes a left thoracotomy and a partial pneumonectomy. Chest tubes are inserted, and one-bottle
water-seal drainage is instituted in the operating room. In the postanesthesia care unit Tonny is placed in Fowler's
position on either his right side or on his back. The nurse is aware that this position:

a. Reduce incisional pain. c. Equalize pressure in the pleural space.


b. Facilitate ventilation of the left lung. d. Increase venous return

15.Kristine is scheduled for a bronchoscopy. When teaching Kristine what to expect afterward, the nurse's highest
priority of information would be:

a. Food and fluids will be withheld for at least 2 hours.


b. Warm saline gargles will be done q 2h.
c. Coughing and deep-breathing exercises will be done q2h.
d. Only ice chips and cold liquids will be allowed initially.

16.Nurse Tristan is caring for a male client in acute renal failure. The nurse should expect hypertonic glucose,
insulin infusions, and sodium bicarbonate to be used to treat:

a. hypernatremia. c. hyperkalemia.
b. hypokalemia. d. hypercalcemia.
17.Ms. X has just been diagnosed with condylomata acuminata (genital warts). What information is appropriate to
tell this client?

a. This condition puts her at a higher risk for cervical cancer; therefore, she should have a Papanicolaou (Pap) smear
annually.
b. The most common treatment is metronidazole (Flagyl), which should eradicate the problem within 7 to 10 days.
c. The potential for transmission to her sexual partner will be eliminated if condoms are used every time they have
sexual intercourse.
d. The human papillomavirus (HPV), which causes condylomata acuminata, can't be transmitted during oral sex.

1V.Maritess was recently diagnosed with a genitourinary problem and is being examined in the emergency
department. When palpating the her kidneys, the nurse should keep which anatomical fact in mind?

a. The left kidney usually is slightly higher than the right one.
b. The kidneys are situated just above the adrenal glands.
c. The average kidney is approximately 5 cm (2") long and 2 to 3 cm (." to 1-1/V") wide.
d. The kidneys lie between the 10th and 12th thoracic vertebrae.

19.Jestoni with chronic renal failure (CRF) is admitted to the urology unit. The nurse is aware that the diagnostic test
are consistent with CRF if the result is:

a. Increased pH with decreased hydrogen ions.


b. Increased serum levels of potassium, magnesium, and calcium.
c. Blood urea nitrogen (BUN) 100 mg/dl and serum creatinine 6.5 mg/ dl.
d. Uric acid analysis 3.5 mg/dl and phenolsulfonphthalein (PSP) excretion 75%.

20. Katrina has an abnormal result on a Papanicolaou test. After admitting that she read her chart while the nurse
was out of the room, Katrina asks what    means. Which definition should the nurse provide?

a. Presence of completely undifferentiated tumor cells that don't resemble cells of the tissues of their origin.
b. Increase in the number of normal cells in a normal arrangement in a tissue or an organ.
c. Replacement of one type of fully differentiated cell by another in tissues where the second type normally isn't
found.
d. Alteration in the size, shape, and organization of differentiated cells.

21. During a routine checkup, Nurse Mariane assesses a male client with acquired immunodeficiency syndrome
(AIDS) for signs and symptoms of cancer. What is the most common AIDS-related cancer?

a. Squamous cell carcinoma c. Leukemia


b. Multiple myeloma d. Kaposi's sarcoma

22.Ricardo is scheduled for a prostatectomy, and the anesthesiologist plans to use a spinal (subarachnoid) block
during surgery. In the operating room, the nurse positions the client according to the anesthesiologist's instructions.
Why does the client require special positioning for this type of anesthesia?

a. To prevent confusion c. To prevent cerebrospinal fluid (CSF) leakage


b. To prevent seizures d. To prevent cardiac arrhythmias

23.A male client had a nephrectomy 2 days ago and is now complaining of abdominal pressure and nausea. The first
nursing action should be to:

a. Auscultate bowel sounds. c. Change the client's position.


b. Palpate the abdomen. d. Insert a rectal tube.
24.Wilfredo with a recent history of rectal bleeding is being prepared for a colonoscopy. How should the nurse
Patricia position the client for this test initially?

a. Lying on the right side with legs straight c. Prone with the torso elevated
b. Lying on the left side with knees bent d. Bent over with hands touching the floor

25.A male client with inflammatory bowel disease undergoes an ileostomy. On the first day after surgery, Nurse
Oliver notes that the client's stoma appears dusky. How should the nurse interpret this finding?

a. Blood supply to the stoma has been interrupted. c. The ostomy bag should be adjusted.
b. This is a normal finding 1 day after surgery. d. An intestinal obstruction has occurred.

26.Anthony suffers burns on the legs, which nursing intervention helps prevent contractures?

a. Applying knee splints c. Hyperextending the client's palms


b. Elevating the foot of the bed d. Performing shoulder range-of-motion exercises

27.Nurse Ron is assessing a client admitted with second- and third-degree burns on the face, arms, and chest. Which
finding indicates a potential problem?

a. Partial pressure of arterial oxygen (PaO2) value of V0 mm Hg.


b. Urine output of 20 ml/hour.
c. White pulmonary secretions.
d. Rectal temperature of 100.6° F (3V° C).

2V. Mr. Mendoza who has suffered a cerebrovascular accident (CVA) is too weak to move on his own. To help the
client avoid pressure ulcers, Nurse Celia should:

a. Turn him frequently.


b. Perform passive range-of-motion (ROM) exercises.
c. Reduce the client's fluid intake.
d. Encourage the client to use a footboard.

29.Nurse Maria plans to administer dexamethasone cream to a female client who has dermatitis over the anterior
chest. How should the nurse apply this topical agent?

a. With a circular motion, to enhance absorption.


b. With an upward motion, to increase blood supply to the affected area
c. In long, even, outward, and downward strokes in the direction of hair growth
d. In long, even, outward, and upward strokes in the direction opposite hair growth

30.Nurse Kate is aware that one of the following classes of medication protect the ischemic myocardium by
blocking catecholamines and sympathetic nerve stimulation is:

a. Beta -adrenergic blockers c. Narcotics


b. Calcium channel blocker d. Nitrates

31.A male client has jugular distention. On what position should the nurse place the head of the bed to obtain the
most accurate reading of jugular vein distention?

a. High Fowler¶s c. Raised 30 degrees


b. Raised 10 degrees d. Supine position
32.The nurse is aware that one of the following classes of medications maximizes cardiac performance in clients
with heart failure by increasing ventricular contractility?

a. Beta-adrenergic blockers c. Diuretics


b. Calcium channel blocker d. Inotropic agents

33.A male client has a reduced serum high-density lipoprotein (HDL) level and an elevated low-density lipoprotein
(LDL) level. Which of the following dietary modifications is not appropriate for this client?

a. Fiber intake of 25 to 30 g daily c. Cholesterol intake of less than 300 mg daily


b. Less than 30% of calories form fat d. Less than 10% of calories from saturated fat

34. A 37-year-old male client was admitted to the coronary care unit (CCU) 2 days ago with an acute myocardial
infarction. Which of the following actions would breach the client confidentiality?

a. The CCU nurse gives a verbal report to the nurse on the telemetry unit before transferring the client to that unit
b. The CCU nurse notifies the on-call physician about a change in the client¶s condition
c. The emergency department nurse calls up the latest electrocardiogram results to check the client¶s progress.
d. At the client¶s request, the CCU nurse updates the client¶s wife on his condition

35. A male client arriving in the emergency department is receiving cardiopulmonary resuscitation from paramedics
who are giving ventilations through an endotracheal (ET) tube that they placed in the client¶s home. During a pause
in compressions, the cardiac monitor shows narrow QRS complexes and a heart rate of beats/minute with a palpable
pulse. Which of the following actions should the nurse take first?

a. Start an L.V. line and administer amiodarone (Cardarone), 300 mg L.V. over 10 minutes.
b. Check endotracheal tube placement.
c. Obtain an arterial blood gas (ABG) sample.
d. Administer atropine, 1 mg L.V.

36. After cardiac surgery, a client¶s blood pressure measures 126/V0 mm Hg. Nurse Katrina determines that mean
arterial pressure (MAP) is which of the following?

a. 46 mm Hg c. 95 mm Hg
b. V0 mm Hg d. 90 mm Hg

37. A female client arrives at the emergency department with chest and stomach pain and a report of black tarry
stool for several months. Which of the following order should the nurse Oliver anticipate?

a. Cardiac monitor, oxygen, creatine kinase and lactate dehydrogenase levels


b. Prothrombin time, partial thromboplastin time, fibrinogen and fibrin split product values.
c. Electrocardiogram, complete blood count, testing for occult blood, comprehensive serum metabolic panel.
d. Electroencephalogram, alkaline phosphatase and aspartate aminotransferase levels, basic serum metabolic panel

3V. Macario had coronary artery bypass graft (CABG) surgery 3 days ago. Which of the following conditions is
suspected by the nurse when a decrease in platelet count from 230,000 ul to 5,000 ul is noted?

a. Pancytopenia
b. Idiopathic thrombocytopemic purpura (ITP)
c. Disseminated intravascular coagulation (DIC)
d. Heparin-associated thrombosis and thrombocytopenia (HATT)
39. Which of the following drugs would be ordered by the physician to improve the platelet count in a male client
with idiopathic thrombocytopenic purpura (ITP)?

a. Acetylsalicylic acid (ASA) c. Methotrezate


b. Corticosteroids d. Vitamin K

40. A female client is scheduled to receive a heart valve replacement with a porcine valve. Which of the following
types of transplant is this?

a. Allogeneic c. Syngeneic
b. Autologous d. Xenogeneic

41. Marco falls off his bicycle and injuries his ankle. Which of the following actions shows the initial response to
the injury in the extrinsic pathway?

a. Release of Calcium c. Conversion of factors XII to factor XIIa


b. Release of tissue thromboplastin d. Conversion of factor VIII to factor VIIIa

42. Instructions for a client with systemic lupus erythematosus (SLE) would include information about which of the
following blood dyscrasias?

a. Dressler¶s syndrome c. Essential thrombocytopenia


b. Polycythemia d. Von Willebrand¶s disease

43. The nurse is aware that the following symptoms is most commonly an early indication of stage 1 Hodgkin¶s
disease?

a. Pericarditis c. Splenomegaly
b. Night sweat d. Persistent hypothermia

44. Francis with leukemia has neutropenia. Which of the following functions must frequently assessed?

a. Blood pressure c. Heart sounds


b. Bowel sounds d. Breath sounds

45. The nurse knows that neurologic complications of multiple myeloma (MM) usually involve which of the
following body system?

a. Brain c. Renal dysfunction


b. Muscle spasm d. Myocardial irritability

46. Nurse Patricia is aware that the average length of time from human immunodeficiency virus (HIV) infection to
the development of acquired immunodeficiency syndrome (AIDS)?

a. Less than 5 years c. 10 years


b. 5 to 7 years d. More than 10 years

47. An 1V-year-old male client admitted with heat stroke begins to show signs of disseminated intravascular
coagulation (DIC). Which of the following laboratory findings is most consistent with DIC?

a. Low platelet count c. Low levels of fibrin degradation products


b. Elevated fibrinogen levels d. Reduced prothrombin time
4V. Mario comes to the clinic complaining of fever, drenching night sweats, and unexplained weight loss over the
past 3 months. Physical examination reveals a single enlarged supraclavicular lymph node. Which of the following
is the most probable diagnosis?

a. Influenza c. Leukemia
b. Sickle cell anemia d. Hodgkin¶s disease

49. A male client with a gunshot wound requires an emergency blood transfusion. His blood type is AB negative.
Which blood type would be the safest for him to receive?

a. AB Rh-positive c. A Rh-negative
b. A Rh-positive d. O Rh-positive

Situation: Stacy is diagnosed with acute lymphoid leukemia (ALL) and beginning chemotherapy.

50. Stacy is discharged from the hospital following her chemotherapy treatments. Which statement of Stacy¶s
mother indicated that she understands when she will contact the physician?

a. ³I should contact the physician if Stacy has difficulty in sleeping´.


b. ³I will call my doctor if Stacy has persistent vomiting and diarrhea´.
c. ³My physician should be called if Stacy is irritable and unhappy´.
d. ³Should Stacy have continued hair loss, I need to call the doctor´.

51. Stacy¶s mother states to the nurse that it is hard to see Stacy with no hair. The best response for the nurse is:

a. ³Stacy looks very nice wearing a hat´.


b. ³You should not worry about her hair, just be glad that she is alive´.
c. ³Yes it is upsetting. But try to cover up your feelings when you are with her or else she may be upset´.
d. ³This is only temporary; Stacy will re-grow new hair in 3-6 months, but may be different in texture´.

52. Stacy has beginning stomatitis. To promote oral hygiene and comfort, the nurse in-charge should:

a. Provide frequent mouthwash with normal saline.


b. Apply viscous Lidocaine to oral ulcers as needed.
c. Use lemon glycerine swabs every 2 hours.
d. Rinse mouth with Hydrogen Peroxide.

53. During the administration of chemotherapy agents, Nurse Oliver observed that the IV site is red and swollen,
when the IV is touched Stacy shouts in pain. The first nursing action to take is:

a. Notify the physician


b. Flush the IV line with saline solution
c. Immediately discontinue the infusion
d. Apply an ice pack to the site, followed by warm compress.

54. The term ³blue bloater´ refers to a male client which of the following conditions?

a. Adult respiratory distress syndrome (ARDS) c. Chronic obstructive bronchitis


b. Asthma d. Emphysema

55. The term ³pink puffer´ refers to the female client with which of the following conditions?

a. Adult respiratory distress syndrome (ARDS) c. Chronic obstructive bronchitis


b. Asthma d. Emphysema
56. Jose is in danger of respiratory arrest following the administration of a narcotic analgesic. An arterial blood gas
value is obtained. Nurse Oliver would expect the paco2 to be which of the following values?

a. 15 mm Hg c. 40 mm Hg
b. 30 mm Hg d. V0 mm Hg

57. Timothy¶s arterial blood gas (ABG) results are as follows; pH 7.16; Paco2 V0 mm Hg; Pao2 46 mm Hg; HCO3-
24mEq/L; Sao2 V1%. This ABG result represents which of the following conditions?
c. Respiratory acidosis
a. Metabolic acidosis d. Respirator y alkalosis
b. Metabolic alkalosis

5V. Norma has started a new drug for hypertension. Thirty minutes after she takes the drug, she develops chest
tightness and becomes short of breath and tachypneic. She has a decreased level of consciousness. These signs
indicate which of the following conditions?

a. Asthma attack c. Respiratory failure


b. Pulmonary embolism d. Rheumatoid arthritis

Situation: Mr. Gonzales was admitted to the hospital with ascites and jaundice. To rule out cirrhosis of the liver:

59. Which laboratory test indicates liver cirrhosis?

a. Decreased red blood cell count c. Elevated white blood cell count
b. Decreased serum acid phosphate level d. Elevated serum aminotransferase

60.The biopsy of Mr. Gonzales confirms the diagnosis of cirrhosis. Mr. Gonzales is at increased risk for excessive
bleeding primarily because of:

a. Impaired clotting mechanism c. Inadequate nutrition


b. Varix formation d. Trauma of invasive procedure

61. Mr. Gonzales develops hepatic encephalopathy. Which clinical manifestation is most common with this
condition?

a. Increased urine output c. Decreased tendon reflex


b. Altered level of consciousness d. Hypotension

62. When Mr. Gonzales regained consciousness, the physician orders 50 ml of Lactose p.o. every 2 hours. Mr.
Gozales develops diarrhea. The nurse best action would be:

a. ³I¶ll see if your physician is in the hospital´.


b. ³Maybe your reacting to the drug; I will withhold the next dose´.
c. ³I¶ll lower the dosage as ordered so the drug causes only 2 to 4 stools a day´.
d. ³Frequently, bowel movements are needed to reduce sodium level´.

63. Which of the following groups of symptoms indicates a ruptured abdominal aortic aneurysm?

a. Lower back pain, increased blood pressure, decreased re blood cell (RBC) count, increased white blood (WBC)
count.
b. Severe lower back pain, decreased blood pressure, decreased RBC count, increased WBC count.
c. Severe lower back pain, decreased blood pressure, decreased RBC count, decreased RBC count, decreased WBC
count.
d. Intermitted lower back pain, decreased blood pressure, decreased RBC count, increased WBC count.
64. After undergoing a cardiac catheterization, Tracy has a large puddle of blood under his buttocks. Which of the
following steps should the nurse take first?

a. Call for help. c. Ask the client to ³lift up´


b. Obtain vital signs d. Apply gloves and assess the groin site

65. Which of the following treatment is a suitable surgical intervention for a client with unstable angina?

a. Cardiac catheterization
b. Echocardiogram
c. Nitroglycerin
d. Percutaneous transluminal coronary angioplasty (PTCA)

66. The nurse is aware that the following terms used to describe reduced cardiac output and perfusion impairment
due to ineffective pumping of the heart is:

a. Anaphylactic shock c. Distributive shock


b. Cardiogenic shock d. Myocardial infarction (MI)

67. A client with hypertension ask the nurse which factors can cause blood pressure to drop to normal levels?

a. Kidneys¶ excretion to sodium only.


b. Kidneys¶ retention of sodium and water
c. Kidneys¶ excretion of sodium and water
d. Kidneys¶ retention of sodium and excretion of water

6V. Nurse Rose is aware that the statement that best explains why furosemide (Lasix) is administered to treat
hypertension is:

a. It dilates peripheral blood vessels.


b. It decreases sympathetic cardioacceleration.
c. It inhibits the angiotensin-coverting enzymes
d. It inhibits reabsorption of sodium and water in the loop of Henle.

69. Nurse Nikki knows that laboratory results supports the diagnosis of systemic lupus erythematosus (SLE) is:

a. Elavated serum complement level


b. Thrombocytosis, elevated sedimentation rate
c. Pancytopenia, elevated antinuclear antibody (ANA) titer
d. Leukocysis, elevated blood urea nitrogen (BUN) and creatinine levels

70. Arnold, a 19-year-old client with a mild concussion is discharged from the emergency department. Before
discharge, he complains of a headache. When offered acetaminophen, his mother tells the nurse the headache is
severe and she would like her son to have something stronger. Which of the following responses by the nurse is
appropriate?

a. ³Your son had a mild concussion, acetaminophen is strong enough.´


b. ³Aspirin is avoided because of the danger of Reye¶s syndrome in children or young adults.´
c. ³Narcotics are avoided after a head injury because they may hide a worsening condition.´
d. Stronger medications may lead to vomiting, which increases the intracarnial pressure (ICP).´
71. When evaluating an arterial blood gas from a male client with a subdural hematoma, the nurse notes the Paco2 is
30 mm Hg. Which of the following responses best describes the result?

a. Appropriate; lowering carbon dioxide (CO2) reduces intracranial pressure (ICP)


b. Emergent; the client is poorly oxygenated
c. Normal
d. Significant; the client has alveolar hypoventilation

72. When prioritizing care, which of the following clients should the nurse Olivia assess first?

a. A 17-year-old clients 24-hours postappendectomy


b. A 33-year-old client with a recent diagnosis of Guillain-Barre syndrome
c. A 50-year-old client 3 days postmyocardial infarction
d. A 50-year-old client with diverticulitis

73. JP has been diagnosed with gout and wants to know why colchicine is used in the treatment of gout. Which of
the following actions of colchicines explains why it¶s effective for gout?

a. Replaces estrogen c. Decreases inflammation


b. Decreases infection d. Decreases bone demineralization

74. Norma asks for information about osteoarthritis. Which of the following statements about osteoarthritis is
correct?

a. Osteoarthritis is rarely debilitating


b. Osteoarthritis is a rare form of arthritis
c. Osteoarthritis is the most common form of arthritis
d. Osteoarthritis afflicts people over 60

75. Ruby is receiving thyroid replacement therapy develops the flu and forgets to take her thyroid replacement
medicine. The nurse understands that skipping this medication will put the client at risk for developing which of the
following lifethreatening complications?

a. Exophthalmos c. Myxedema coma


b. Thyroid storm d. Tibial myxedema

76. Nurse Sugar is assessing a client with Cushing's syndrome. Which observation should the nurse report to the
physician immediately?

a. Pitting edema of the legs c. Dry mucous membranes


b. An irregular apical pulse d. Frequent urination

77. Cyrill with severe head trauma sustained in a car accident is admitted to the intensive care unit. Thirty-six hours
later, the client's urine output suddenly rises above 200 ml/hour, leading the nurse to suspect diabetes insipidus.
Which laboratory findings support the nurse's suspicion of diabetes insipidus?

a. Above-normal urine and serum osmolality levels


b. Below-normal urine and serum osmolality levels
c. Above-normal urine osmolality level, below-normal serum osmolality level
d. Below-normal urine osmolality level, above-normal serum osmolality level
7V. Jomari is diagnosed with hyperosmolar hyperglycemic nonketotic syndrome (HHNS) is stabilized and prepared
for discharge. When preparing the client for discharge and home management, which of the following statements
indicates that the client understands her condition and how to control it?

a. "I can avoid getting sick by not becoming dehydrated and by paying attention to my need to urinate, drink, or eat
more than usual."
b. "If I experience trembling, weakness, and headache, I should drink a glass of soda that contains sugar."
c. "I will have to monitor my blood glucose level closely and notify the physician if it's constantly elevated."
d. "If I begin to feel especially hungry and thirsty, I'll eat a snack high in carbohydrates."

79. A 66-year-old client has been complaining of sleeping more, increased urination, anorexia, weakness, irritability,
depression, and bone pain that interferes with her going outdoors. Based on these assessment findings, the nurse
would suspect which of the following disorders?

a. Diabetes mellitus c. Hypoparathyroidism


b. Diabetes insipidus d. Hyperparathyroidism

V0. Nurse Lourdes is teaching a client recovering from addisonian crisis about the need to take fludrocortisone
acetate and hydrocortisone at home. Which statement by the client indicates an understanding of the instructions?

a. "I'll take my hydrocortisone in the late afternoon, before dinner."


b. "I'll take all of my hydrocortisone in the morning, right after I wake up."
c. "I'll take two-thirds of the dose when I wake up and one-third in the late afternoon."
d. "I'll take the entire dose at bedtime."

V1..Which of the following laboratory test results would suggest to the nurse Len that a client has a corticotropin-
secreting pituitary adenoma?

a. High corticotropin and low cortisol levels c. High corticotropin and high cortisol levels
b. Low corticotropin and high cortisol levels d. Low corticotropin and low cortisol levels

V2. A male client is scheduled for a transsphenoidal hypophysectomy to remove a pituitary tumor. Preoperatively,
the nurse should assess for potential complications by doing which of the following?

a. Testing for ketones in the urine


b. Testing urine specific gravity
c. Checking temperature every 4 hours
d. Performing capillary glucose testing every 4 hours

V3. Capillary glucose monitoring is being performed every 4 hours for a client diagnosed with diabetic ketoacidosis.
Insulin is administered using a scale of regular insulin according to glucose results. At 2 p.m., the client has a
capillary glucose level of 250 mg/dl for which he receives V U of regular insulin. Nurse Mariner should expect the
dose's:

a. onset to be at 2 p.m. and its peak to be at 3 p.m.


b. onset to be at 2:15 p.m. and its peak to be at 3 p.m.
c. onset to be at 2:30 p.m. and its peak to be at 4 p.m.
d. onset to be at 4 p.m. and its peak to be at 6 p.m.

V4. The physician orders laboratory tests to confirm hyperthyroidism in a female client with classic signs and
symptoms of this disorder. Which test result would confirm the diagnosis?

a. No increase in the thyroid-stimulating hormone (TSH) level after 30 minutes during the TSH stimulation test
b. A decreased TSH level
c. An increase in the TSH level after 30 minutes during the TSH stimulation test
d. Below-normal levels of serum triiodothyronine (T3) and serum thyroxine (T4) as detected by radioimmunoassay
V5. Rico with diabetes mellitus must learn how to self-administer insulin. The physician has prescribed 10 U of U
100 regular insulin and 35 U of U-100 isophane insulin suspension (NPH) to be taken before breakfast. When
teaching the client how to select and rotate insulin injection sites, the nurse should provide which instruction?

a. "Inject insulin into healthy tissue with large blood vessels and nerves."
b. "Rotate injection sites within the same anatomic region, not among different regions."
c. "Administer insulin into areas of scar tissue or hypotrophy whenever possible."
d. "Administer insulin into sites above muscles that you plan to exercise heavily later that day."

V6. Nurse Sarah expects to note an elevated serum glucose level in a client with hyperosmolar hyperglycemic
nonketotic syndrome (HHNS). Which other laboratory finding should the nurse anticipate?

a. Elevated serum acetone level c. Serum alkalosis


b. Serum ketone bodies d. Below-normal serum potassium level

V7. For a client with Graves' disease, which nursing intervention promotes comfort?

a. Restricting intake of oral fluids


b. Placing extra blankets on the client's bed
c. Limiting intake of high-carbohydrate foods
d. Maintaining room temperature in the low-normal range

VV. Patrick is treated in the emergency department for a Colles' fracture sustained during a fall. What is a Colles'
fracture?

a. Fracture of the distal radius c. Fracture of the humerus


b. Fracture of the olecranon d. Fracture of the carpal scaphoid

V9. Cleo is diagnosed with osteoporosis. Which electrolytes are involved in the development of this disorder?

a. Calcium and sodium c. Phosphorous and potassium


b. Calcium and phosphorous d. Potassium and sodium

90. Johnny a firefighter was involved in extinguishing a house fire and is being treated to smoke inhalation. He
develops severe hypoxia 4V hours after the incident, requiring intubation and mechanical ventilation. He most likely
has developed which of the following conditions?

a. Adult respiratory distress syndrome (ARDS) c. Bronchitis


b. Atelectasis d. Pneumonia

91. A 67-year-old client develops acute shortness of breath and progressive hypoxia requiring right femur. The
hypoxia was probably caused by which of the following conditions?

a. Asthma attack c. Bronchitis


b. Atelectasis d. Fat embolism

92. A client with shortness of breath has decreased to absent breath sounds o the right side, from the apex to the
base. Which of the following conditions would best explain this?

a. Acute asthma c. Pneumonia


b. Chronic bronchitis d. Spontaneous pneumothorax
93. A 62-year-old male client was in a motor vehicle accident as an unrestrained driver. He¶s now in the emergency
department complaining of difficulty of breathing and chest pain. On auscultation of his lung field, no breath sounds
are present in the upper lobe. This client may have which of the following conditions?

a. Bronchitis c. Pneumothorax
b. Pneumonia d. Tuberculosis (TB)

94. If a client requires a pneumonectomy, what fills the area of the thoracic cavity?

a. The space remains filled with air only


b. The surgeon fills the space with a gel
c. Serous fluids fills the space and consolidates the region
d. The tissue from the other lung grows over to the other side

95. Hemoptysis may be present in the client with a pulmonary embolism because of which of the following reasons?

a. Alveolar damage in the infracted area


b. Involvement of major blood vessels in the occluded area
c. Loss of lung parenchyma
d. Loss of lung tissue

96. Aldo with a massive pulmonary embolism will have an arterial blood gas analysis performed to determine the
extent of hypoxia. The acid-base disorder that may be present is?

a. Metabolic acidosis c. Respiratory acidosis


b. Metabolic alkalosis d. Respiratory alkalosis

97. After a motor vehicle accident, Armand an 22-year-old client is admitted with a pneumothorax. The surgeon
inserts a chest tube and attaches it to a chest drainage system. Bubbling soon appears in the water seal chamber.
Which of the following is the most likely cause of the bubbling?

a. Air leak c. Inadequate suction


b. Adequate suction d. Kinked chest tube

9V. Nurse Michelle calculates the IV flow rate for a postoperative client. The client receives 3,000 ml of Ringer¶s
lactate solution IV to run over 24 hours. The IV infusion set has a drop factor of 10 drops per milliliter. The nurse
should regulate the client¶s IV to deliver how many drops per minute?

a. 1V c. 35
b. 21 d. 40

99. Mickey, a 6-year-old child with a congenital heart disorder is admitted with congestive heart failure. Digoxin
(lanoxin) 0.12 mg is ordered for the child. The bottle of Lanoxin contains .05 mg of Lanoxin in 1 ml of solution.
What amount should the nurse administer to the child?

a. 1.2 ml c. 3.5 ml
b. 2.4 ml d. 4.2 ml

100. Nurse Alexandra teaches a client about elastic stockings. Which of the following statements, if made by the
client, indicates to the nurse that the teaching was successful?

a. ³I will wear the stockings until the physician tells me to remove them.´
b. ³I should wear the stockings even when I am sleep.´
c. ³Every four hours I should remove the stockings for a half hour.´
d. ³I should put on the stockings before getting out of bed in the morning.´
NURSING PRACTICE V
   c   c      

1. Mr. Marquez reports of losing his job, not being able to sleep at night, and feeling upset with his wife. Nurse John
responds to the client, ³You may want to talk about your employment situation in group today.´ The Nurse is using
which therapeutic technique?

a. Observations c. Exploring
b. Restating d. Focusing

2. Tony refuses his evening dose of Haloperidol (Haldol), then becomes extremely agitated in the dayroom while
other clients are watching television. He begins cursing and throwing furniture. Nurse Oliver first action is to:

a. Check the client¶s medical record for an order for an as-needed I.M. dose of medication for agitation.
b. Place the client in full leather restraints.
c. Call the attending physician and report the behavior.
d. Remove all other clients from the dayroom.

3. Tina who is manic, but not yet on medication, comes to the drug treatment center. The nurse would not let this
client join the group session because:

a. The client is disruptive. c. The client is harmful to others.


b. The client is harmful to self. d. The client needs to be on medication first.

4. Dervid, an adolescent boy was admitted for substance abuse and hallucinations. The client¶s mother asks Nurse
Armando to talk with his husband when he arrives at the hospital. The mother says that she is afraid of what the
father might say to the boy. The most appropriate nursing intervention would be to:

a. Inform the mother that she and the father can work through this problem themselves.
b. Refer the mother to the hospital social worker.
c. Agree to talk with the mother and the father together.
d. Suggest that the father and son work things out.

5. What is Nurse John likely to note in a male client being admitted for alcohol withdrawal?

a. Perceptual disorders. c. Recent alcohol intake.


b. Impending coma. d. Depression with mutism.

6. Aira has taken amitriptyline HCL (Elavil) for 3 days, but now complains that it ³doesn¶t help´ and refuses to take
it. What should the nurse say or do?

a. Withhold the drug.


b. Record the client¶s response.
c. Encourage the client to tell the doctor.
d. Suggest that it takes awhile before seeing the results.

7. Dervid, an adolescent has a history of truancy from school, running away from home and ³barrowing´ other
people¶s things without their permission. The adolescent denies stealing, rationalizing instead that as long as no one
was using the items, it was all right to borrow them. It is important for the nurse to understand the
psychodynamically, this behavior may be largely attributed to a developmental defect related to the:

a. Id
b. Ego
c. Superego
d. Oedipal complex
V. In preparing a female client for electroconvulsive therapy (ECT), Nurse Michelle knows that succinylcoline
(Anectine) will be administered for which therapeutic effect?

a. Short-acting anesthesia c. Skeletal muscle paralysis.


b. Decreased oral and respiratory secretions. d. Analgesia.

9. Nurse Gina is aware that the dietary implications for a client in manic phase of bipolar disorder is:

a. Serve the client a bowl of soup, buttered French bread, and apple slices.
b. Increase calories, decrease fat, and decrease protein.
c. Give the client pieces of cut-up steak, carrots, and an apple.
d. Increase calories, carbohydrates, and protein.

10.What parental behavior toward a child during an admission procedure should cause Nurse Ron to suspect child
abuse?

a. Flat affect c. Acting overly solicitous toward the child.


b. Expressing guilt d. Ignoring the child.

11.Nurse Lynnette notices that a female client with obsessive-compulsive disorder washes her hands for long
periods each day. How should the nurse respond to this compulsive behavior?

a. By designating times during which the client can focus on the behavior.
b. By urging the client to reduce the frequency of the behavior as rapidly as possible.
c. By calling attention to or attempting to prevent the behavior.
d. By discouraging the client from verbalizing anxieties.

12.After seeking help at an outpatient mental health clinic, Ruby who was raped while walking her dog is diagnosed
with posttraumatic stress disorder (PTSD). Three months later, Ruby returns to the clinic, complaining of fear, loss
of control, and helpless feelings. Which nursing intervention is most appropriate for Ruby?

a. Recommending a high-protein, low-fat diet.


b. Giving sleep medication, as prescribed, to restore a normal sleepwake cycle.
c. Allowing the client time to heal.
d. Exploring the meaning of the traumatic event with the client.

13.Meryl, age 19, is highly dependent on her parents and fears leaving home to go away to college. Shortly before
the semester starts, she complains that her legs are paralyzed and is rushed to the emergency department. When
physical examination rules out a physical cause for her paralysis, the physician admits her to the psychiatric unit
where she is diagnosed with conversion disorder. Meryl asks the nurse, "Why has this happened to me?" What is the
nurse's best response?

a. "You've developed this paralysis so you can stay with your parents. You must deal with this conflict if you want to walk
again."
b. "It must be awful not to be able to move your legs. You may feel better if you realize the problem is psychological, not
physical."
c. "Your problem is real but there is no physical basis for it. We'll work on what is going on in your life to find out why it's
happened."
d. "It isn't uncommon for someone with your personality to develop a conversion disorder during times of stress."

14.Nurse Krina knows that the following drugs have been known to be effective in treating obsessive-compulsive
disorder (OCD):

a. benztropine (Cogentin) and diphenhydramine (Benadryl).


b. chlordiazepoxide (Librium) and diazepam (Valium)
c. fluvoxamine (Luvox) and clomipramine (Anafranil)
d. divalproex (Depakote) and lithium (Lithobid)
15.Alfred was newly diagnosed with anxiety disorder. The physician prescribed buspirone (BuSpar). The nurse is
aware that the teaching instructions for newly prescribed buspirone should include which of the following?

a. A warning about the drugs delayed therapeutic effect, which is from 14 to 30 days.
b. A warning about the incidence of neuroleptic malignant syndrome (NMS).
c. A reminder of the need to schedule blood work in 1 week to check blood levels of the drug.
d. A warning that immediate sedation can occur with a resultant drop in pulse.

16.Richard with agoraphobia has been symptom-free for 4 months. Classic signs and symptoms of phobias include:

a. Insomnia and an inability to concentrate.


b. Severe anxiety and fear.
c. Depression and weight loss.
d. Withdrawal and failure to distinguish reality from fantasy.

17.Which medications have been found to help reduce or eliminate panic attacks?

a. Antidepressants c. Antipsychotics
b. Anticholinergics d. Mood stabilizers

1V.A client seeks care because she feels depressed and has gained weight. To treat her atypical depression, the
physician prescribes tranylcypromine sulfate (Parnate), 10 mg by mouth twice per day. When this drug is used to
treat atypical depression, what is its onset of action?

a. 1 to 2 days c. 6 to V days
b. 3 to 5 days d. 10 to 14 days

19. A 65 years old client is in the first stage of Alzheimer's disease. Nurse Patricia should plan to focus this client's
care on:

a. Offering nourishing finger foods to help maintain the client's nutritional status.
b. Providing emotional support and individual counseling.
c. Monitoring the client to prevent minor illnesses from turning into major problems.
d. Suggesting new activities for the client and family to do together.

20.The nurse is assessing a client who has just been admitted to the emergency department. Which signs would
suggest an overdose of an antianxiety agent?

a. Combativeness, sweating, and confusion


b. Agitation, hyperactivity, and grandiose ideation
c. Emotional lability, euphoria, and impaired memory
d. Suspiciousness, dilated pupils, and increased blood pressure

21.The nurse is caring for a client diagnosed with antisocial personality disorder. The client has a history of fighting,
cruelty to animals, and stealing. Which of the following traits would the nurse be most likely to uncover during
assessment?

a. History of gainful employment


b. Frequent expression of guilt regarding antisocial behavior
c. Demonstrated ability to maintain close, stable relationships
d. A low tolerance for frustration
22.Nurse Amy is providing care for a male client undergoing opiate withdrawal. Opiate withdrawal causes severe
physical discomfort and can be life-threatening. To minimize these effects, opiate users are commonly detoxified
with:

a. Barbiturates c. Methadone
b. Amphetamines d. Benzodiazepines

23.Nurse Cristina is caring for a client who experiences false sensory perceptions with no basis in reality. These
perceptions are known as:

a. Delusions c. Loose associations


b. Hallucinations d. Neologisms

24. Nurse Marco is developing a plan of care for a client with anorexia nervosa. Which action should the nurse
include in the plan?

a. Restricts visits with the family and friends until the client begins to eat.
b. Provide privacy during meals.
c. Set up a strict eating plan for the client.
d. Encourage the client to exercise, which will reduce her anxiety.

25.Tim is admitted with a diagnosis of delusions of grandeur. The nurse is aware that this diagnosis reflects a belief
that one is:

a. Highly important or famous. c. Connected to events unrelated to oneself


b. Being persecuted d. Responsible for the evil in the world.

26.Nurse Jen is caring for a male client with manic depression. The plan of care for a client in a manic state would
include:

a. Offering a high-calorie meals and strongly encouraging the client to finish all food.
b. Insisting that the client remain active through the day so that he¶ll sleep at night.
c. Allowing the client to exhibit hyperactive, demanding, manipulative behavior without setting limits.
d. Listening attentively with a neutral attitude and avoiding power struggles.

27.Ramon is admitted for detoxification after a cocaine overdose. The client tells the nurse that he frequently uses
cocaine but that he can control his use if he chooses. Which coping mechanism is he using?

a. Withdrawal c. Repression
b. Logical thinking d. Denial

2V.Richard is admitted with a diagnosis of schizotypal personality disorder. Which signs would this client exhibit
during social situations?

a. Aggressive behavior c. Emotional affect


b. Paranoid thoughts d. Independence needs

29. Nurse Mickey is caring for a client diagnosed with bulimia. The most appropriate initial goal for a client
diagnosed with bulimia is to:

a. Avoid shopping for large amounts of food.


b. Control eating impulses.
c. Identify anxiety-causing situations
d. Eat only three meals per day.
30.Rudolf is admitted for an overdose of amphetamines. When assessing the client, the nurse should expect to see:

a. Tension and irritability c. Hypotension


b. Slow pulse d. Constipation

31.Nicolas is experiencing hallucinations tells the nurse, ³The voices are telling me I¶m no good.´ The client asks if
the nurse hears the voices. The most appropriate response by the nurse would be:

a. ³It is the voice of your conscience, which only you can control.´
b. ³No, I do not hear your voices, but I believe you can hear them´.
c. ³The voices are coming from within you and only you can hear them.´
d. ³Oh, the voices are a symptom of your illness; don¶t pay any attention to them.´

32.The nurse is aware that the side effect of electroconvulsive therapy that a client may experience:

a. Loss of appetite c. Confusion for a time after treatment


b. Postural hypotension d. Complete loss of memory for a time

33.A dying male client gradually moves toward resolution of feelings regarding impending death. Basing care on the
theory of Kubler-Ross, Nurse Trish plans to use nonverbal interventions when assessment reveals that the client is in
the:

a. Anger stage c. Bargaining stage


b. Denial stage d. Acceptance stage

34.The outcome that is unrelated to a crisis state is:

a. Learning more constructive coping skills


b. Decompensation to a lower level of functioning.
c. Adaptation and a return to a prior level of functioning.
d. A higher level of anxiety continuing for more than 3 months.

35.Miranda a psychiatric client is to be discharged with orders for haloperidol (haldol) therapy. When developing a
teaching plan for discharge, the nurse should include cautioning the client against:

a. Driving at night c. Ingesting wines and cheeses


b. Staying in the sun d. Taking medications containing aspirin

36.Jen a nursing student is anxious about the upcoming board examination but is able to study intently and does not
become distracted by a roommate¶s talking and loud music. The student¶s ability to ignore distractions and to focus
on studying demonstrates:

a. Mild-level anxiety c. Severe-level anxiety


b. Panic-level anxiety d. Moderate-level anxiety

37.When assessing a premorbid personality characteristics of a client with a major depression, it would be unusual
for the nurse to find that this client demonstrated:

a. Rigidity c. Diverse interest


b. Stubbornness d. Over meticulousness

3V.Nurse Krina recognizes that the suicidal risk for depressed client is greatest:
a. As their depression begins to improve c. Before nay type of treatment is started
b. When their depression is most severe d. As they lose interest in the environment
39.Nurse Kate would expect that a client with vascular dementis would experience:

a. Loss of remote memory related to anoxia


b. Loss of abstract thinking related to emotional state
c. Inability to concentrate related to decreased stimuli
d. Disturbance in recalling recent events related to cerebral hypoxia.

40.Josefina is to be discharged on a regimen of lithium carbonate. In the teaching plan for discharge the nurse should
include:

a. Advising the client to watch the diet carefully


b. Suggesting that the client take the pills with milk
c. Reminding the client that a CBC must be done once a month.
d. Encouraging the client to have blood levels checked as ordered.

41.The psychiatrist orders lithium carbonate 600 mg p.o t.i.d for a female client. Nurse Katrina would be aware that
the teaching about the side effects of this drug were understood when the client state, ³I will call my doctor
immediately if I notice any:

a. Sensitivity to bright light or sun c. Sexual dysfunction or breast enlargement


b. Fine hand tremors or slurred speech d. Inability to urinate or difficulty when urinating

42.Nurse Mylene recognizes that the most important factor necessary for the establishment of trust in a critical care
area is:

a. Privacy c. Empathy
b. Respect d. Presence

43.When establishing an initial nurse-client relationship, Nurse Hazel should explore with the client the:

a. Client¶s perception of the presenting problem.


b. Occurrence of fantasies the client may experience.
c. Details of any ritualistic acts carried out by the client
d. Client¶s feelings when external; controls are instituted.

44.Tranylcypromine sulfate (Parnate) is prescribed for a depressed client who has not responded to the tricyclic
antidepressants. After teaching the client about the medication, Nurse Marian evaluates that learning has occurred
when the client states, ³I will avoid:

a. Citrus fruit, tuna, and yellow vegetables.´ c. Green leafy vegetables, chicken, and milk.´
b. Chocolate milk, aged cheese, and yogurt¶´ d. Whole grains, red meats, and carbonated soda.´

45.Nurse John is a aware that most crisis situations should resolve in about:

a. 1 to 2 weeks c. 4 to 6 months
b. 4 to 6 weeks d. 6 to 12 months

46. Nurse Judy knows that statistics show that in adolescent suicide behavior:

a. Females use more dramatic methods than males


b. Males account for more attempts than do females
c. Females talk more about suicide before attempting it
d. Males are more likely to use lethal methods than are females
47. Dervid with paranoid schizophrenia repeatedly uses profanity during an activity therapy session. Which response
by the nurse would be most appropriate?

a. "Your behavior won't be tolerated. Go to your room immediately."


b. "You're just doing this to get back at me for making you come to therapy."
c. "Your cursing is interrupting the activity. Take time out in your room for 10 minutes."
d. "I'm disappointed in you. You can't control yourself even for a few minutes."

4V.Nurse Maureen knows that the nonantipsychotic medication used to treat some clients with schizoaffective
disorder is:

a. phenelzine (Nardil) c. lithium carbonate (Lithane)


b. chlordiazepoxide (Librium) d. imipramine (Tofranil)

49.Which information is most important for the nurse Trinity to include in a teaching plan for a male schizophrenic
client taking clozapine (Clozaril)?

a. Monthly blood tests will be necessary.


b. Report a sore throat or fever to the physician immediately.
c. Blood pressure must be monitored for hypertension.
d. Stop the medication when symptoms subside.

50.Ricky with chronic schizophrenia takes neuroleptic medication is admitted to the psychiatric unit. Nursing
assessment reveals rigidity, fever, hypertension, and diaphoresis. These findings suggest which lifethreatening
reaction:

a. Tardive dyskinesia. c. Neuroleptic malignant syndrome.


b. Dystonia. d. Akathisia.

51.Which nursing intervention would be most appropriate if a male client develop orthostatic hypotension while
taking amitriptyline (Elavil)?

a. Consulting with the physician about substituting a different type of antidepressant.


b. Advising the client to sit up for 1 minute before getting out of bed.
c. Instructing the client to double the dosage until the problem resolves.
d. Informing the client that this adverse reaction should disappear within 1 week.

52.Mr. Cruz visits the physician's office to seek treatment for depression, feelings of hopelessness, poor appetite,
insomnia, fatigue, low selfesteem, poor concentration, and difficulty making decisions. The client states that these
symptoms began at least 2 years ago. Based on this report, the nurse Tyfany suspects:

a. Cyclothymic disorder. c. Major depression.


b. Atypical affective disorder. d. Dysthymic disorder.

53. After taking an overdose of phenobarbital (Barbita), Mario is admitted to the emergency department. Dr.
Trinidad prescribes activated charcoal (Charcocaps) to be administered by mouth immediately. Before administering
the dose, the nurse verifies the dosage ordered. What is the usual minimum dose of activated charcoal?

a. 5 g mixed in 250 ml of water c. 30 g mixed in 250 ml of water


b. 15 g mixed in 500 ml of water d. 60 g mixed in 500 ml of water

54.What herbal medication for depression, widely used in Europe, is now being prescribed in the United States?

a. Ginkgo biloba c. St. John's wort


b. Echinacea d. Ephedra
55.Cely with manic episodes is taking lithium. Which electrolyte level should the nurse check before administering
this medication?

a. Calcium c. Chloride
b. Sodium d. Potassium

56.Nurse Josefina is caring for a client who has been diagnosed with delirium. Which statement about delirium is
true?

a. It's characterized by an acute onset and lasts about 1 month.


b. It's characterized by a slowly evolving onset and lasts about 1 week.
c. It's characterized by a slowly evolving onset and lasts about 1 month.
d. It's characterized by an acute onset and lasts hours to a number of days.

57.Edward, a 66 year old client with slight memory impairment and poor concentration is diagnosed with primary
degenerative dementia of the Alzheimer's type. Early signs of this dementia include subtle personality changes and
withdrawal from social interactions. To assess for progression to the middle stage of Alzheimer's disease, the nurse
should observe the client for:

a. Occasional irritable outbursts. c. Lack of spontaneity.


b. Impaired communication. d. Inability to perform self-care activities.

5V.Isabel with a diagnosis of depression is started on imipramine (Tofranil), 75 mg by mouth at bedtime. The nurse
should tell the client that:

a. This medication may be habit forming and will be discontinued as soon as the client feels better.
b. This medication has no serious adverse effects.
c. The client should avoid eating such foods as aged cheeses, yogurt, and chicken livers while taking the medication.
d. This medication may initially cause tiredness, which should become less bothersome over time.

59.Kathleen is admitted to the psychiatric clinic for treatment of anorexia nervosa. To promote the client's physical
health, the nurse should plan to:

a. Severely restrict the client's physical activities.


b. Weigh the client daily, after the evening meal.
c. Monitor vital signs, serum electrolyte levels, and acid-base balance.
d. Instruct the client to keep an accurate record of food and fluid intake.

60.Celia with a history of polysubstance abuse is admitted to the facility. She complains of nausea and vomiting 24
hours after admission. The nurse assesses the client and notes piloerection, pupillary dilation, and lacrimation. The
nurse suspects that the client is going through which of the following withdrawals?

a. Alcohol withdrawal c. Cocaine withdrawal


b. Cannibis withdrawal d. Opioid withdrawal

61.Mr. Garcia, an attorney who throws books and furniture around the office after losing a case is referred to the
psychiatric nurse in the law firm's employee assistance program. Nurse Beatriz knows that the client's behavior most
likely represents the use of which defense mechanism?

a. Regression c. Reaction-formation
b. Projection d. Intellectualization
62.Nurse Anne is caring for a client who has been treated long term with antipsychotic medication. During the
assessment, Nurse Anne checks the client for tardive dyskinesia. If tardive dyskinesia is present, Nurse Anne would
most likely observe:

a. Abnormal movements and involuntary movements of the mouth, tongue, and face.
b. Abnormal breathing through the nostrils accompanied by a ³thrill.´
c. Severe headache, flushing, tremors, and ataxia.
d. Severe hypertension, migraine headache,

63.Dennis has a lithium level of 2.4 mEq/L. The nurse immediately would assess the client for which of the
following signs or symptoms?

a. Weakness c. Blurred vision


b. Diarrhea d. Fecal incontinence

64.Nurse Jannah is monitoring a male client who has been placed inrestraints because of violent behavior. Nurse
determines that it will be safe to remove the restraints when:

a. The client verbalizes the reasons for the violent behavior.


b. The client apologizes and tells the nurse that it will never happen again.
c. No acts of aggression have been observed within 1 hour after the release of two of the extremity restraints.
d. The administered medication has taken effect.

65.Nurse Irish is aware that Ritalin is the drug of choice for a child with ADHD. The side effects of the following
may be noted by the nurse:

a. Increased attention span and concentration c. Sleepiness and lethargy


b. Increase in appetite d. Bradycardia and diarrhea

66.Kitty, a 9 year old child has very limited vocabulary and interaction skills. She has an I.Q. of 45. She is
diagnosed to have Mental retardation of this classification:

a. Profound c. Moderate
b. Mild d. Severe

67.The therapeutic approach in the care of Armand an autistic child include the following EXCEPT:

a. Engage in diversionary activities when acting -out


b. Provide an atmosphere of acceptance
c. Provide safety measures
d. Rearrange the environment to activate the child

6V.Jeremy is brought to the emergency room by friends who state that he took something an hour ago. He is actively
hallucinating, agitated, with irritated nasal septum.

a. Heroin c. LSD
b. Cocaine d. Marijuana

69.Nurse Pauline is aware that Dementia unlike delirium is characterized by:

a. Slurred speech c. Clouding of consciousness


b. Insidious onset d. Sensory perceptual change
70.A 35 year old female has intense fear of riding an elevator. She claims ³ As if I will die inside.´ The client is
suffering from:

a. Agoraphobia c. Claustrophobia
b. Social phobia d. Xenophobia

71.Nurse Myrna develops a counter-transference reaction. This is evidenced by:

a. Revealing personal information to the client


b. Focusing on the feelings of the client.
c. Confronting the client about discrepancies in verbal or non-verbal behavior
d. The client feels angry towards the nurse who resembles his mother.

72.Tristan is on Lithium has suffered from diarrhea and vomiting. What should the nurse in-charge do first:

a. Recognize this as a drug interaction


b. Give the client Cogentin
c. Reassure the client that these are common side effects of lithium therapy
d. Hold the next dose and obtain an order for a stat serum lithium level

73.Nurse Sarah ensures a therapeutic environment for all the client. Which of the following best describes a
therapeutic milieu?

a. A therapy that rewards adaptive behavior c. A living, learning or working environment.


b. A cognitive approach to change behavior d. A permissive and congenial environment

74.Anthony is very hostile toward one of the staff for no apparent reason. He is manifesting:

a. Splitting c. Countertransference
b. Transference d. Resistance

75.Marielle, 17 years old was sexually attacked while on her way home from school. She is brought to the hospital
by her mother. Rape is an example of which type of crisis:

a. Situational c. Developmental
b. Adventitious d. Internal

76. Nurse Greta is aware that the following is classified as an Axis I disorder by the '   
 
  '  Text Revision (DSM-IV-TR) is:

a. Obesity c. Major depression


b. Borderline personality disorder d. Hypertension

77.Katrina, a newly admitted is extremely hostile toward a staff member she has just met, without apparent reason.
According to Freudian theory, the nurse should suspect that the client is experiencing which of the following
phenomena?

a. Intellectualization c. Triangulation
b. Transference d. Splitting

7V.An V3year-old male client is in extended care facility is anxious most of the time and frequently complains of a
number of vague symptoms that interfere with his ability to eat. These symptoms indicate which of the following
disorders?

a. Conversion disorder c. Severe anxiety


b. Hypochondriasis d. Sublimation
79. Charina, a college student who frequently visited the health center during the past year with multiple vague
complaints of GI symptoms before course examinations. Although physical causes have been eliminated, the student
continues to express her belief that she has a serious illness. These symptoms are typically of which of the following
disorders?

a. Conversion disorder c. Hypochondriasis


b. Depersonalization d. Somatization disorder

V0. Nurse Daisy is aware that the following pharmacologic agents are sedativehypnotic medication is used to induce
sleep for a client experiencing a sleep disorder is:

a. Triazolam (Halcion) c. Fluoxetine (Prozac)


b. Paroxetine (Paxil)\ d. Risperidone (Risperdal)

V1. Aldo, with a somatoform pain disorder may obtain secondary gain. Which of the following statement refers to a
secondary gain?

a. It brings some stability to the family


b. It decreases the preoccupation with the physical illness
c. It enables the client to avoid some unpleasant activity
d. It promotes emotional support or attention for the client

V2. Dervid is diagnosed with panic disorder with agoraphobia is talking with the nurse in-charge about the progress
made in treatment. Which of the following statements indicates a positive client response?

a. ³I went to the mall with my friends last Saturday´


b. ³I¶m hyperventilating only when I have a panic attack´
c. ³Today I decided that I can stop taking my medication´
d. ³Last night I decided to eat more than a bowl of cereal´

V3. The effectiveness of monoamine oxidase (MAO) inhibitor drug therapy in a client with posttraumatic stress
disorder can be demonstrated by which of the following client self ±reports?

a. ³I¶m sleeping better and don¶t have nightmares´


b. ³I¶m not losing my temper as much´
c. ³I¶ve lost my craving for alcohol´
d. I¶ve lost my phobia for water´

V4. Mark, with a diagnosis of generalized anxiety disorder wants to stop taking his lorazepam (Ativan). Which of the
following important facts should nurse Betty discuss with the client about discontinuing the medication?

a. Stopping the drug may cause depression


b. Stopping the drug increases cognitive abilities
c. Stopping the drug decreases sleeping difficulties
d. Stopping the drug can cause withdrawal symptoms

V5. Jennifer, an adolescent who is depressed and reported by his parents as having difficulty in school is brought to
the community mental health center to be evaluated. Which of the following other health problems would the nurse
suspect?

a. Anxiety disorder c. Cognitive impairment


b. Behavioral difficulties d. Labile moods
V6. Ricardo, an outpatient in psychiatric facility is diagnosed with dysthymic disorder. Which of the following
statement about dysthymic disorder is true?

a. It involves a mood range from moderate depression to hypomania


b. It involves a single manic depression
c. It¶s a form of depression that occurs in the fall and winter
d. It¶s a mood disorder similar to major depression but of mild to moderate severity

V7. The nurse is aware that the following ways in vascular dementia different from Alzheimer¶s disease is:

a. Vascular dementia has more abrupt onset


b. The duration of vascular dementia is usually brief
c. Personality change is common in vascular dementia
d. The inability to perform motor activities occurs in vascular dementia

VV. Loretta, a newly admitted client was diagnosed with delirium and has histo ry of hypertension and anxiety. She
had been taking digoxin, furosemide (Lasix),and diazepam (Valium) for anxiety. This client¶s impairment may be
related to which of the following conditions?

a. Infection c. Drug intoxication


b. Metabolic acidosis d. Hepatic encephalopathy

V9. Nurse Ron enters a client¶s room, the client says, ³They¶re crawling on my sheets! Get them off my bed!´ Which
of the following assessment is the most accurate?

a. The client is experiencing aphasia c. The client is experiencing a flight of ideas


b. The client is experiencing dysarthria d. The client is experiencing visual hallucination

90. Which of the following descriptions of a client¶s experience and behavior can be assessed as an illusion?

a. The client tries to hit the nurse when vital signs must be taken
b. The client says, ³I keep hearing a voice telling me to run away´
c. The client becomes anxious whenever the nurse leaves the bedside
d. The client looks at the shadow on a wall and tells the nurse she sees frightening faces on the wall.

91. During conversation of Nurse John with a client, he observes that the client shift from one topic to the next on a
regular basis. Which of the following terms describes this disorder?

a. Flight of ideas c. Ideas of reference


b. Concrete thinking d. Loose association

92. Francis tells the nurse that her coworkers are sabotaging the computer. When the nurse asks questions, the client
becomes argumentative. This behavior shows personality traits associated with which of the following personality
disorder?

a. Antisocial c. Paranoid
b. Histrionic d. Schizotypal

93. Which of the following interventions is important for a Cely experiencing with paranoid personality disorder
taking olanzapine (Zyprexa)?

a. Explain effects of serotonin syndrome


b. Teach the client to watch for extrapyramidal adverse reaction
c. Explain that the drug is less affective if the client smokes
d. Discuss the need to report paradoxical effects such as euphoria
94. Nurse Alexandra notices other clients on the unit avoiding a client diagnosed with antisocial personality
disorder. When discussing appropriate behavior in group therapy, which of the following comments is expected
about this client by his peers?

a. Lack of honesty c. Show of temper tantrums


b. Belief in superstition d. Constant need for attention

95. Tommy, with dependent personality disorder is working to increase his self esteem. Which of the following
statements by the Tommy shows teaching was successful?

a. ³I¶m not going to look just at the negative things about myself´
b. ³I¶m most concerned about my level of competence and progress´
c. ³I¶m not as envious of the things other people have as I used to be´
d. ³I find I can¶t stop myself from taking over things other should be doing´

96. Norma, a 42-year-old client with a diagnosis of chronic undifferentiated schizophrenia lives in a rooming house
that has a weekly nursing clinic. She scratches while she tells the nurse she feels creatures eating away at her skin.
Which of the following interventions should be done first?

a. Talk about his hallucinations and fears


b. Refer him for anticholinergic adverse reactions
c. Assess for possible physical problems such as rash
d. Call his physician to get his medication increased to control his psychosis

97. Ivy, who is on the psychiatric unit is copying and imitating the movements of her primary nurse. During
recovery, she says, ³I thought the nurse was my mirror. I felt connected only when I saw my nurse.´ This behavior is
known by which of the following terms?

a. Modeling c. Ego-syntonicity
b. Echopraxia d. Ritualism

9V. Jun approaches the nurse and tells that he hears a voice telling him that he¶s evil and deserves to die. Which of
the following terms describes the client¶s perception?

a. Delusion c. Hallucination
b. Disorganized speech d. Idea of reference

99. Mike is admitted to a psychiatric unit with a diagnosis of undifferentiated schizophrenia. Which of the following
defense mechanisms is probably used by mike?

a. Projection c. Regression
b. Rationalization d. Repression

100. Rocky has started taking haloperidol (Haldol). Which of the following instructions is most appropriate for
Ricky before taking haloperidol?

a. Should report feelings of restlessness or agitation at once


b. Use a sunscreen outdoors on a year-round basis
c. Be aware you¶ll feel increased energy taking this drug
d. This drug will indirectly control essential hypertension

   (
  c  
 c 
1. (D) The actions of a reasonably prudent nurse with similar education and experience.
  : The standard of care is determined by the average degree of skill, care, and diligence by nurses in similar
circumstances.
2. : (B) I.M
  With a platelet count of 22,000/ȝl, the clients tends to bleed easily. Therefore, the nurse should avoid
using the I.M. route because the area is a highly vascular and can bleed readily when penetrated by a needle. The
bleeding can be difficult to stop.
3. (C) ³Digoxin 0.125 mg P.O. once daily´
  The nurse should always place a zero before a decimal point so that no one misreads the figure, which
could result in a dosage error. The nurse should never insert a zero at the end of a dosage that includes a decimal
point because this could be misread, possibly leading to a tenfold increase in the dosage.
4. (A) Ineffective peripheral tissue perfusion related to venous congestion.
  Ineffective peripheral tissue perfusion related to venous congestion takes the highest priority because
venous inflammation and clot formation impede blood flow in a client with deep vein thrombosis.
5. : (B) A 44 year-old myocardial infarction (MI) client who is complaining of nausea.
  Nausea is a symptom of impending myocardial infarction (MI) and should be assessed immediately so
that treatment can be instituted and further damage to the heart is avoided.
6. (C) Check circulation every 15-30 minutes.
  : Restraints encircle the limbs, which place the client at risk for circulation being restricted to the distal
areas of the extremities. Checking the client¶s circulation every 15-30 minutes will allow the nurse to adjust the
restraints before injury from decreased blood flow occurs.
7. (A) Prevent stress ulcer
  Curling¶s ulcer occurs as a generalized stress response in burn patients. This results in a decreased
production of mucus and increased secretion of gastric acid. The best treatment for this prophylactic use of antacids
and H2 receptor blockers.
V. (D) Continue to monitor and record hourly urine output
  Normal urine output for an adult is approximately 1 ml/minute (60 ml/hour). Therefore, this client's
output is normal. Beyond continued evaluation, no nursing action is warranted.
9. (B) ³My ankle feels warm´.
  Ice application decreases pain and swelling. Continued or increased pain, redness, and increased warmth
are signs of inflammation that shouldn't occur after ice application
10. (B) Hyperkalemia
  A loop diuretic removes water and, along with it, sodium and potassium. This may result in
hypokalemia, hypovolemia, and hyponatremia.
11. :(A) Have condescending trust and confidence in their subordinates
  : Benevolent-authoritative managers pretentiously show their trust and confidence to their followers.
12. (A) Provides continuous, coordinated and comprehensive nursing services.
  Functional nursing is focused on tasks and activities and not on the care of the patients.
13. (B) Standard written order
  This is a standard written order. Prescribers write a single order for medications given only once. A stat
order is written for medications given immediately for an urgent client problem. A standing order, also known as a
protocol, establishes guidelines for treating a particular disease or set of symptoms in special care areas such as the
coronary care unit. Facilities also may institute medication protocols that specifically designate drugs that a nurse
may not give.
14. : (D) Liquid or semi-liquid stools
  : Passage of liquid or semi-liquid stools results from seepage of unformed bowel contents around the
impacted stool in the rectum. Clients with fecal impaction don't pass hard, brown, formed stools because the feces
can't move past the impaction. These clients typically report the urge to defecate (although they can't pass stool) and
a decreased appetite.
15. : (C) Pulling the helix up and back
  To perform an otoscopic examination on an adult, the nurse grasps the helix of the ear and pulls it up and
back to straighten the ear canal. For a child, the nurse grasps the helix and pulls it down to straighten the ear canal.
Pulling the lobule in any direction wouldn't
straighten the ear canal for visualization.
16. (A) Protect the irritated skin from sunlight.
  Irradiated skin is very sensitive and must be protected with clothing or sunblock. The priority approach
is the avoidance of strong sunlight.
17. (C) Assist the client in removing dentures and nail polish.
  Dentures, hairpins, and combs must be removed. Nail polish must be removed so that cyanosis can be
easily monitored by observing the nail beds.
1V. (D) Sudden onset of continuous epigastric and back pain.
  The autodigestion of tissue by the pancreatic enzymes results in pain from inflammation, edema, and
possible hemorrhage. Continuous, unrelieved epigastric or back pain reflects the inflammatory process in the
pancreas.
19. (B) Provide high-protein, high-carbohydrate diet.
  A positive nitrogen balance is important for meeting metabolic needs, tissue repair, and resistance to
infection. Caloric goals may be as high as 5000 calories per day.
20. (A) Blood pressure and pulse rate.
  : The baseline must be established to recognize the signs of an anaphylactic or hemolytic reaction to the
transfusion.
21. (D) Immobilize the leg before moving the client.
  If the nurse suspects a fracture, splinting the area before moving the client is imperative. The nurse
should call for emergency help if the client is not hospitalized and call for a physician for the hospitalized client.
22. (B) Admit the client into a private room.
  The client who has a radiation implant is placed in a private room and has a limited number of visitors.
This reduces the exposure of others to the radiation.
23. (C) Risk for infection
  Agranulocytosis is characterized by a reduced number of leukocytes (leucopenia) and neutrophils
(neutropenia) in the blood. The client is at high risk for infection because of the decreased body defenses against
microorganisms. Deficient knowledge related to the nature of the disorder may be appropriate diagnosis but is not
the priority.
24. (B) Place the client on the left side in the Trendelenburg position.
  Lying on the left side may prevent air from flowing into the pulmonary veins. The Trendelenburg
position increases intrathoracic pressure, which decreases the amount of blood pulled into the vena cava during
aspiration.
25. (A) Autocratic.
  The autocratic style of leadership is a task-oriented and directive.
26. : (D) 2.5 cc
  : 2.5 cc is to be added, because only a 500 cc bag of solution is being medicated instead of a 1 liter.
27. : (A) 50 cc/ hour
  A rate of 50 cc/hr. The child is to receive 400 cc over a period of V hours = 50 cc/hr.
2V. (B) Assess the client for presence of pain.
  Assessing the client for pain is a very important measure. Postoperative pain is an indication of
complication. The nurse should also assess the client for pain to provide for the client¶s comfort.
29. (A) BP ± V0/60, Pulse ± 110 irregular
  : The classic signs of cardiogenic shock are low blood pressure, rapid and weak irregular pulse, cold,
clammy skin, decreased urinary output, and cerebral hypoxia.
30. (A) Take the proper equipment, place the client in a comfortable position, and record the appropriate
information in the client¶s chart.
  It is a general or comprehensive statement about the correct procedure, and it includes the basic ideas
which are found in the other options
31. : (B) Evaluation
  Evaluation includes observing the person, asking questions, and comparing the patient¶s behavioral
responses with the expected outcomes.
32. (C) History of present illness
  The history of present illness is the single most important factor in assisting the health professional in
arriving at a diagnosis or determining the person¶s needs.
33. (A) Trochanter roll extending from the crest of the ileum to the mid-thigh.
  A trochanter roll, properly placed, provides resistance to the external rotation of the hip.
34. : (C) Stage III
  Clinically, a deep crater or without undermining of adjacent tissue is noted.
35. (A) Second intention healing
  When wounds dehisce, they will allowed to heal by secondary intention
36. (D) Tachycardia
  With an extracellular fluid or plasma volume deficit, compensatory mechanisms stimulate the heart,
causing an increase in heart rate.
37. (A) 0.75
  To determine the number of milliliters the client should receive, the nurse uses the fraction method in the
following equation.
75 mg/X ml = 100 mg/1 ml
To solve for X, cross-multiply:
75 mg x 1 ml = X ml x 100 mg
75 = 100X
75/100 = X
0.75 ml (or . ml) = X
3V. (D) It¶s a measure of effect, not a standard measure of weight or quantity.
  An insulin unit is a measure of effect, not a standard measure of weight or quantity. Different drugs
measured in units may have no relationship to one another in quality or quantity.
39. (B) 3V.9 °C
  To convert Fahrenheit degreed to Centigrade, use this formula
°C = (°F ± 32) ÷ 1.V
°C = (102 ± 32) ÷ 1.V
°C = 70 ÷ 1.V
°C = 3V.9
40. (C) Failing eyesight, especially close vision.
  Failing eyesight, especially close vision, is one of the first signs of aging in middle life (ages 46 to 64).
More frequent aches and pains begin in the early late years (ages 65 to 79). Increase in loss of muscle tone occurs in
later years (age V0 and older).
41. (A) Checking and taping all connections
  Air leaks commonly occur if the system isn¶t secure. Checking all connections and taping them will
prevent air leaks. The chest drainage system is kept lower to promote drainage not to prevent leaks.
42. (A) Check the client¶s identification band.
  Checking the client¶s identification band is the safest way to verify a client¶s identity because the band is
assigned on admission and isn¶t be removed at any time. (If it is removed, it must be replaced). Asking the client¶s
name or having the client repeated his name would be appropriate only for a client who¶s alert, oriented, and able to
understand what is being said, but isn¶t the safe standard of practice. Names on bed aren¶t always reliable
43. (B) 32 drops/minute
  Giving 1,000 ml over V hours is the same as giving 125 ml over 1 hour (60 minutes). Find the number of
milliliters per minute as follows:
125/60 minutes = X/1 minute
60X = 125 = 2.1 ml/minute
To find the number of drops per minute:
2.1 ml/X gtt = 1 ml/ 15 gtt
X = 32 gtt/minute, or 32 drops/minute
44. : (A) Clamp the catheter
  : If a central venous catheter becomes disconnected, the nurse should immediately apply a catheter clamp,
if available. If a clamp isn¶t available, the nurse can place a sterile syringe or catheter plug in the catheter hub. After
cleaning the hub with alcohol or povidone-iodine solution, the nurse must replace the I.V. extension and restart the
infusion.
45. (D) Auscultation, percussion, and palpation.
  The correct order of assessment for examining the abdomen is inspection, auscultation, percussion, and
palpation. The reason for this approach is that the less intrusive techniques should be performed before the more
intrusive techniques. Percussion and palpation can alter natural findings during auscultation.
46. (D) Ulnar surface of the hand
  The nurse uses the ulnar surface, or ball, of the hand to asses tactile fremitus, thrills, and vocal vibrations
through the chest wall. The fingertips and finger pads best distinguish texture and shape. The dorsal surface best
feels warmth.
47. : (C) Formative
  Formative (or concurrent) evaluation occurs continuously throughout the teaching and learning process.
One benefit is that the nurse can adjust teaching strategies as necessary to enhance learning. Summative, or
retrospective, evaluation occurs at the conclusion of the teaching and learning session. Informative is not a type of
evaluation.
4V. (B) Once per year
  Yearly mammograms should begin at age 40 and continue for as long as the woman is in good health. If
health risks, such as family history, genetic tendency, or past breast cancer, exist, more frequent examinations may
be necessary.
49. (A) Respiratory acidosis
  The client has a below-normal (acidic) blood pH value and an above-normal partial pressure of arterial
carbon dioxide (Paco2) value, indicating respiratory acidosis. In respiratory alkalosis, the pH value is above normal
and in the Paco2 value is below normal. In metabolic acidosis, the pH and bicarbonate (Hco3) values are below
normal. In metabolic alkalosis, the pH and Hco3 values are above normal.
50. (B) To provide support for the client and family in coping with terminal illness.
  Hospices provide supportive care for terminally ill clients and their families. Hospice care doesn¶t focus
on counseling regarding health care costs. Most client referred to hospices have been treated for their disease
without success and will receive only palliative care in the hospice.
51. (C) Using normal saline solution to clean the ulcer and applying a protective dressing as necessary.
  Washing the area with normal saline solution and applying a protective dressing are within the nurse¶s
realm of interventions and will protect the area. Using a povidone-iodine wash and an antibiotic cream require a
physician¶s order. Massaging with an astringent can further damage the skin.
52. (D) Foot
  An elastic bandage should be applied form the distal area to the proximal area. This method promotes
venous return. In this case, the nurse should begin applying the bandage at the client¶s foot. Beginning at the ankle,
lower thigh, or knee does not promote venous return.
53. (B) Hypokalemia
  Insulin administration causes glucose and potassium to move into the cells, causing hypokalemia.
54. (A) Throbbing headache or dizziness
  : Headache and dizziness often occur when nitroglycerin is taken at the beginning of therapy. However,
the client usually develops tolerance
55. (D) Check the client¶s level of consciousness
  Determining unresponsiveness is the first step assessment action to take. When a client is in ventricular
tachycardia, there is a significant decrease in cardiac output. However, checking the unresponsiveness ensures
whether the client is affected by the decreased cardiac output.
56. (B) On the affected side of the client.
  When walking with clients, the nurse should stand on the affected side and grasp the security belt in the
midspine area of the small of the back. The nurse should position the free hand at the shoulder area so that the client
can be pulled toward the nurse in the event that there is a forward fall. The client is instructed to look up and
outward rather than at his or her feet.
57. : (A) Urine output: 45 ml/hr
  : Adequate perfusion must be maintained to all vital organs in order for the client to remain visible as an
organ donor. A urine output of 45 ml per hour indicates adequate renal perfusion. Low blood pressure and delayed
capillary refill time are circulatory system indicators of inadequate perfusion. A serum pH of 7.32 is acidotic, which
adversely affects all body tissues.
5V. (D ) Obtaining the specimen from the urinary drainage bag.
  : A urine specimen is not taken from the urinary drainage bag. Urine undergoes chemical changes while
sitting in the bag and does not necessarily reflect the current client status. In addition, it may become contaminated
with bacteria from opening the system.
59. (B) Cover the client, place the call light within reach, and answer the phone call.
  Because telephone call is an emergency, the nurse may need to answer it. The other appropriate action is
to ask another nurse to accept the call. However, is not one of the options. To maintain privacy and safety, the nurse
covers the client and places the call light within the client¶s reach. Additionally, the client¶s door should be closed or
the room curtains pulled around the bathing area.
60. (C) Use a sterile plastic container for obtaining the specimen.
  : Sputum specimens for culture and sensitivity testing need to be obtained using sterile techniques because
the test is done to determine the presence of organisms. If the procedure for obtaining the specimen is not sterile,
then the specimen is not sterile, then the specimen would be contaminated and the results of the test would be
invalid.
61. (A) Puts all the four points of the walker flat on the floor, puts weight on the hand pieces, and then
walks into it.
  : When the client uses a walker, the nurse stands adjacent to the affected side. The client is instructed to
put all four points of the walker 2 feet forward flat on the floor before putting weight on hand pieces. This will
ensure client safety and prevent stress cracks in the walker. The client is then instructed to move the walker forward
and walk into it.
62. (C) Draws one line to cross out the incorrect information and then initials the change.
  To correct an error documented in a medical record, the nurse draws one line through the incorrect
information and then initials the error. An error is never erased and correction fluid is never used in the medical
record.
63. (C) Secures the client safety belts after transferring to the stretcher.
  During the transfer of the client after the surgical procedure is complete, the nurse should avoid exposure
of the client because of the risk for potential heat loss. Hurried movements and rapid changes in the position should
be avoided because these predispose the client to hypotension. At the time of the transfer from the surgery table to
the stretcher, the client is still affected by the effects of the anesthesia; therefore, the client should not move self.
Safety belts can prevent the client from falling off the stretcher.
64. (B) Gown and gloves
  Contact precautions require the use of gloves and a gown if direct client contact is anticipated. Goggles
are not necessary unless the nurse anticipates the splashes of blood, body fluids, secretions, or excretions may occur.
Shoe protectors are not necessary.
65. : (C) Quad cane
  Crutches and a walker can be difficult to maneuver for a client with weakness on one side. A cane is
better suited for client with weakness of the arm and leg on one side. However, the quad cane would provide the
most stability because of the structure of the cane and because a quad cane has four legs.
66. (D) Left side-lying with the head of the bed elevated 45 degrees.
  : To facilitate removal of fluid from the chest wall, the client is positioned sitting at the edge of the bed
leaning over the bedside table with the feet supported on a stool. If the client is unable to sit up, the client is
positioned lying in bed on the unaffected side with the head of the bed elevated 30 to 45 degrees.
67. (D) Reliability
  Reliability is consistency of the research instrument. It refers to the repeatability of the instrument in
extracting the same responses upon its repeated administration.
6V. : (A) Keep the identities of the subject secret
  Keeping the identities of the research subject secret will ensure anonymity because this will hinder
providing link between the information given to whoever is its source.
69. (A) Descriptive- correlational
  Descriptive- correlational study is the most appropriate for this study because it studies the variables that
could be the antecedents of the increased incidence of nosocomial infection.
70. (C) Use of laboratory data
  Incidence of nosocomial infection is best collected through the use of biophysiologic measures,
particularly in vitro measurements, hence laboratory data is essential.
71. (B) Quasi-experiment
  Quasi-experiment is done when randomization and control of the variables are not possible.
72. )C) Primary source
  This refers to a primary source which is a direct account of the investigation done by the investigator. In
contrast to this is a secondary source, which is written by someone other than the original researcher.
73. )A) Non-maleficence
  Non-maleficence means do not cause harm or do any action that will cause any harm to the
patient/client. To do good is referred as beneficence.
74. )C) Res ipsa loquitor
  : Res ipsa loquitor literally means the thing speaks for itself. This means in operational terms that the
injury caused is the proof that there was a negligent act.
75. : (B) The Board can investigate violations of the nursing law and code of ethics
  Quasi-judicial power means that the Board of Nursing has the authority to investigate violations of the
nursing law and can issue summons, subpoena or subpoena duces tecum as needed.
76. (C) May apply for re-issuance of his/her license based on certain conditions stipulated in RA 9173
  RA 9173 sec. 24 states that for equity and justice, a revoked license maybe re-issued provided that the
following conditions are met: a) the cause for revocation of license has already been corrected or removed; and, b) at
least four years has elapsed since the license has been revoked.
77. (B) Review related literature
  After formulating and delimiting the research problem, the researcher conducts a review of related
literature to determine the extent of what has been done on the study by previous researchers.
7V. (B) Hawthorne effect
  : Hawthorne effect is based on the study of Elton Mayo and company about the effect of an intervention
done to improve the working conditions of the workers on their productivity. It resulted to an increased productivity
but not due to the intervention but due to the psychological effects of being observed. They performed differently
because they were under observation.
79. (B) Determines the different nationality of patients frequently admitted and decides to get
representations samples from each.
  : Judgment sampling involves including samples according to the knowledge of the investigator about the
participants in the study.
V0. (B) Madeleine Leininger
  : Madeleine Leininger developed the theory on transcultural theory based on her observations on the
behavior of selected people within a culture.
V1. )A) Random
  : Random sampling gives equal chance for all the elements in the population to be picked as part of the
sample.
V2. (A) Degree of agreement and disagreement
  : Likert scale is a 5-point summated scale used to determine the degree of agreement or disagreement of
the respondents to a statement in a study
V3. (B) Sr. Callista Roy
  Sr. Callista Roy developed the Adaptation Model which involves the physiologic mode, self-concept
mode, role function mode and dependence mode.
V4. : (A) Span of control
  : Span of control refers to the number of workers who report directly to a manager.
V5. (B) Autonomy
  Informed consent means that the patient fully understands about the surgery, including the risks involved
and the alternative solutions. In giving consent it is done with full knowledge and is given freely. The action of
allowing the patient to decide whether a surgery is to be done or not exemplifies the bioethical principle of
autonomy.
V6. (C) Avoid wearing canvas shoes.
  The client should be instructed to avoid wearing canvas shoes. Canvas shoes cause the feet to perspire,
which may, in turn, cause skin irritation and breakdown. Both cotton and cornstarch absorb perspiration. The client
should be instructed to cut toenails straight across with nail clippers.
V7. (D) Ground beef patties
  : Meat is an excellent source of complete protein, which this client needs to repair the tissue breakdown
caused by pressure ulcers. Oranges and broccoli supply vitamin C but not protein. Ice cream supplies only some
incomplete protein, making it less helpful in tissue repair.
VV. (D) Sims¶ left lateral
  The Sims' left lateral position is the most common position used to administer a cleansing enema
because it allows gravity to aid the flow of fluid along the curve of the sigmoid colon. If the client can't assume this
position nor has poor sphincter control, the dorsal recumbent or right lateral position may be used. The supine and
prone positions are inappropriate and uncomfortable for the client.
V9. (A) Arrange for typing and cross matching of the client¶s blood.
  The nurse first arranges for typing and cross matching of the client's blood to ensure compatibility with
donor blood. The other options, although appropriate when preparing to administer a blood transfusion, come later.
90. (A) Independent
  : Nursing interventions are classified as independent, interdependent, or dependent. Altering the drug
schedule to coincide with the client's daily routine represents an independent intervention, whereas consulting with
the physician and pharmacist to change a client's medication because of adverse reactions represents an
interdependent intervention. Administering an already-prescribed drug on time is a dependent intervention. An
intradependent nursing intervention doesn't exist.
91. (D) Evaluation
  The nursing actions described constitute evaluation of the expected outcomes. The findings show that
the expected outcomes have been achieved. Assessment consists of the client's history, physical examination, and
laboratory studies. Analysis consists of considering assessment information to derive the appropriate nursing
diagnosis. Implementation is the phase of the nursing process where the nurse puts the plan of care into action.
92. : (B) To observe the lower extremities
  Elastic stockings are used to promote venous return. The nurse needs to remove them once per day to
observe the condition of the skin underneath the stockings. Applying the stockings increases blood flow to the heart.
When the stockings are in place, the leg muscles can still stretch and relax, and the veins can fill with blood.
93. (A) Instructing the client to report any itching, swelling, or dyspnea.
  : Because administration of blood or blood products may cause serious adverse effects such as allergic
reactions, the nurse must monitor the client for these effects. Signs and symptoms of life-threatening allergic
reactions include itching, swelling, and dyspnea. Although the nurse should inform the client of the duration of the
transfusion and should document its administration, these actions are less critical to the client's immediate health.
The nurse should assess vital signs at least hourly during the transfusion.
94. : (B) Decrease the rate of feedings and the concentration of the formula.
  : Complaints of abdominal discomfort and nausea are common in clients receiving tube feedings.
Decreasing the rate of the feeding and the concentration of the formula should decrease the client's discomfort.
Feedings are normally given at room temperature to minimize abdominal cramping. To prevent aspiration during
feeding, the head of the client's bed should be elevated at least 30 degrees. Also, to prevent bacterial growth, feeding
containers should be routinely changed every V to 12 hours.
95. (D) Roll the vial gently between the palms.
  Rolling the vial gently between the palms produces heat, which helps dissolve the medication. Doing
nothing or inverting the vial wouldn't help dissolve the medication. Shaking the vial vigorously could cause the
medication to break down, altering its action.
96. (B) Assist the client to the semi-Fowler position if possible.
  By assisting the client to the semi-Fowler position, the nurse promotes easier chest expansion, breathing,
and oxygen intake. The nurse should secure the elastic band so that the face mask fits comfortably and snugly rather
than tightly, which could lead to irritation. The nurse should apply the face mask from the client's nose down to the
chin ² not vice versa. The nurse should check the connectors between the oxygen equipment and humidifier to
ensure that they're airtight; loosened connectors can cause loss of oxygen.
97. (B) 4 hours
  A unit of packed RBCs may be given over a period of between 1 and 4 hours. It shouldn't infuse for longer than 4
hours because the risk of contamination and sepsis increases after that time. Discard or return to the blood bank any blood not
given within this time, according to facility policy.
9V. : (B) Immediately before administering the next dose.
  Measuring the blood drug concentration helps determine whether the dosing has achieved the therapeutic goal. For
measurement of the trough, or lowest, blood level of a drug, the nurse draws a blood sample immediately before administering
the next dose. Depending on the drug's duration of action and half-life, peak blood drug levels typically are drawn after
administering the next dose.
99. (A) The nurse can implement medication orders quickly.
  A floor stock system enables the nurse to implement medication orders quickly. It doesn't allow for pharmacist input,
nor does it minimize transcription errors or reinforce accurate calculations.
100. (C) Shifting dullness over the abdomen.
  Shifting dullness over the abdomen indicates ascites, an abnormal finding. The other options are normal abdominal
findings.


   ( 
 
    
   
1. (A) Inevitable
  An inevitable abortion is termination of pregnancy that cannot be prevented. Moderate to severe
bleeding with mild cramping and cervical dilation would be noted in this type of abortion.
2. (B) History of syphilis
  Maternal infections such as syphilis, toxoplasmosis, and rubella are causes of spontaneous abortion.
3. : (C) Monitoring apical pulse
  Nursing care for the client with a possible ectopic pregnancy is focused on preventing or identifying
hypovolemic shock and controlling pain. An elevated pulse rate is an indicator of shock.
4. : (B) Increased caloric intake
  Glucose crosses the placenta, but insulin does not. High fetal demands for glucose, combined with the
insulin resistance caused by hormonal changes in the last half of pregnancy can result in elevation of maternal blood
glucose levels. This increases the mother¶s demand for insulin and is referred to as the diabetogenic effect of
pregnancy.
5. (A) Excessive fetal activity.
  : The most common signs and symptoms of hydatidiform mole includes elevated levels of human
chorionic gonadotropin, vaginal bleeding, larger than normal uterus for gestational age, failure to detect fetal heart
activity even with sensitive instruments, excessive nausea and vomiting, and early development of pregnancy-
induced hypertension. Fetal activity would not be noted.
6. (B) Absent patellar reflexes
  Absence of patellar reflexes is an indicator of hypermagnesemia, which requires administration of
calcium gluconate.
7. (C) Presenting part in 2 cm below the plane of the ischial spines.
  Fetus at station plus two indicates that the presenting part is 2 cm below the plane of the ischial spines.
V. (A) Contractions every 1 . minutes lasting 70-V0 seconds.
  Contractions every 1 . minutes lasting 70-V0 seconds, is indicative of hyperstimulation of the uterus,
which could result in injury to the mother and the fetus if Pitocin is not discontinued.
9. (C) EKG tracings
  A potential side effect of calcium gluconate administration is cardiac arrest. Continuous monitoring of
cardiac activity (EKG) throught administration of calcium gluconate is an essential part of care.
10. : (D) First low transverse caesarean was for breech position. Fetus in this pregnancy is in a vertex
presentation.
  : This type of client has no obstetrical indication for a caesarean section as she did with her first caesarean
delivery.
11. : (A) Talk to the mother first and then to the toddler.
  When dealing with a crying toddler, the best approach is to talk to the mother and ignore the toddler
first. This approach helps the toddler get used to the nurse before she attempts any procedures. It also gives the
toddler an opportunity to see that the mother trusts the nurse.
12. : (D) Place the infant¶s arms in soft elbow restraints.
  : Soft restraints from the upper arm to the wrist prevent the infant from touching her lip but allow him to
hold a favorite item such as a blanket. Because they could damage the operative site, such as objects as pacifiers,
suction catheters, and small spoons shouldn¶t be placed in a baby¶s mouth after cleft repair. A baby in a prone
position may rub her face on the sheets and traumatize the operative site. The suture line should be cleaned gently to
prevent infection, which could interfere with healing and damage the cosmetic appearance of the repair.
13. : (B) Allow the infant to rest before feeding.
  Because feeding requires so much energy, an infant with heart failure should rest before feeding.
14. (C) Iron-rich formula only.
  The infants at age 5 months should receive iron-rich formula and that they shouldn¶t receive solid food,
even baby food until age 6 months.
15. (D) 10 months
  : A 10 month old infant can sit alone and understands object permanence, so he would look for the hidden
toy. At age 4 to 6 months, infants can¶t sit securely alone. At age V months, infants can sit securely alone but cannot
understand the permanence of objects.
16. (D) Public health nursing focuses on preventive, not curative, services.
  : The catchments area in PHN consists of a residential community, many of whom are well individuals
who have greater need for preventive rather than curative services.
17. (B) Efficiency
  Efficiency is determining whether the goals were attained at the least possible cost.
1V. (D) Rural Health Unit
  R.A. 7160 devolved basic health services to local government units (LGU¶s ). The public health nurse is
an employee of the LGU.
19. : (A) Mayor
  The local executive serves as the chairman of the Municipal Health Board.
20. )A) 1
  : Each rural health midwife is given a population assignment of about 5,000.
21. (B) Health education and community organizing are necessary in providing community health services.
  The community health nurse develops the health capability of people through health education and
community organizing activities.
22. (B) Measles
  : Presidential Proclamation No. 4 is on the Ligtas Tigdas Program.
23. (D) Core group formation
  : In core group formation, the nurse is able to transfer the technology of community organizing to the
potential or informal community leaders through a training program.
24. (D) To maximize the community¶s resources in dealing with health problems.
  Community organizing is a developmental service, with the goal of developing the people¶s self-reliance
in dealing with community health problems. A, B and C are objectives of contributory objectives to this goal.
25. (D) Terminal
  : Tertiary prevention involves rehabilitation, prevention of permanent disability and disability limitation
appropriate for convalescents, the disabled, complicated cases and the terminally ill (those in the terminal stage of a
disease).
26. )A) Intrauterine fetal death.
  Intrauterine fetal death, abruptio placentae, septic shock, and amniotic fluid embolism may trigger
normal clotting mechanisms; if clotting factors are depleted, DIC may occur. Placenta accreta, dysfunctional labor,
and premature rupture of the membranes aren't associated with DIC.
27. (C) 120 to 160 beats/minute
  : A rate of 120 to 160 beats/minute in the fetal heart appropriate for filling the heart with blood and
pumping it out to the system.
2V. (A) Change the diaper more often.
  Decreasing the amount of time the skin comes contact with wet soiled diapers will help heal the
irritation.
29. (D) Endocardial cushion defect
  : Endocardial cushion defects are seen most in children with Down syndrome, asplenia, or polysplenia.
30. : (B) Decreased urine output
  : Decreased urine output may occur in clients receiving I.V. magnesium and should be monitored closely
to keep urine output at greater than 30 ml/hour, because magnesium is excreted through the kidneys and can easily
accumulate to toxic levels.
31. (A) Menorrhagia
  Menorrhagia is an excessive menstrual period.
32. (C) Blood typing
  : Blood type would be a critical value to have because the risk of blood loss is always a potential
complication during the labor and delivery process. Approximately 40% of a woman¶s cardiac output is delivered to
the uterus, therefore, blood loss can occur quite rapidly in the event of uncontrolled bleeding.
33. (D) Physiologic anemia
  Hemoglobin values and hematocrit decrease during pregnancy as the increase in plasma volume exceeds
the increase in red blood cell production.
34. (D) A 2 year old infant with stridorous breath sounds, sitting up in his mother¶s arms and drooling.
  : The infant with the airway emergency should be treated first, because of the risk of epiglottitis.
35. (A) Placenta previa
  Placenta previa with painless vaginal bleeding.
36. : (D) Early in the morning
  : Based on the nurse¶s knowledge of microbiology, the specimen should be collected early in the morning.
The rationale for this timing is that, because the female worm lays eggs at night around the perineal area, the first
bowel movement of the day will yield the best results. The specific type of stool specimen used in the diagnosis of
pinworms is called the tape test.
37. (A) Irritability and seizures
  : Lead poisoning primarily affects the CNS, causing increased intracranial pressure. This condition results
in irritability and changes in level of consciousness, as well as seizure disorders, hyperactivity, and learning
disabilities.
3V. (D) ³I really need to use the diaphragm and jelly most during the middle of my menstrual cycle´.
  The woman must understand that, although the ³fertile´ period is approximately mid-cycle, hormonal
variations do occur and can result in early or late ovulation. To be effective, the diaphragm should be inserted before
every intercourse.
39. (C) Restlessness
  : In a child, restlessness is the earliest sign of hypoxia. Late signs of hypoxia in a child are associated with
a change in color, such as pallor or cyanosis.
40. r: (B) Walk one step ahead, with the child¶s hand on the nurse¶s elbow.
  : This procedure is generally recommended to follow in guiding a person who is blind.
41. : (A) Loud, machinery-like murmur.
  e: A loud, machinery-like murmur is a characteristic finding associated with patent ductus arteriosus.
42. : (C) More oxygen, and the newborn¶s metabolic rate increases.
  : When cold, the infant requires more oxygen and there is an increase in metabolic rate. Non-shievering
thermogenesis is a complex process that increases the metabolic rate and rate of oxygen consumption, therefore, the
newborn increase heat production.
43. (D) Voided
  Before administering potassium I.V. to any client, the nurse must first check that the client¶s kidneys are
functioning and that the client is voiding. If the client is not voiding, the nurse should withhold the potassium and
notify the physician.
44. (c) Laundry detergent
  : Eczema or dermatitis is an allergic skin reaction caused by an offending allergen. The topical allergen
that is the most common causative factor is laundry detergent.
45. (A) 6 inches
  This distance allows for easy flow of the formula by gravity, but the flow will be slow enough not to
overload the stomach too rapidly.
46. (A) The older one gets, the more susceptible he becomes to the complications of chicken pox.
  : Chicken pox is usually more severe in adults than in children. Complications, such as pneumonia, are
higher in incidence in adults.
47. (D) Consult a physician who may give them rubella immunoglobulin.
  : Rubella vaccine is made up of attenuated German measles viruses. This is contraindicated in pregnancy.
Immune globulin, a specific prophylactic against German measles, may be given to pregnant women.
4V. : (A) Contact tracing
  : Contact tracing is the most practical and reliable method of finding possible sources of person-to-person
transmitted infections, such as sexually transmitted diseases.
49. : (D) Leptospirosis
  : Leptospirosis is transmitted through contact with the skin or mucous membrane with water or moist soil
contaminated with urine of infected animals, like rats.
50. : (B) Cholera
  Passage of profuse watery stools is the major symptom of cholera. Both amebic and bacillary dysentery
are characterized by the presence of blood and/or mucus in the stools. Giardiasis is characterized by fat
malabsorption and, therefore, steatorrhea.
51. (A) Hemophilus influenzae
  Hemophilus meningitis is unusual over the age of 5 years. In developing countries, the peak incidence is
in children less than 6 months of age. Morbillivirus is the etiology of measles. Streptococcus pneumonia and
Neisseria meningitidis may cause meningitis, but age distribution is not specific in young children.
52. (B) Buccal mucosa
  Koplik¶s spot may be seen on the mucosa of the mouth or the throat.
53. : (A) 3 seconds
  Adequate blood supply to the area allows the return of the color of the nailbed within 3 seconds.
54. (B) Severe dehydration
  : The order of priority in the management of severe dehydration is as follows: intravenous fluid therapy,
referral to a facility where IV fluids can be initiated within 30 minutes, Oresol or nasogastric tube. When the
foregoing measures are not possible or effective, then urgent referral to the hospital is done.
55. (A) 45 infants
  To estimate the number of infants, multiply total population by 3%.
56. : (A) DPT
  : DPT is sensitive to freezing. The appropriate storage temperature of DPT is 2 to V° C only. OPV and
measles vaccine are highly sensitive to heat and require freezing. MMR is not an immunization in the Expanded
Program on Immunization.
57. (C) Proper use of sanitary toilets
  The ova of the parasite get out of the human body together with feces. Cutting the cycle at this stage is
the most effective way of preventing the spread of the disease to susceptible hosts.
5V. (D) 5 skin lesions, positive slit skin smear
  : A multibacillary leprosy case is one who has a positive slit skin smear and at least 5 skin lesions.
59. (C) Thickened painful nerves
  The lesion of leprosy is not macular. It is characterized by a change in skin color (either reddish or
whitish) and loss of sensation, sweating and hair growth over the lesion. Inability to close the eyelids
(lagophthalmos) and sinking of the nosebridge are late symptoms.
60. : (B) Ask where the family resides.
  Because malaria is endemic, the first question to determine malaria risk is where the client¶s family
resides. If the area of residence is not a known endemic area, ask if the child had traveled within the past 6 months,
where she was brought and whether she stayed overnight in that area.
61. : (A) Inability to drink
  A sick child aged 2 months to 5 years must be referred urgently to a hospital if he/she has one or more of
the following signs: not able to feed or drink, vomits everything, convulsions, abnormally sleepy or difficult to
awaken.
62. : (A) Refer the child urgently to a hospital for confinement.
  ³Baggy pants´ is a sign of severe marasmus. The best management is urgent referral to a hospital.
63. : (D) Let the child rest for 10 minutes then continue giving Oresol more slowly.
  : If the child vomits persistently, that is, he vomits everything that he takes in, he has to be referred
urgently to a hospital. Otherwise, vomiting is managed by letting the child rest for 10 minutes and then continuing
with Oresol administration. Teach the mother to give Oresol more slowly.
64. (B) Some dehydration
  : Using the assessment guidelines of IMCI, a child (2 months to 5 years old) with diarrhea is classified as
having SOME DEHYDRATION if he shows 2 or more of the following signs: restless or irritable, sunken eyes, the
skin goes back slow after a skin pinch.
65. : (C) Normal
  : In IMCI, a respiratory rate of 50/minute or more is fast breathing for an infant aged 2 to 12 months.
66. : (A) 1 year
  The baby will have passive natural immunity by placental transfer of antibodies. The mother will have
active artificial immunity lasting for about 10 years. 5 doses will give the mother lifetime protection.
67. (B) 4 hours
  While the unused portion of other biologicals in EPI may be given until the end of the day, only BCG is
discarded 4 hours after reconstitution. This is why BCG immunization is scheduled only in the morning.
6V. (B) 6 months
  After 6 months, the baby¶s nutrient needs, especially the baby¶s iron requirement, can no longer be
provided by mother¶s milk alone.
69. (C: 24 weeks
  At approximately 23 to 24 weeks¶ gestation, the lungs are developed enough to sometimes maintain
extrauterine life. The lungs are the most immature system during the gestation period. Medical care for premature
labor begins much earlier (aggressively at 21 weeks¶ gestation)
70. (B) Sudden infant death syndrome (SIDS)
  : Supine positioning is recommended to reduce the risk of SIDS in infancy. The risk of aspiration is
slightly increased with the supine position. Suffocation would be less likely with an infant supine than prone and the
position for GER requires the head of the bed to be elevated.
71. (C) Decreased temperature
  Temperature instability, especially when it results in a low temperature in the neonate, may be a sign of
infection. The neonate¶s color often changes with an infection process but generally becomes ashen or mottled. The
neonate with an infection will usually show a decrease in activity level or lethargy.
72. (D) Polycythemia probably due to chronic fetal hypoxia
  : The small-for-gestation neonate is at risk for developing polycythemia during the transitional period in
an attempt to decrease hypoxia. The neonates are also at increased risk for developing hypoglycemia and
hypothermia due to decreased glycogen stores.
73. (C) Desquamation of the epidermis
  Postdate fetuses lose the vernix caseosa, and the epidermis may become desquamated. These neonates
are usually very alert. Lanugo is missing in the postdate neonate.
74. (C) Respiratory depression
  : Magnesium sulfate crosses the placenta and adverse neonatal effects are respiratory depression,
hypotonia, and bradycardia. The serum blood sugar isn¶t affected by magnesium sulfate. The neonate would be
floppy, not jittery.
75. (C) Respiratory rate 40 to 60 breaths/minute
  A respiratory rate 40 to 60 breaths/minute is normal for a neonate during the transitional period. Nasal
flaring, respiratory rate more than 60 breaths/minute, and audible grunting are signs of respiratory distress.
76. (C) Keep the cord dry and open to air
  Keeping the cord dry and open to air helps reduce infection and hastens drying. Infants aren¶t given tub
bath but are sponged off until the cord falls off. Petroleum jelly prevents the cord from drying and encourages
infection. Peroxide could be painful and isn¶t recommended.
77. : (B) Conjunctival hemorrhage
  : Conjunctival hemorrhages are commonly seen in neonates secondary to the cranial pressure applied
during the birth process. Bulging fontanelles are a sign of intracranial pressure. Simian creases are present in 40% of
the neonates with trisomy 21. Cystic hygroma is a neck mass that can affect the airway.
7V. (B) To assess for prolapsed cord
  After a client has an amniotomy, the nurse should assure that the cord isn't prolapsed and that the baby
tolerated the procedure well. The most effective way to do this is to check the fetal heart rate. Fetal well-being is
assessed via a nonstress test. Fetal position is determined by vaginal examination. Artificial rupture of membranes
doesn't indicate an imminent delivery.
79. : (D) The parents¶ interactions with each other.
  Parental interaction will provide the nurse with a good assessment of the stability of the family's home
life but it has no indication for parental bonding. Willingness to touch and hold the newborn, expressing interest
about the newborn's size, and indicating a desire to see the newborn are behaviors indicating parental bonding.
V0. (B) Instructing the client to use two or more peripads to cushion the area
  Using two or more peripads would do little to reduce the pain or promote perineal healing. Cold
applications, sitz baths, and Kegel exercises are important measures when the client has a fourth-degree laceration.
V1. (C) ³What is your expected due date?´
  When obtaining the history of a client who may be in labor, the nurse's highest priority is to determine
her current status, particularly her due date, gravidity, and parity. Gravidity and parity affect the duration of labor
and the potential for labor complications. Later, the nurse should ask about chronic illnesses, allergies, and support
persons.
V2. (D) Aspirate the neonate¶s nose and mouth with a bulb syringe.
  The nurse's first action should be to clear the neonate's airway with a bulb syringe. After the airway is
clear and the neonate's color improves, the nurse should comfort and calm the neonate. If the problem recurs or the
neonate's color doesn't improve readily, the nurse should notify the physician. Administering oxygen when the
airway isn't clear would be ineffective.
V3. : (C) Conducting a bedside ultrasound for an amniotic fluid index.
  It isn't within a nurse's scope of practice to perform and interpret a bedside ultrasound under these conditions and
without specialized training. Observing for pooling of straw-colored fluid, checking vaginal discharge with nitrazine paper, and
observing for flakes of vernix are appropriate assessments for determining whether a client has ruptured membranes.
V4. : (C) Monitor partial pressure of oxygen (Pao2) levels.
  Monitoring PaO2 levels and reducing the oxygen concentration to keep PaO2 within normal limits
reduces the risk of retinopathy of prematurity in a premature infant receiving oxygen. Covering the infant's eyes and
humidifying the oxygen don't reduce the risk of retinopathy of prematurity. Because cooling increases the risk of
acidosis, the infant should be kept warm so that his respiratory distress isn't aggravated.
V5. (A) 110 to 130 calories per kg.
  Calories per kg is the accepted way of determined appropriate nutritional intake for a newborn. The
recommended calorie requirement is 110 to 130 calories per kg of newborn body weight. This level will maintain a
consistent blood glucose level and provide enough calories for continued growth and development.
V6. (C) 30 to 32 weeks
  Individual twins usually grow at the same rate as singletons until 30 to 32 weeks¶ gestation, then twins
don¶t¶ gain weight as rapidly as singletons of the same gestational age. The placenta can no longer keep pace with
the nutritional requirements of both fetuses after 32 weeks, so there¶s some growth retardation in twins if they
remain    at 3V to 40 weeks.
V7. (A) conjoined twins
  The type of placenta that develops in monozygotic twins depends on the time at which cleavage of the
ovum occurs. Cleavage in conjoined twins occurs more than 13 days after fertilization. Cleavage that occurs less
than 3 day after fertilization results in diamniotic dicchorionic twins. Cleavage that occurs between days 3 and V
results in diamniotic monochorionic twins. Cleavage that occurs between days V to 13 result in monoamniotic
monochorionic twins.
VV. (D) Ultrasound
  Once the mother and the fetus are stabilized, ultrasound evaluation of the placenta should be done to
determine the cause of the bleeding. Amniocentesis is contraindicated in placenta previa. A digital or speculum
examination shouldn¶t be done as this may lead to severe bleeding or hemorrhage. External fetal monitoring won¶t
detect a placenta previa, although it will detect fetal distress, which may result from blood loss or placenta
separation.
V9. (A) Increased tidal volume
  A pregnant client breathes deeper, which increases the tidal volume of gas moved in and out of the
respiratory tract with each breath. The expiratory volume and residual volume decrease as the pregnancy progresses.
The inspiratory capacity increases during pregnancy. The increased oxygen consumption in the pregnant client is
15% to 20% greater than in the nonpregnant state.
90. : (A) Diet
  : Clients with gestational diabetes are usually managed by diet alone to control their glucose intolerance.
Oral hypoglycemic drugs are contraindicated in pregnancy. Long-acting insulin usually isn¶t needed for blood
glucose control in the client with gestational diabetes.
91. : (D) Seizure
  The anticonvulsant mechanism of magnesium is believes to depress seizure foci in the brain and
peripheral neuromuscular blockade. Hypomagnesemia isn¶t a complication of preeclampsia. Antihypertensive drug
other than magnesium are preferred for sustained hypertension. Magnesium doesn¶t help prevent hemorrhage in
preeclamptic clients.
92. : (C) I.V. fluids
  A sickle cell crisis during pregnancy is usually managed by exchange transfusion oxygen, and L.V.
Fluids. The client usually needs a stronger analgesic than acetaminophen to control the pain of a crisis.
Antihypertensive drugs usually aren¶t necessary. Diuretic wouldn¶t be used unless fluid overload resulted.
93. er: (A) Calcium gluconate (Kalcinate)
  Calcium gluconate is the antidote for magnesium toxicity. Ten milliliters of 10% calcium gluconate is
given L.V. push over 3 to 5 minutes. Hydralazine is given for sustained elevated blood pressure in preeclamptic
clients. Rho (D) immune globulin is given to women with Rh-negative blood to prevent antibody formation from
RH-positive conceptions. Naloxone is used to correct narcotic toxicity.
94. (B) An indurated wheal over 10 mm in diameter appears in 4V to
72 hours.
  : A positive PPD result would be an indurated wheal over 10 mm in diameter that appears in 4V to 72
hours. The area must be a raised wheal, not a flat circumcised area to be considered positive.
95. (C) Pyelonephritis
  The symptoms indicate acute pyelonephritis, a serious condition in a pregnant client. UTI symptoms
include dysuria, urgency, frequency, and suprapubic tenderness. Asymptomatic bacteriuria doesn¶t cause symptoms.
Bacterial vaginosis causes milky white vaginal discharge but no systemic symptoms.
96. (B) Rh-positive fetal blood crosses into maternal blood, stimulating maternal antibodies.
  Rh isoimmunization occurs when Rh-positive fetal blood cells cross into the maternal circulation and
stimulate maternal antibody production. In subsequent pregnancies with Rh-positive fetuses, maternal antibodies
may cross back into the fetal circulation and destroy the fetal blood cells.
97. (C) Supine position
  The supine position causes compression of the client's aorta and inferior vena cava by the fetus. This, in
turn, inhibits maternal circulation, leading to maternal hypotension and, ultimately, fetal hypoxia. The other
positions promote comfort and aid labor progress. For instance, the lateral, or side-lying, position improves maternal
and fetal circulation, enhances comfort, increases maternal relaxation, reduces muscle tension, and eliminates
pressure points. The squatting position promotes comfort by taking advantage of gravity. The standing position also
takes advantage of gravity and aligns the fetus with the pelvic angle.
9V. (B) Irritability and poor sucking.
  Neonates of heroin-addicted mothers are physically dependent on the drug and experience withdrawal
when the drug is no longer supplied. Signs of heroin withdrawal include irritability, poor sucking, and restlessness.
Lethargy isn't associated with neonatal heroin addiction. A flattened nose, small eyes, and thin lips are seen in
infants with fetal alcohol syndrome. Heroin use during pregnancy hasn't been linked to specific congenital
anomalies.
99. (A) 7th to 9th day postpartum
  The normal involutional process returns the uterus to the pelvic cavity in 7 to 9 days. A significant
involutional complication is the failure of the uterus to return to the pelvic cavity within the prescribed time period.
This is known as subinvolution.
100. (B) Uterine atony
  Multiple fetuses, extended labor stimulation with oxytocin, and traumatic delivery commonly are
associated with uterine atony, which may lead to postpartum hemorrhage. Uterine inversion may precede or follow
delivery and commonly results from apparent excessive traction on the umbilical cord and attempts to deliver the
placenta manually. Uterine involution and some uterine discomfort are normal after delivery.


   (   c   
c      
1. (C) Loose, bloody
  : Normal bowel function and soft-formed stool usually do not occur until around the seventh day
following surgery. The stool consistency is related to how much water is being absorbed.
2. (A) On the client¶s right side
  : The client has left visual field blindness. The client will see only from the right side.
3. : (C) Check respirations, stabilize spine, and check circulation
  : Checking the airway would be priority, and a neck injury should be suspected.
4. (D) Decreasing venous return through vasodilation.
  The significant effect of nitroglycerin is vasodilation and decreased venous return, so the heart does not
have to work hard.
5. (A) Call for help and note the time.
  : Having established, by stimulating the client, that the client is unconscious rather than sleep, the nurse
should immediately call for help. This may be done by dialing the operator from the client¶s phone and giving the
hospital code for cardiac arrest and the client¶s room number to the operator, of if the phone is not available, by
pulling the emergency call button. Noting the time is important baseline information for cardiac arrest procedure.
6. : (C) Make sure that the client takes food and medications at prescribed intervals.
  : Food and drug therapy will prevent the accumulation of hydrochloric acid, or will neutralize and buffer
the acid that does accumulate.
7. (B) Continue treatment as ordered.
  : The effects of heparin are monitored by the PTT is normally 30 to 45 seconds; the therapeutic level is 1.5
to 2 times the normal level.
V. : (B) In the operating room.
  The stoma drainage bag is applied in the operating room. Drainage from the ileostomy contains
secretions that are rich in digestive enzymes and highly irritating to the skin. Protection of the skin from the effects
of these enzymes is begun at once. Skin exposed to these enzymes even for a short time becomes reddened, painful,
and excoriated.
9. : (B) Flat on back.
  : To avoid the complication of a painful spinal headache that can last for several days, the client is kept in
flat in a supine position for approximately 4 to 12 hours postoperatively. Headaches are believed to be causes by the
seepage of cerebral spinal fluid from the puncture site. By keeping the client flat, cerebral spinal fluid pressures are
equalized, which avoids trauma to the neurons.
10. (C) The client is oriented when aroused from sleep, and goes back to sleep immediately.
  This finding suggest that the level of consciousness is decreasing.
11. ) *Altered mental status and dehydration
  Fever, chills, hemortysis, dyspnea, cough, and pleuritic chest pain are the common symptoms of
pneumonia, but elderly clients may first appear with only an altered lentil status and dehydration due to a blunted
immune response.
12. (B) Chills, fever, night sweats, and hemoptysis
  Typical signs and symptoms are chills, fever, night sweats, and hemoptysis. Chest pain may be present
from coughing, but isn¶t usual. Clients with TB typically have low-grade fevers, not higher than 102°F (3V.9°C).
Nausea, headache, and photophobia aren¶t usual TB symptoms.
13. :(A) Acute asthma
  : Based on the client¶s history and symptoms, acute asthma is the most likely diagnosis. He¶s unlikely to
have bronchial pneumonia without a productive cough and fever and he¶s too young to have developed (COPD) and
emphysema.
14. (B) Respiratory arrest
  : Narcotics can cause respiratory arrest if given in large quantities. It¶s unlikely the client will have asthma
attack or a seizure or wake up on his own.
15. (D) Decreased vital capacity
  : Reduction in vital capacity is a normal physiologic changes include decreased elastic recoil of the lungs,
fewer functional capillaries in the alveoli, and an increased in residual volume.
16. (C) Presence of premature ventricular contractions (PVCs) on a cardiac monitor.
  Lidocaine drips are commonly used to treat clients whos arrhythmias haven¶t been controlled with oral
medication and who are having PVCs that are visible on the cardiac monitor. SaO2, blood pressure, and ICP are
important factors but aren¶t as significant as PVCs in the situation.
17. : (B) Avoid foods high in vitamin K
  The client should avoid consuming large amounts of vitamin K because vitamin K can interfere with
anticoagulation. The client may need to report diarrhea, but isn¶t effect of taking an anticoagulant. An electric razor-
not a straight razor-should be used to prevent cuts that cause bleeding. Aspirin may increase the risk of bleeding;
acetaminophen should be used to pain relief.
1V. : (C) Clipping the hair in the area
  : Hair can be a source of infection and should be removed by clipping. Shaving the area can cause skin
abrasions and depilatories can irritate the skin.
19. : (A) Bone fracture
  : Bone fracture is a major complication of osteoporosis that results when loss of calcium and phosphate
increased the fragility of bones. Estrogen deficiencies result from menopause-not osteoporosis. Calcium and vitamin
D supplements may be used to support normal bone metabolism, But a negative calcium balance isn¶t a
complication of osteoporosis. Dowager¶s hump results from bone fractures. It develops when repeated vertebral
fractures increase spinal curvature.
20. r: (C) Changes from previous examinations.
  : Women are instructed to examine themselves to discover changes that have occurred in the breast. Only
a physician can diagnose lumps that are cancerous, areas of thickness or fullness that signal the presence of a
malignancy, or masses that are fibrocystic as opposed to malignant.
21. (C) Balance the client¶s periods of activity and rest.
  A client with hyperthyroidism needs to be encouraged to balance periods of activity and rest. Many
clients with hyperthyroidism are hyperactive and complain of feeling very warm.
22. : (B) Increase his activity level.
  : The client should be encouraged to increase his activity level. Maintaining an ideal weight; following a
low-cholesterol, low sodium diet; and avoiding stress are all important factors in decreasing the risk of
atherosclerosis.
23. : (A) Laminectomy
  The client who has had spinal surgery, such as laminectomy, must be log rolled to keep the spinal
column straight when turning. Thoracotomy and cystectomy may turn themselves or may be assisted into a
comfortable position. Under normal circumstances, hemorrhoidectomy is an outpatient procedure, and the client
may resume normal activities immediately after surgery.
24. (D) Avoiding straining during bowel movement or bending at the waist.
  : The client should avoid straining, lifting heavy objects, and coughing harshly because these activities
increase intraocular pressure. Typically, the client is instructed to avoid lifting objects weighing more than 15 lb
(7kg) ± not 5lb. instruct the client when lying in bed to lie on either the side or back. The client should avoid bright
light by wearing sunglasses.
25. (D) Before age 20.
  Testicular cancer commonly occurs in men between ages 20 and 30. A male client should be taught how
to perform testicular selfexamination before age 20, preferably when he enters his teens.
26. (B) Place a saline-soaked sterile dressing on the wound.
  The nurse should first place saline-soaked sterile dressings on the open wound to prevent tissue drying
and possible infection. Then the nurse should call the physician and take the client¶s vital signs. The dehiscence
needs to be surgically closed, so the nurse should never try to close it.
27. (A) A progressively deeper breaths followed by shallower breaths with apneic periods.
  Cheyne-Strokes respirations are breaths that become progressively deeper fallowed by shallower
respirations with apneas periods. Biot¶s respirations are rapid, deep breathing with abrupt pauses between each
breath, and equal depth between each breath. Kussmaul¶s respirationa are rapid, deep breathing without pauses.
Tachypnea is shallow breathing with increased respiratory rate.
2V. (B) Fine crackles
  : Fine crackles are caused by fluid in the alveoli and commonly occur in clients with heart failure.
Tracheal breath sounds are auscultated over the trachea. Coarse crackles are caused by secretion accumulation in the
airways. Friction rubs occur with pleural inflammation.
29. (B) The airways are so swollen that no air cannot get through
  : During an acute attack, wheezing may stop and breath sounds become inaudible because the airways are
so swollen that air can¶t get through. If the attack is over and swelling has decreased, there would be no more
wheezing and less emergent concern. Crackles do not replace wheezes during an acute asthma attack.
30. (D) Place the client on his side, remove dangerous objects, and protect his head.
  : During the active seizure phase, initiate precautions by placing the client on his side, removing
dangerous objects, and protecting his head from injury. A bite block should never be inserted during the active
seizure phase. Insertion can break the teeth and lead to aspiration.
31. : (B) Kinked or obstructed chest tube
  : Kinking and blockage of the chest tube is a common cause of a tension pneumothorax. Infection and
excessive drainage won¶t cause a tension pneumothorax. Excessive water won¶t affect the chest tube drainage.
32. : (D) Stay with him but not intervene at this time.
  If the client is coughing, he should be able to dislodge the object or cause a complete obstruction. If
complete obstruction occurs, the nurse should perform the abdominal thrust maneuver with the client standing. If the
client is unconscious, she should lay him down. A nurse should never leave a choking client alone.
33. (B) Current health promotion activities
  Recognizing an individual¶s positive health measures is very useful. General health in the previous 10
years is important, however, the current activities of an V4 year old client are most significant in planning care.
Family history of disease for a client in later years is of minor significance. Marital status information may be
important for discharge planning but is not as significant for addressing the immediate medical problem.
34. : (C) Place the client in a side lying position, with the head of the bed lowered.
  The client should be positioned in a side-lying position with the head of the bed lowered to prevent
aspiration. A small amount of toothpaste should be used and the mouth swabbed or suctioned to remove pooled
secretions. Lemon glycerin can be drying if used for extended periods. Brushing the teeth with the client lying
supine may lead to aspiration. Hydrogen peroxide is caustic to tissues and should not be used.
35. (C) Pneumonia
  : Fever productive cough and pleuritic chest pain are common signs and symptoms of pneumonia. The
client with ARDS has dyspnea and hypoxia with worsening hypoxia over time, if not treated aggressively. Pleuritic
chest pain varies with respiration, unlike the constant chest pain during an MI; so this client most likely isn¶t having
an MI. the client with TB typically has a cough producing blood-tinged sputum. A sputum culture should be
obtained to confirm the nurse¶s suspicions.
36. )*A 43-yesr-old homeless man with a history of alcoholism
  Clients who are economically disadvantaged, malnourished, and have reduced immunity, such as a client
with a history of alcoholism, are at extremely high risk for developing TB. A high school student, daycare worker,
and businessman probably have a much low risk of contracting TB.
37. )*To determine the extent of lesions
  : If the lesions are large enough, the chest X-ray will show their presence in the lungs. Sputum culture
confirms the diagnosis. There can be false-positive and false-negative skin test results. A chest X-ray can¶t
determine if this is a primary or secondary infection.
3V. )+*Bronchodilators
  Bronchodilators are the first line of treatment for asthma because broncho-constriction is the cause of
reduced airflow. Betaadrenergic blockers aren¶t used to treat asthma and can cause bronchoconstriction. Inhaled oral
steroids may be given to reduce the inflammation but aren¶t used for emergency relief.
39. )*Chronic obstructive bronchitis
  : Because of this extensive smoking history and symptoms the client most likely has chronic obstructive
bronchitis. Client with ARDS have acute symptoms of hypoxia and typically need large amounts of oxygen. Clients
with asthma and emphysema tend not to have chronic cough or peripheral edema.
40. (A) The patient is under local anesthesia during the procedure
  : Before the procedure, the patient is administered with drugs that would help to prevent infection and
rejection of the transplanted cells such as antibiotics, cytotoxic, and corticosteroids. During the transplant, the
patient is placed under general anesthesia.
41. : (D) Raise the side rails
  : A patient who is disoriented is at risk of falling out of bed. The initial action of the nurse should be
raising the side rails to ensure patients safety.
42. (A) Crowd red blood cells
  : The excessive production of white blood cells crowd out red blood cells production which causes anemia
to occur.
43. : (B) Leukocytosis
  Chronic Lymphocytic leukemia (CLL) is characterized by increased production of leukocytes and
lymphocytes resulting in leukocytosis, and proliferation of these cells within the bone marrow, spleen and liver.
44. (A) Explain the risks of not having the surgery
  The best initial response is to explain the risks of not having the surgery. If the client understands the
risks but still refuses the nurse should notify the physician and the nurse supervisor and then record the client¶s
refusal in the nurses¶ notes.
45. (D) The 75-year-old client who was admitted 1 hour ago with new-onset atrial fibrillation and is
receiving L.V. dilitiazem (Cardizem)
  : The client with atrial fibrillation has the greatest potential to become unstable and is on L.V. medication
that requires close monitoring. After assessing this client, the nurse should assess the client with thrombophlebitis
who is receiving a heparin infusion, and then the 5V- year-old client admitted 2 days ago with heart failure (his signs
and symptoms are resolving and don¶t require immediate attention). The lowest priority is the V9-year-old with end-
stage right-sided heart failure, who requires time-consuming supportive measures.
46. (C) Cocaine
  Because of the client¶s age and negative medical history, the nurse should question her about cocaine
use. Cocaine increases myocardial oxygen consumption and can cause coronary artery spasm, leading to
tachycardia, ventricular fibrillation, myocardial ischemia, and myocardial infarction. Barbiturate overdose may
trigger respiratory depression and slow pulse. Opioids can cause marked respiratory depression, while
benzodiazepines can cause drowsiness and confusion.
47. : )+*Nonmobile mass with irregular edges
  Breast cancer tumors are fixed, hard, and poorly delineated with irregular edges. A mobile mass that is
soft and easily delineated is most often a fluid-filled benign cyst. Axillary lymph nodes may or may not be palpable
on initial detection of a cancerous mass. Nipple retraction ² not eversion ² may be a sign of cancer.
4V. : (C) Radiation
  The usual treatment for vaginal cancer is external or intravaginal radiation therapy. Less often, surgery is
performed. Chemotherapy typically is prescribed only if vaginal cancer is diagnosed in an early stage, which is rare.
Immunotherapy isn't used to treat vaginal cancer.
49. (B) Carcinoma in situ, no abnormal regional lymph nodes, and no evidence of distant metastasis
  TIS, N0, M0 denotes carcinoma in situ, no abnormal regional lymph nodes, and no evidence of distant
metastasis. No evidence of primary tumor, no abnormal regional lymph nodes, and no evidence of distant metastasis
is classified as T0, N0, M0. If the tumor and regional lymph nodes can't be assessed and no evidence of metastasis
exists, the lesion is classified as TX, NX, M0. A progressive increase in tumor size, no demonstrable metastasis of
the regional lymph nodes, and ascending degrees of distant metastasis is classified as T1, T2, T3, or T4; N0; and
M1, M2, or M3.
50. (D) "Keep the stoma moist."
  The nurse should instruct the client to keep the stoma moist, such as by applying a thin layer of
petroleum jelly around the edges, because a dry stoma may become irritated. The nurse should recommend placing a
stoma bib over the stoma to filter and warm air before it enters the stoma. The client should begin performing stoma
care without assistance as soon as possible to gain independence in self-care activities.
51. (B) Lung cancer
  Lung cancer is the most deadly type of cancer in both women and men. Breast cancer ranks second in
women, followed (in descending order) by colon and rectal cancer, pancreatic cancer, ovarian cancer, uterine cancer,
lymphoma, leukemia, liver cancer, brain cancer, stomach cancer, and multiple myeloma.
52. (A) miosis, partial eyelid ptosis, and anhidrosis on the affected side of the face.
  Horner's syndrome, which occurs when a lung tumor invades the ribs and affects the sympathetic nerve
ganglia, is characterized by miosis, partial eyelid ptosis, and anhidrosis on the affected side of the face. Chest pain,
dyspnea, cough, weight loss, and fever are associated with pleural tumors. Arm and shoulder pain and atrophy of the
arm and hand muscles on the affected side suggest Pancoast's tumor, a lung tumor involving the first thoracic and
eighth cervical nerves within the brachial plexus. Hoarseness in a client with lung cancer suggests that the tumor has
extended to the recurrent laryngeal nerve; dysphagia suggests that the lung tumor is compressing the esophagus.
53. (A) prostate-specific antigen, which is used to screen for prostate cancer.
  PSA stands for prostate-specific antigen, which is used to screen for prostate cancer. The other answers
are incorrect.
54. (D) "Remain supine for the time specified by the physician."
  The nurse should instruct the client to remain supine for the time specified by the physician. Local
anesthetics used in a subarachnoid block don't alter the gag reflex. No interactions between local anesthetics and
food occur. Local anesthetics don't cause hematuria.
55. (C) Sigmoidoscopy
  Used to visualize the lower GI tract, sigmoidoscopy and proctoscopy aid in the detection of two-thirds of
all colorectal cancers. Stool Hematest detects blood, which is a sign of colorectal cancer; however, the test doesn't
confirm the diagnosis. CEA may be elevated in colorectal cancer but isn't considered a confirming test. An
abdominal CT scan is used to stage the presence of colorectal cancer.
56. : (B) A fixed nodular mass with dimpling of the overlying skin
  A fixed nodular mass with dimpling of the overlying skin is common during late stages of breast cancer.
Many women have slightly asymmetrical breasts. Bloody nipple discharge is a sign of intraductal papilloma, a
benign condition. Multiple firm, round, freely movable masses that change with the menstrual cycle indicate
fibrocystic breasts, a benign condition.
57. : (A) Liver
  The liver is one of the five most common cancer metastasis sites. The others are the lymph nodes, lung, bone, and
brain. The colon, reproductive tract, and WBCs are occasional metastasis sites.
5V. (D) The client wears a watch and wedding band.
  During an MRI, the client should wear no metal objects, such as jewelry, because the strong magnetic field can pull
on them, causing injury to the client and (if they fly off) to others. The client must lie still during the MRI but can talk to those
performing the test by way of the microphone inside the scanner tunnel. The client should hear thumping sounds, which are
caused by the sound waves thumping on the magnetic field.
59. : (C) The recommended daily allowance of calcium may be found in a wide variety of foods.
  Premenopausal women require 1,000 mg of calcium per day. Postmenopausal women require 1,500 mg per day. It's
often, though not always, possible to get the recommended daily requirement in the foods we eat. Supplements are available but
not always necessary. Osteoporosis doesn't show up on ordinary X-rays until 30% of the bone loss has occurred. Bone
densitometry can detect bone loss of 3% or less. This test is sometimes recommended routinely for women over 35 who are at
risk. Strenuous exercise won't cause fractures.
60. (C) Joint flexion of less than 50%
  Arthroscopy is contraindicated in clients with joint flexion of less than 50% because of technical
problems in inserting the instrument into the joint to see it clearly. Other contraindications for this procedure include
skin and wound infections. Joint pain may be an indication, not a contraindication, for arthroscopy. Joint deformity
and joint stiffness aren't contraindications for this procedure.
61. (D) Gouty arthritis
  Gouty arthritis, a metabolic disease, is characterized by urate deposits and pain in the joints, especially
those in the feet and legs. Urate deposits don't occur in septic or traumatic arthritis. Septic arthritis results from
bacterial invasion of a joint and leads to inflammation of the synovial lining. Traumatic arthritis results from blunt
trauma to a joint or ligament. Intermittent arthritis is a rare, benign condition marked by regular, recurrent joint
effusions, especially in the knees.
62. (B) 30 ml/hou
  An infusion prepared with 25,000 units of heparin in 500 ml of saline solution yields 50 units of heparin
per milliliter of solution. The equation is set up as 50 units times X (the unknown quantity) equals 1,500 units/hour,
X equals 30 ml/hour.
63. (B) Loss of muscle contraction decreasing venous return
  : In clients with hemiplegia or hemiparesis loss of muscle contraction decreases venous return and may
cause swelling of the affected extremity. Contractures, or bony calcifications may occur with a stroke, but don¶t
appear with swelling. DVT may develop in clients with a stroke but is more likely to occur in the lower extremities.
A stroke isn¶t linked to protein loss.
64. (B) It appears on the distal interphalangeal joint
  Heberden¶s nodes appear on the distal interphalageal joint on both men and women. Bouchard¶s node
appears on the dorsolateral aspect of the proximal interphalangeal joint.
65. (B) Osteoarthritis is a localized disease rheumatoid arthritis is systemic
  Osteoarthritis is a localized disease, rheumatoid arthritis is systemic. Osteoarthritis isn¶t gender-specific,
but rheumatoid arthritis is. Clients have dislocations and subluxations in both disorders.
66. : (C) The cane should be used on the unaffected side
  A cane should be used on the unaffected side. A client with osteoarthritis should be encouraged to
ambulate with a cane, walker, or other assistive device as needed; their use takes weight and stress off joints.
67. (A) a. 9 U regular insulin and 21 U neutral protamine Hagedorn (NPH).
  A 70/30 insulin preparation is 70% NPH and 30% regular insulin. Therefore, a correct substitution
requires mixing 21 U of NPH and 9 U of regular insulin. The other choices are incorrect dosages for the prescribed
insulin.
6V. (C) colchicines
  A disease characterized by joint inflammation (especially in the great toe), gout is caused by urate
crystal deposits in the joints. The physician prescribes colchicine to reduce these deposits and thus ease joint
inflammation. Although aspirin is used to reduce joint inflammation and pain in clients with osteoarthritis and
rheumatoid arthritis, it isn't indicated for gout because it has no effect on urate crystal formation. Furosemide, a
diuretic, doesn't relieve gout. Calcium gluconate is used to reverse a negative calcium balance and relieve muscle
cramps, not to treat gout.
69. : (A) Adrenal cortex
  Excessive secretion of aldosterone in the adrenal cortex is responsible for the client's hypertension. This
hormone acts on the renal tubule, where it promotes reabsorption of sodium and excretion of potassium and
hydrogen ions. The pancreas mainly secretes hormones involved in fuel metabolism. The adrenal medulla secretes
the catecholamines ² epinephrine and norepinephrine. The parathyroids secrete parathyroid hormone.
70. : (C) They debride the wound and promote healing by secondary intention
  For this client, wet-to-dry dressings are most appropriate because they clean the foot ulcer by debriding
exudate and necrotic tissue, thus promoting healing by secondary intention. Moist, transparent dressings contain
exudate and provide a moist wound environment. Hydrocolloid dressings prevent the entrance of microorganisms
and minimize wound discomfort. Dry sterile dressings protect the wound from mechanical trauma and promote
healing.
71. (A) Hyperkalemia
  In adrenal insufficiency, the client has hyperkalemia due to reduced aldosterone secretion. BUN
increases as the glomerular filtration rate is reduced. Hyponatremia is caused by reduced aldosterone secretion.
Reduced cortisol secretion leads to impaired glyconeogenesis and a reduction of glycogen in the liver and muscle,
causing hypoglycemia.
72. (C) Restricting fluids
  To reduce water retention in a client with the SIADH, the nurse should restrict fluids. Administering
fluids by any route would further increase the client's already heightened fluid load.
73. (D) glycosylated hemoglobin level.
  Because some of the glucose in the bloodstream attaches to some of the hemoglobin and stays attached
during the 120-day life span of red blood cells, glycosylated hemoglobin levels provide information about blood
glucose levels during the previous 3 months. Fasting blood glucose and urine glucose levels only give information
about glucose levels at the point in time when they were obtained. Serum fructosamine levels provide information
about blood glucose control over the past 2 to 3 weeks.
74. (C) 4:00 pm
  NPH is an intermediate-acting insulin that peaks V to 12 hours after administration. Because the nurse
administered NPH insulin at 7 a.m., the client is at greatest risk for hypoglycemia from 3 p.m. to 7 p.m.
75. : (A) Glucocorticoids and androgens
  The adrenal glands have two divisions, the cortex and medulla. The cortex produces three types of
hormones: glucocorticoids, mineralocorticoids, and androgens. The medulla produces catecholamines ²
epinephrine and norepinephrine.
76. (A) Hypocalcemia
  Hypocalcemia may follow thyroid surgery if the parathyroid glands were removed accidentally. Signs
and symptoms of hypocalcemia may be delayed for up to 7 days after surgery. Thyroid surgery doesn't directly
cause serum sodium, potassium, or magnesium abnormalities. Hyponatremia may occur if the client inadvertently
received too much fluid; however, this can happen to any surgical client receiving I.V. fluid therapy, not just one
recovering from thyroid surgery. Hyperkalemia and hypermagnesemia usually are associated with reduced renal
excretion of potassium and magnesium, not thyroid surgery.
77. : (D) Carcinoembryonic antigen level
  In clients who smoke, the level of carcinoembryonic antigen is elevated. Therefore, it can't be used as a
general indicator of cancer. However, it is helpful in monitoring cancer treatment because the level usually falls to
normal within 1 month if treatment is successful. An elevated acid phosphatase level may indicate prostate cancer.
An elevated alkaline phosphatase level may reflect bone metastasis. An elevated serum calcitonin level usually
signals thyroid cancer.
7V. : (B) Dyspnea, tachycardia, and pallor
  Signs of iron-deficiency anemia include dyspnea, tachycardia, and pallor as well as fatigue, listlessness,
irritability, and headache. Night sweats, weight loss, and diarrhea may signal acquired immunodeficiency syndrome
(AIDS). Nausea, vomiting, and anorexia may be signs of hepatitis B. Itching, rash, and jaundice may result from an
allergic or hemolytic reaction.
79. (D) "I'll need to have a C-section if I become pregnant and have a baby."
  The human immunodeficiency virus (HIV) is transmitted from mother to child via the transplacental
route, but a Cesarean section delivery isn't necessary when the mother is HIV-positive. The use of birth control will
prevent the conception of a child who might have HIV. It's true that a mother who's HIV positive can give birth to a
baby who's HIV negative.
V0. (C) "Avoid sharing such articles as toothbrushes and razors."
  The human immunodeficiency virus (HIV), which causes AIDS, is most concentrated in the blood. For
this reason, the client shouldn't share personal articles that may be blood-contaminated, such as toothbrushes and
razors, with other family members. HIV isn't transmitted by bathing or by eating from plates, utensils, or serving
dishes used by a person with AIDS.
V1. (B) Pallor, tachycardia, and a sore tongue
  Pallor, tachycardia, and a sore tongue are all characteristic findings in pernicious anemia. Other clinical
manifestations include anorexia; weight loss; a smooth, beefy red tongue; a wide pulse pressure; palpitations;
angina; weakness; fatigue; and paresthesia of the hands and feet. Bradycardia, reduced pulse pressure, weight gain,
and double vision aren't characteristic findings in pernicious anemia.
V2. (B) Administer epinephrine, as prescribed, and prepare to intubate the client if necessary.
  To reverse anaphylactic shock, the nurse first should administer epinephrine, a potent bronchodilator as
prescribed. The physician is likely to order additional medications, such as antihistamines and corticosteroids; if
these medications don't relieve the respiratory compromise associated with anaphylaxis, the nurse should prepare to
intubate the client. No antidote for penicillin exists; however, the nurse should continue to monitor the client's vital
signs. A client who remains hypotensive may need fluid resuscitation and fluid intake and output monitoring;
however, administering epinephrine is the first priority.
V3. (D) bilateral hearing loss.
  Prolonged use of aspirin and other salicylates sometimes causes bilateral hearing loss of 30 to 40
decibels. Usually, this adverse effect resolves within 2 weeks after the therapy is discontinued. Aspirin doesn't lead
to weight gain or fine motor tremors. Large or toxic salicylate doses may cause respiratory alkalosis, not respiratory
acidosis.
V4. (D) Lymphocyte
  The lymphocyte provides adaptive immunity ² recognition of a foreign antigen and formation of
memory cells against the antigen. Adaptive immunity is mediated by B and T lymphocytes and can be acquired
actively or passively. The neutrophil is crucial to phagocytosis. The basophil plays an important role in the release
of inflammatory mediators. The monocyte functions in phagocytosis and monokine production.
V5. (A) moisture replacement.
  Sjogren's syndrome is an autoimmune disorder leading to progressive loss of lubrication of the skin, GI
tract, ears, nose, and vagina. Moisture replacement is the mainstay of therapy. Though malnutrition and electrolyte
imbalance may occur as a result of Sjogren's syndrome's effect on the GI tract, it isn't the predominant problem.
Arrhythmias aren't a problem associated with Sjogren's syndrome.
V6. : (C) stool for V  

  test.
  Immunosuppressed clients ² for example, clients receiving chemotherapy, ² are at risk for infection
with V 

   which causes "horse barn" smelling diarrhea. Successful treatment begins with an accurate
diagnosis, which includes a stool test. The ELISA test is diagnostic for human immunodeficiency virus (HIV) and
isn't indicated in this case. An electrolyte panel and hemogram may be useful in the overall evaluation of a client but
aren't diagnostic for specific causes of diarrhea. A flat plate of the abdomen may provide useful information about
bowel function but isn't indicated in the case of "horse barn" smelling diarrhea.
V7. (D) Western blot test with ELISA.
  HIV infection is detected by analyzing blood for antibodies to HIV, which form approximately 2 to 12
weeks after exposure to HIV and denote infection. The Western blot test ² electrophoresis of antibody proteins ²
is more than 9V% accurate in detecting HIV antibodies when used in conjunction with the ELISA. It isn't specific
when used alone. Erosette immunofluorescence is used to detect viruses in general; it doesn't confirm HIV infection.
Quantification of T-lymphocytes is a useful monitoring test but isn't diagnostic for HIV. The ELISA test detects
HIV antibody particles but may yield inaccurate results; a positive ELISA result must be confirmed by the Western
blot test.
VV. (C) Abnormally low hematocrit (HCT) and hemoglobin (Hb) levels
  Low preoperative HCT and Hb levels indicate the client may require a blood transfusion before surgery.
If the HCT and Hb levels decrease during surgery because of blood loss, the potential need for a transfusion
increases. Possible renal failure is indicated by elevated BUN or creatinine levels. Urine constituents aren't found in
the blood. Coagulation is determined by the presence of appropriate clotting factors, not electrolytes.
V9. (A) Platelet count, prothrombin time, and partial thromboplastin time
  The diagnosis of DIC is based on the results of laboratory studies of prothrombin time, platelet count,
thrombin time, partial thromboplastin time, and fibrinogen level as well as client history and other assessment
factors. Blood glucose levels, WBC count, calcium levels, and potassium levels aren't used to confirm a diagnosis of
DIC.
90. (D) Strawberries
  Common food allergens include berries, peanuts, Brazil nuts, cashews, shellfish, and eggs. Bread,
carrots, and oranges rarely cause allergic reactions.
91. (B) A client with cast on the right leg who states, ³I have a funny feeling in my right leg.´
  It may indicate neurovascular compromise, requires immediate assessment.
92. : (D) A 62-year-old who had an abdominal-perineal resection three days ago; client complaints of chills.
  : The client is at risk for peritonitis; should be assessed for further symptoms and infection.
93. : (C) The client spontaneously flexes his wrist when the blood pressure is obtained.
  Carpal spasms indicate hypocalcemia.
94. (D) Use comfort measures and pillows to position the client.
  Using comfort measures and pillows to position the client is a non-pharmacological methods of pain
relief.
95. (B) Warm the dialysate solution.
  Cold dialysate increases discomfort. The solution should be warmed to body temperature in warmer or
heating pad; don¶t use microwave oven.
96. (C) The client holds the cane with his left hand, moves the cane forward followed by the right leg, and
then moves the left leg.
  The cane acts as a support and aids in weight bearing for the weaker right leg.
97. (A) Ask the woman¶s family to provide personal items such as photos or mementos.
  Photos and mementos provide visual stimulation to reduce sensory deprivation.
9V. (B) The client lifts the walker, moves it forward 10 inches, and then takes several small steps forward.
  A walker needs to be picked up, placed down on all legs.
99. (C) Isolation from their families and familiar surroundings.
  Gradual loss of sight, hearing, and taste interferes with normal functioning.
100. (A) Encourage the client to perform pursed lip breathing.
  Purse lip breathing prevents the collapse of lung unit and helps client control rate and depth of breathing.

'
   (   c   
c      
1. (C) Hypertension
  Hypertension, along with fever, and tenderness over the grafted kidney, reflects acute rejection.
2. (A) Pain
  : Sharp, severe pain (renal colic) radiating toward the genitalia and thigh is caused by uretheral distention
and smooth muscle spasm; relief form pain is the priority.
3. (D) Decrease the size and vascularity of the thyroid gland.
  Lugol¶s solution provides iodine, which aids in decreasing the vascularity of the thyroid gland, which
limits the risk of hemorrhage when surgery is performed.
4. (A) Liver Disease
  The client with liver disease has a decreased ability to metabolize carbohydrates because of a decreased
ability to form glycogen (glycogenesis) and to form glucose from glycogen.
5. (C) Leukopenia
  : Leukopenia, a reduction in WBCs, is a systemic effect of chemotherapy as a result of myelosuppression.
6. (C) Avoid foods that in the past caused flatus.
  : Foods that bothered a person preoperatively will continue to do so after a colostomy.
7. (B) Keep the irrigating container less than 1V inches above the stoma.´
  This height permits the solution to flow slowly with little force so that excessive peristalsis is not
immediately precipitated.
V. (A) Administer Kayexalate
  Kayexalate,a potassium exchange resin, permits sodium to be exchanged for potassium in the intestine,
reducing the serum potassium level.
9. (B) 2V gtt/min
  This is the correct flow rate; multiply the amount to be infused (2000 ml) by the drop factor (10) and
divide the result by the amount of time in minutes (12 hours x 60 minutes)
10. (D) Upper trunk
  The percentage designated for each burned part of the body using the rule of nines: Head and neck 9%;
Right upper extremity 9%; Left upper extremity 9%; Anterior trunk 1V%; Posterior trunk 1V%; Right lower
extremity 1V%; Left lower extremity 1V%; Perineum 1%.
11. (C) Bleeding from ears
  The nurse needs to perform a thorough assessment that could indicate alterations in cerebral function,
increased intracranial pressures, fractures and bleeding. Bleeding from the ears occurs only with basal skull fractures
that can easily contribute to increased intracranial pressure and brain herniation.
12. (D) may engage in contact sports
  The client should be advised by the nurse to avoid contact sports. This will prevent trauma to the area of
the pacemaker generator.
13. : (A) Oxygen at 1-2L/min is given to maintain the hypoxic stimulus for breathing.
  COPD causes a chronic CO2 retention that renders the medulla insensitive to the CO2 stimulation for
breathing. The hypoxic state of the client then becomes the stimulus for breathing. Giving the client oxygen in low
concentrations will maintain the client¶s hypoxic drive.
14. (B) Facilitate ventilation of the left lung.
  Since only a partial pneumonectomy is done, there is a need to promote expansion of this remaining Left
lung by positioning the client on the opposite unoperated side.
15. (A) Food and fluids will be withheld for at least 2 hours.
  Prior to bronchoscopy, the doctors sprays the back of the throat with anesthetic to minimize the gag
reflex and thus facilitate the insertion of the bronchoscope. Giving the client food and drink after the procedure
without checking on the return of the gag reflex can cause the client to aspirate. The gag reflex usually returns after
two hours.
16. (C) hyperkalemia.
  Hyperkalemia is a common complication of acute renal failure. It's life-threatening if immediate action
isn't taken to reverse it. The administration of glucose and regular insulin, with sodium bicarbonate if necessary, can
temporarily prevent cardiac arrest by moving tassium into the cells and temporarily reducing serum potassium
levels. Hypernatremia, hypokalemia, and hypercalcemia don't usually occur with acute renal failure and aren't
treated with glucose, insulin, or sodium bicarbonate.
17. (A) This condition puts her at a higher risk for cervical cancer; therefore, she should have a
Papanicolaou (Pap) smear annually.
  Women with condylomata acuminata are at risk for cancer of the cervix and vulva. Yearly Pap smears
are very important for early detection. Because condylomata acuminata is a virus, there is no permanent cure.
Because condylomata acuminata can occur on the vulva, a condom won't protect sexual partners. HPV can be
transmitted to other parts of the body, such as the mouth, oropharynx, and larynx.
1V. (A) The left kidney usually is slightly higher than the right one.
  The left kidney usually is slightly higher than the right one. An adrenal gland lies atop each kidney. The
average kidney measures approximately 11 cm (4-3/V") long, 5 to 5.V cm (2" to 2.") wide, and 2.5 cm (1") thick. The
kidneys are located retroperitoneally, in the posterior aspect of the abdomen, on either side of the vertebral column.
They lie between the 12th thoracic and 3rd lumbar vertebrae.
19. (C) Blood urea nitrogen (BUN) 100 mg/dl and serum creatinine
6.5 mg/dl.
  The normal BUN level ranges V to 23 mg/dl; the normal serum creatinine level ranges from 0.7 to 1.5
mg/dl. The test results in option C are abnormally elevated, reflecting CRF and the kidneys' decreased ability to
remove nonprotein nitrogen waste from the blood. CRF causes decreased pH and increased hydrogen ions ² not
vice versa. CRF also increases serum levels of potassium, magnesium, and phosphorous, and decreases serum levels
of calcium. A uric acid analysis of 3.5 mg/dl falls within the normal range of 2.7 to 7.7 mg/dl; PSP excretion of 75%
also falls with the normal range of 60% to 75%.
20. (D) Alteration in the size, shape, and organization of differentiated cells
  '   refers to an alteration in the size, shape, and organization of differentiated cells. The presence
of completely undifferentiated tumor cells that don't resemble cells of the tissues of their origin is called  
An increase in the number of normal cells in a normal arrangement in a tissue or an organ is called    
Replacement of one type of fully differentiated cell by another in tissues where the second type normally isn't found
is called   
21. (D) Kaposi's sarcoma
  Kaposi's sarcoma is the most common cancer associated with AIDS. Squamous cell carcinoma, multiple
myeloma, and leukemia may occur in anyone and aren't associated specifically with AIDS.
22. (C) To prevent cerebrospinal fluid (CSF) leakage
  The client receiving a subarachnoid block requires special positioning to prevent CSF leakage and
headache and to ensure proper anesthetic distribution. Proper positioning doesn't help prevent confusion, seizures, or
cardiac arrhythmias.
23. (A) Auscultate bowel sounds.
  If abdominal distention is accompanied by nausea, the nurse must first auscultate bowel sounds. If bowel
sounds are absent, the nurse should suspect gastric or small intestine dilation and these findings must be reported to
the physician. Palpation should be avoided postoperatively with abdominal distention. If peristalsis is absent,
changing positions and inserting a rectal tube won't relieve the client's discomfort.
24. (B) Lying on the left side with knees bent
  For a colonoscopy, the nurse initially should position the client on the left side with knees bent. Placing
the client on the right side with legs straight, prone with the torso elevated, or bent over with hands touching the
floor wouldn't allow proper visualization of the large intestine.
25. (A) Blood supply to the stoma has been interrupted
  An ileostomy stoma forms as the ileum is brought through the abdominal wall to the surface skin,
creating an artificial opening for waste elimination. The stoma should appear cherry red, indicating adequate arterial
perfusion. A dusky stoma suggests decreased perfusion, which may result from interruption of the stoma's blood
supply and may lead to tissue damage or necrosis. A dusky stoma isn't a normal finding. Adjusting the ostomy bag
wouldn't affect stoma color, which depends on blood supply to the area. An intestinal obstruction also wouldn't
change stoma color.
26. (A) Applying knee splints
  Applying knee splints prevents leg contractures by holding the joints in a position of function. Elevating
the foot of the bed can't prevent contractures because this action doesn't hold the joints in a position of function.
Hyperextending a body part for an extended time is inappropriate because it can cause contractures. Performing
shoulder range-of-motion exercises can prevent contractures in the shoulders, but not in the legs.
27. (B) Urine output of 20 ml/hour.
  A urine output of less than 40 ml/hour in a client with burns indicates a fluid volume deficit. This client's
PaO2 value falls within the normal range (V0 to 100 mm Hg). White pulmonary secretions also are normal. The
client's rectal temperature isn't significantly elevated and probably results from the fluid volume deficit.
2V. (A) Turn him frequently.
  The most important intervention to prevent pressure ulcers is frequent position changes, which relieve
pressure on the skin and underlying tissues. If pressure isn't relieved, capillaries become occluded, reducing
circulation and oxygenation of the tissues and resulting in cell death and ulcer formation. During passive ROM
exercises, the nurse moves each joint through its range of movement, which improves joint mobility and circulation
to the affected area but doesn't prevent pressure ulcers. Adequate hydration is necessary to maintain healthy skin and
ensure tissue repair. A footboard prevents plantar flexion and footdrop by maintaining the foot in a dorsiflexed
position.
29. (C) In long, even, outward, and downward strokes in the direction of hair growth
  When applying a topical agent, the nurse should begin at the midline and use long, even, outward, and
downward strokes in the direction of hair growth. This application pattern reduces the risk of follicle irritation and
skin inflammation.
30. : (A) Beta -adrenergic blockers
  Beta-adrenergic blockers work by blocking beta receptors in the myocardium, reducing the response to
catecholamines and sympathetic nerve stimulation. They protect the myocardium, helping to reduce the risk of
another infraction by decreasing myocardial oxygen demand. Calcium channel blockers reduce the workload of the
heart by decreasing the heart rate. Narcotics reduce myocardial oxygen demand, promote vasodilation, and decrease
anxiety. Nitrates reduce myocardial oxygen consumption bt decreasing left ventricular end diastolic pressure
(preload) and systemic vascular resistance (afterload).
31. : (C) Raised 30 degrees
  Jugular venous pressure is measured with a centimeter ruler to obtain the vertical distance between the
sternal angle and the point of highest pulsation with the head of the bed inclined between 15 to 30 degrees. Increased
pressure can¶t be seen when the client is supine or when the head of the bed is raised 10 degrees because the point
that marks the pressure level is above the jaw (therefore, not visible). In high Fowler¶s position, the veins would be
barely discernible above the clavicle.
32. (D) Inotropic agents
  Inotropic agents are administered to increase the force of the heart¶s contractions, thereby increasing
ventricular contractility and ultimately increasing cardiac output. Beta-adrenergic blockers and calcium channel
blockers decrease the heart rate and ultimately decreased the workload of the heart. Diuretics are administered to
decrease the overall vascular volume, also decreasing the workload of the heart.
33. (B) Less than 30% of calories form fat
  : A client with low serum HDL and high serum LDL levels should get less than 30% of daily calories
from fat. The other modifications are appropriate for this client.
34. : (C) The emergency department nurse calls up the latest electrocardiogram results to check the client¶s
progress
  The emergency department nurse is no longer directly involved with the client¶s care and thus has no
legal right to information about his present condition. Anyone directly involved in his care (such as the telemetry
nurse and the on-call physician) has the right to information about his condition. Because the client requested that
the nurse update his wife on his condition, doing so doesn¶t breach confidentiality.
35. : (B) Check endotracheal tube placement.
  ET tube placement should be confirmed as soon as the client arrives in the emergency department. Once
the airways is secured, oxygenation and ventilation should be confirmed using an end-tidal carbon dioxide monitor
and pulse oximetry. Next, the nurse should make sure L.V. access is established. If the client experiences
symptomatic bradycardia, atropine is administered as ordered 0.5 to 1 mg every 3 to 5 minutes to a total of 3 mg.
Then the nurse should try to find the cause of the client¶s arrest by obtaining an ABG sample. Amiodarone is
indicated for ventricular tachycardia, ventricular fibrillation and atrial flutter ± not symptomatic bradycardia.
36. : (C) 95 mm Hg
  Use the following formula to calculate MAP
MAP = systolic + 2 (diastolic)
3
MAP=126 mm Hg + 2 (V0 mm Hg)
3
MAP=2V6 mm HG
3
MAP=95 mm Hg
37. : (C) Electrocardiogram, complete blood count, testing for occult blood, comprehensive serum metabolic
panel.
  An electrocardiogram evaluates the complaints of chest pain, laboratory tests determines anemia, and the
stool test for occult blood determines blood in the stool. Cardiac monitoring, oxygen, and creatine kinase and lactate
dehydrogenase levels are appropriate for a cardiac primary problem. A basic metabolic panel and alkaline
phosphatase and aspartate aminotransferase levels assess liver function. Prothrombin time, partial thromboplastin
time, fibrinogen and fibrin split products are measured to verify bleeding dyscrasias, An electroencephalogram
evaluates brain electrical activity.
3V. : (D) Heparin-associated thrombosis and thrombocytopenia (HATT)
  HATT may occur after CABG surgery due to heparin use during surgery. Although DIC and ITP cause
platelet aggregation and bleeding, neither is common in a client after revascularization surgery. Pancytopenia is a
reduction in all blood cells.
39. : (B) Corticosteroids
  : Corticosteroid therapy can decrease antibody production and phagocytosis of the antibody-coated
platelets, retaining more functioning platelets. Methotrexate can cause thrombocytopenia. Vitamin K is used to treat
an excessive anticoagulate state from warfarin overload, and ASA decreases platelet aggregation.
40. : (D) Xenogeneic
  An xenogeneic transplant is between is between human and another species. A syngeneic transplant is
between identical twins, allogeneic transplant is between two humans, and autologous is a transplant from the same
individual.
41. : (B)
  Tissue thromboplastin is released when damaged tissue comes in contact with clotting factors. Calcium
is released to assist the conversion of factors X to Xa. Conversion of factors XII to XIIa and VIII to VIIIa are part of
the  pathway.
42. : (C) Essential thrombocytopenia
  Essential thrombocytopenia is linked to immunologic disorders, such as SLE and human
immunodeficiency vitus. The disorder known as von Willebrand¶s disease is a type of hemophilia and isn¶t linked to
SLE. Moderate to severe anemia is associated with SLE, not polycythermia. Dressler¶s syndrome is pericarditis that
occurs after a myocardial infarction and isn¶t linked to SLE.
43. : (B) Night sweat
  In stage 1, symptoms include a single enlarged lymph node (usually), unexplained fever, night sweats,
malaise, and generalized pruritis. Although splenomegaly may be present in some clients, night sweats are generally
more prevalent. Pericarditis isn¶t associated with Hodgkin¶s disease, nor is hypothermia. Moreover, splenomegaly
and pericarditis aren¶t symptoms. Persistent hypothermia is associated with Hodgkin¶s but isn¶t an early sign of the
disease.
44. : (D) Breath sounds
  Pneumonia, both viral and fungal, is a common cause of death in clients with neutropenia, so frequent
assessment of respiratory rate and breath sounds is required. Although assessing blood pressure, bowel sounds, and
heart sounds is important, it won¶t help detect pneumonia.
45. : (B) Muscle spasm
  Back pain or paresthesia in the lower extremities may indicate impending spinal cord compression from
a spinal tumor. This should be recognized and treated promptly as progression of the tumor may result in paraplegia.
The other options, which reflect parts of the nervous system, aren¶t usually affected by MM.
46. : (C)10 years
  Epidermiologic studies show the average time from initial contact with HIV to the development of AIDS
is 10 years.
47. : (A) Low platelet count
  In DIC, platelets and clotting factors are consumed, resulting in microthrombi and excessive bleeding.
As clots form, fibrinogen levels decrease and the prothrombin time increases. Fibrin degeneration products increase
as fibrinolysis takes places.
4V. : (D) Hodgkin¶s disease
  Hodgkin¶s disease typically causes fever night sweats, weight loss, and lymph mode enlargement.
Influenza doesn¶t last for months. Clients with sickle cell anemia manifest signs and symptoms of chronic anemia
with pallor of the mucous membrane, fatigue, and decreased tolerance for exercise; they don¶t show fever, night
sweats, weight loss or lymph node enlargement. Leukemia doesn¶t cause lymph node enlargement.
49. : (C) A Rh-negative
  Human blood can sometimes contain an inherited D antigen. Persons with the D antigen have Rh-
positive blood type; those lacking the antigen have Rh-negative blood. It¶s important that a person with Rhnegative
blood receives Rh-negative blood. If Rh-positive blood is administered to an Rh-negative person, the recipient
develops anti-Rh agglutinins, and sub sequent transfusions with Rh-positive blood may cause serious reactions with
clumping and hemolysis of red blood cells.
50. (B) ³I will call my doctor if Stacy has persistent vomiting and diarrhea´.
  Persistent (more than 24 hours) vomiting, anorexia, and diarrhea are signs of toxicity and the patient
should stop the medication and notify the health care provider. The other manifestations are expected side effects of
chemotherapy.
51. (D) ³This is only temporary; Stacy will re-grow new hair in 3-6 months, but may be different in
texture´.
  This is the appropriate response. The nurse should help the mother how to cope with her own feelings
regarding the child¶s disease so as not to affect the child negatively. When the hair grows back, it is still of the same
color and texture.
52. (B) Apply viscous Lidocaine to oral ulcers as needed.
  Stomatitis can cause pain and this can be relieved by applying topical anesthetics such as lidocaine
before mouth care. When the patient is already comfortable, the nurse can proceed with providing the patient with
oral rinses of saline solution mixed with equal part of water or hydrogen peroxide mixed water in 1:3 concentrations
to promote oral hygiene. Every 2-4 hours.
53. : (C) Immediately discontinue the infusion
  Edema or swelling at the IV site is a sign that the needle has been dislodged and the IV solution is
leaking into the tissues causing the edema. The patient feels pain as the nerves are irritated by pressure and the IV
solution. The first action of the nurse would be to discontinue the infusion right away to prevent further edema and
other complication.
54. : (C) Chronic obstructive bronchitis
  Clients with chronic obstructive bronchitis appear bloated; they have large barrel chest and peripheral
edema, cyanotic nail beds, and at times, circumoral cyanosis. Clients with ARDS are acutely short of breath and
frequently need intubation for mechanical ventilation and large amount of oxygen. Clients with asthma don¶t exhibit
characteristics of chronic disease, and clients with emphysema appear pink and cachectic.
55. : (D) Emphysema
  Because of the large amount of energy it takes to breathe, clients with emphysema are usually cachectic.
They¶re pink and usually breathe through pursed lips, hence the term ³puffer.´ Clients with ARDS are usually
acutely short of breath. Clients with asthma don¶t have any particular characteristics, and clients with chronic
obstructive bronchitis are bloated and cyanotic in appearance.
56. : D V0 mm Hg
  A client about to go into respiratory arrest will have inefficient ventilation and will be retaining carbon
dioxide. The value expected would be around V0 mm Hg. All other values are lower than expected.
57. : (C) Respiratory acidosis
  Because Paco2 is high at V0 mm Hg and the metabolic measure, HCO3- is normal, the client has
respiratory acidosis. The pH is less than 7.35, academic, which eliminates metabolic and respiratory alkalosis as
possibilities. If the HCO3- was below 22 mEq/L the client would have metabolic acidosis.
5V. : (C) Respiratory failure
  The client was reacting to the drug with respiratory signs of impending anaphylaxis, which could lead to
eventually respiratory failure. Although the signs are also related to an asthma attack or a pulmonary embolism,
consider the new drug first. Rheumatoid arthritis doesn¶t manifest these signs.
59. (D) Elevated serum aminotransferase
  Hepatic cell death causes release of liver enzymes alanine aminotransferase (ALT), aspartate
aminotransferase (AST) and lactate dehydrogenase (LDH) into the circulation. Liver cirrhosis is a chronic and
irreversible disease of the liver characterized by generalized inflammation and fibrosis of the liver tissues.
60. (A) Impaired clotting mechanism
  Cirrhosis of the liver results in decreased Vitamin K absorption and formation of clotting factors
resulting in impaired clotting mechanism.
61. (B) Altered level of consciousness
  Changes in behavior and level of consciousness are the first sins of hepatic encephalopathy. Hepatic
encephalopathy is caused by liver failure and develops when the liver is unable to convert protein metabolic product
ammonia to urea. This results in accumulation of ammonia and other toxic in the blood that damages the cells.
62. (C) ³I¶ll lower the dosage as ordered so the drug causes only 2 to 4 stools a day´.
  Lactulose is given to a patients with hepatic encephalopathy to reduce absorption of ammonia in the
intestines by binding with ammonia and promoting more frequent bowel movements. If the patient experience
diarrhea, it indicates over dosage and the nurse must reduce the amount of medication given to the patient. The stool
will be mashy or soft. Lactulose is also very sweet and may cause cramping and bloating.
63. (B) Severe lower back pain, decreased blood pressure, decreased RBC count, increased WBC count.
  Severe lower back pain indicates an aneurysm rupture, secondary to pressure being applied within the
abdominal cavity. When ruptured occurs, the pain is constant because it can¶t be alleviated until the aneurysm is
repaired. Blood pressure decreases due to the loss of blood. After the aneurysm ruptures, the vasculature is
interrupted and blood volume is lost, so blood pressure wouldn¶t increase. For the same reason, the RBC count is
decreased ± not increased. The WBC count increases as cell migrate to the site of injury.
64. (D) Apply gloves and assess the groin site
  Observing standard precautions is the first priority when dealing with any blood fluid. Assessment of the
groin site is the second priority. This establishes where the blood is coming from and determines how much blood
has been lost. The goal in this situation is to stop the bleeding. The nurse would call for help if it were warranted
after the assessment of the situation. After determining the extent of the bleeding, vital signs assessment is
important. The nurse should never move the client, in case a clot has formed. Moving can disturb the clot and cause
rebleeding.
65. (D) Percutaneous transluminal coronary angioplasty (PTCA)
  PTCA can alleviate the blockage and restore blood flow and oxygenation. An echocardiogram is a
noninvasive diagnosis test. Nitroglycerin is an oral sublingual medication. Cardiac catheterization is a diagnostic
tool ± not a treatment.
66. (B) Cardiogenic shock
  Cardiogenic shock is shock related to ineffective pumping of the heart. Anaphylactic shock results from
an allergic reaction. Distributive shock results from changes in the intravascular volume distribution and is usually
associated with increased cardiac output. MI isn¶t a shock state, though a severe MI can lead to shock.
67. )C) Kidneys¶ excretion of sodium and water
  The kidneys respond to rise in blood pressure by excreting sodium and excess water. This response
ultimately affects sysmolic blood pressure by regulating blood volume. Sodium or water retention would only
further increase blood pressure. Sodium and water travel together across the membrane in the kidneys; one can¶t
travel without the other.
6V. : (D) It inhibits reabsorption of sodium and water in the loop of Henle.
  Furosemide is a loop diuretic that inhibits sodium and water reabsorption in the loop Henle, thereby
causing a decrease in blood pressure. Vasodilators cause dilation of peripheral blood vessels, directly relaxing
vascular smooth muscle and decreasing blood pressure. Adrenergic blockers decrease sympathetic
cardioacceleration and decrease blood pressure. Angiotensin-converting enzyme inhibitors decrease blood pressure
due to their action on angiotensin.
69. : (C) Pancytopenia, elevated antinuclear antibody (ANA) titer
  Laboratory findings for clients with SLE usually show pancytopenia, elevated ANA titer, and decreased
serum complement levels. Clients may have elevated BUN and creatinine levels from nephritis, but the increase
does  indicate SLE.
70. : (C) Narcotics are avoided after a head injury because they may hide a worsening condition.
  Narcotics may mask changes in the level of consciousness that indicate increased ICP and shouldn¶t
acetaminophen is strong enough ignores the mother¶s question and therefore isn¶t appropriate. Aspirin is
contraindicated in conditions that may have bleeding, such as trauma, and for children or young adults with viral
illnesses due to the danger of Reye¶s syndrome. Stronger medications may not necessarily lead to vomiting but will
sedate the client, thereby masking changes in his level of consciousness.
71. : (A) Appropriate; lowering carbon dioxide (CO2) reduces intracranial pressure (ICP)
  A normal Paco2 value is 35 to 45 mm Hg CO2 has vasodilating properties; therefore, lowering Paco2
through hyperventilation will lower ICP caused by dilated cerebral vessels. Oxygenation is evaluated through Pao2
and oxygen saturation. Alveolar hypoventilation would be reflected in an increased Paco2.
72. : (B) A 33-year-old client with a recent diagnosis of Guillain-Barre syndrome
  Guillain-Barre syndrome is characterized by ascending paralysis and potential respiratory failure. The
order of client assessment should follow client priorities, with disorder of airways, breathing, and then circulation.
There¶s no information to suggest the postmyocardial infarction client has an arrhythmia or other complication.
There¶s no evidence to suggest hemorrhage or perforation for the remaining clients as a priority of care.
73. : (C) Decreases inflammation
  Then action of colchicines is to decrease inflammation by reducing the migration of leukocytes to
synovial fluid. Colchicine doesn¶t replace estrogen, decrease infection, or decrease bone demineralization.
74. : (C) Osteoarthritis is the most common form of arthritis
  Osteoarthritis is the most common form of arthritis and can be extremely debilitating. It can afflict
people of any age, although most are elderly.
75. : (C) Myxedema coma
  Myxedema coma, severe hypothyroidism, is a life-threatening condition that may develop if thyroid
replacement medication isn't taken. Exophthalmos, protrusion of the eyeballs, is seen with hyperthyroidism. Thyroid
storm is life-threatening but is caused by severe hyperthyroidism. Tibial myxedema, peripheral mucinous edema
involving the lower leg, is associated with hypothyroidism but isn't life-threatening.
76. (B) An irregular apical pulse
  Because Cushing's syndrom0e causes aldosterone overproduction, which increases urinary potassium
loss, the disorder may lead to hypokalemia. Therefore, the nurse should immediately report signs and symptoms of
hypokalemia, such as an irregular apical pulse, to the physician. Edema is an expected finding because aldosterone
overproduction causes sodium and fluid retention. Dry mucous membranes and frequent urination signal
dehydration, which isn't associated with Cushing's syndrome.
77. (D) Below-normal urine osmolality level, above-normal serum osmolality level
  In diabetes insipidus, excessive polyuria causes dilute urine, resulting in a below-normal urine
osmolality level. At the same time, polyuria depletes the body of water, causing dehydration that leads to an above-
normal serum osmolality level. For the same reasons, diabetes insipidus doesn't cause above-normal urine
osmolality or below-normal serum osmolality levels.
7V. (A) "I can avoid getting sick by not becoming dehydrated and by paying attention to my need to
urinate, drink, or eat more than usual."
  Inadequate fluid intake during hyperglycemic episodes often leads to HHNS. By recognizing the signs of
hyperglycemia (polyuria, polydipsia, and polyphagia) and increasing fluid intake, the client may prevent HHNS.
Drinking a glass of nondiet soda would be appropriate for hypoglycemia. A client whose diabetes is controlled with
oral antidiabetic agents usually doesn't need to monitor blood glucose levels. A highcarbohydrate diet would
exacerbate the client's condition, particularly if fluid intake is low.
79. (D) Hyperparathyroidism
  Hyperparathyroidism is most common in older women and is characterized by bone pain and weakness
from excess parathyroid hormone (PTH). Clients also exhibit hypercaliuria-causing polyuria. While clients with
diabetes mellitus and diabetes insipidus also have polyuria, they don't have bone pain and increased sleeping.
Hypoparathyroidism is characterized by urinary frequency rather than polyuria.
V0. (C) "I'll take two-thirds of the dose when I wake up and one-third in the late afternoon."
  Hydrocortisone, a glucocorticoid, should be administered according to a schedule that closely reflects
the body's own secretion of this hormone; therefore, two-thirds of the dose of hydrocortisone should be taken in the
morning and one-third in the late afternoon. This dosage schedule reduces adverse effects.
V1. (C) High corticotropin and high cortisol levels
  A corticotropin-secreting pituitary tumor would cause high corticotropin and high cortisol levels. A high
corticotropin level with a low cortisol level and a low corticotropin level with a low cortisol level would be
associated with hypocortisolism. Low corticotropin and high cortisol levels would be seen if there was a primary
defect in the adrenal glands.
V2. (D) Performing capillary glucose testing every 4 hours
  The nurse should perform capillary glucose testing every 4 hours because excess cortisol may cause
insulin resistance, placing the client at risk for hyperglycemia. Urine ketone testing isn't indicated because the client
does secrete insulin and, therefore, isn't at risk for ketosis. Urine specific gravity isn't indicated because although
fluid balance can be compromised, it usually isn't dangerously imbalanced. Temperature regulation may be affected
by excess cortisol and isn't an accurate indicator of infection.
V3. (C) onset to be at 2:30 p.m. and its peak to be at 4 p.m.
  Regular insulin, which is a short-acting insulin, has an onset of 15 to 30 minutes and a peak of 2 to 4
hours. Because the nurse gave the insulin at 2 p.m., the expected onset would be from 2:15 p.m. to 2:30 p.m. and the
peak from 4 p.m. to 6 p.m.
V4. (A) No increase in the thyroid-stimulating hormone (TSH) level after 30 minutes during the TSH
stimulation test
  In the TSH test, failure of the TSH level to rise after 30 minutes confirms hyperthyroidism. A decreased
TSH level indicates a pituitary deficiency of this hormone. Below-normal levels of T3 and T4, as detected by
radioimmunoassay, signal hypothyroidism. A below-normal T4 level also occurs in malnutrition and liver disease
and may result from administration of phenytoin and certain other drugs.
V5. (B) "Rotate injection sites within the same anatomic region, not among different regions."
  The nurse should instruct the client to rotate injection sites within the same anatomic region. Rotating
sites among different regions may cause excessive day-to-day variations in the blood glucose level; also, insulin
absorption differs from one region to the next. Insulin should be injected only into healthy tissue lacking large blood
vessels, nerves, or scar tissue or other deviations. Injecting insulin into areas of hypertrophy may delay absorption.
The client shouldn't inject insulin into areas of lipodystrophy (such as hypertrophy or atrophy); to prevent
lipodystrophy, the client should rotate injection sites systematically. Exercise speeds drug absorption, so the client
shouldn't inject insulin into sites above muscles that will be exercised heavily.
V6. (D) Below-normal serum potassium level
  A client with HHNS has an overall body deficit of potassium resulting from diuresis, which occurs
secondary to the hyperosmolar, hyperglycemic state caused by the relative insulin deficiency. An elevated serum
acetone level and serum ketone bodies are characteristic of diabetic ketoacidosis. Metabolic acidosis, not serum
alkalosis, may occur in HHNS.
V7. (D) Maintaining room temperature in the low-normal range
  Graves' disease causes signs and symptoms of hypermetabolism, such as heat intolerance, diaphoresis,
excessive thirst and appetite, and weight loss. To reduce heat intolerance and diaphoresis, the nurse should keep the
client's room temperature in the low-normal range. To replace fluids lost via diaphoresis, the nurse should
encourage, not restrict, intake of oral fluids. Placing extra blankets on the bed of a client with heat intolerance would
cause discomfort. To provide needed energy and calories, the nurse should encourage the client to eat high-
carbohydrate foods.
VV. (A) Fracture of the distal radius
  Colles' fracture is a fracture of the distal radius, such as from a fall on an outstretched hand. It's most
common in women. Colles' fracture doesn't refer to a fracture of the olecranon, humerus, or carpal scaphoid.
V9. (B) Calcium and phosphorous
  In osteoporosis, bones lose calcium and phosphate salts, becoming porous, brittle, and abnormally
vulnerable to fracture. Sodium and potassium aren't involved in the development of osteoporosis.
90. (A) Adult respiratory distress syndrome (ARDS)
  Severe hypoxia after smoke inhalation is typically related to ARDS. The other conditions listed aren¶t
typically associated with smoke inhalation and severe hypoxia.
91. (D) Fat embolism
  Long bone fractures are correlated with fat emboli, which cause shortness of breath and hypoxia. It¶s
unlikely the client has developed asthma or bronchitis without a previous history. He could develop atelectasis but it
typically doesn¶t produce progressive hypoxia.
92. (D) Spontaneous pneumothorax
  A spontaneous pneumothorax occurs when the client¶s lung collapses, causing an acute decreased in the
amount of functional lung used in oxygenation. The sudden collapse was the cause of his chest pain and shortness of
breath. An asthma attack would show wheezing breath sounds, and bronchitis would have rhonchi. Pneumonia
would have bronchial breath sounds over the area of consolidation.
93. (C) Pneumothorax
  From the trauma the client experienced, it¶s unlikely he has bronchitis, pneumonia, or TB; rhonchi with
bronchitis, bronchial breath sounds with TB would be heard.
94. : (C) Serous fluids fills the space and consolidates the region
  Serous fluid fills the space and eventually consolidates, preventing extensive mediastinal shift of the
heart and remaining lung. Air can¶t be left in the space. There¶s no gel that can be placed in the pleural space. The
tissue from the other lung can¶t cross the mediastinum, although a temporary mediastinal shift exits until the space is
filled.
95. : (A) Alveolar damage in the infracted area
  The infracted area produces alveolar damage that can lead to the production of bloody sputum,
sometimes in massive amounts. Clot formation usually occurs in the legs. There¶s a loss of lung parenchyma and
subsequent scar tissue formation.
96. : (D) Respiratory alkalosis
  A client with massive pulmonary embolism will have a large region and blow off large amount of carbon
dioxide, which crosses the unaffected alveolar-capillary membrane more readily than does oxygen and results in
respiratory alkalosis.
97. : (A) Air leak
  Bubbling in the water seal chamber of a chest drainage system stems from an air leak. In pneumothorax
an air leak can occur as air is pulled from the pleural space. Bubbling doesn¶t normally occur with either adequate or
inadequate suction or any preexisting bubbling in the water seal chamber.
9V. : (B) 21
  3000 x 10 divided by 24 x 60.
99. (B) 2.4 ml
  .05 mg/ 1 ml = .12mg/ x ml, .05x = .12, x = 2.4 ml.
100. (D) ³I should put on the stockings before getting out of bed in the morning.
  Promote venous return by applying external pressure on veins.

'
   (   c   
c      
1. : (D) Focusing
  : The nurse is using focusing by suggesting that the client discuss a specific issue. The nurse didn¶t restate
the question, make observation, or ask further question (exploring).
2. : (D) Remove all other clients from the dayroom.
  : The nurse¶s first priority is to consider the safety of the clients in the therapeutic setting. The other
actions are appropriate responses after ensuring the safety of other clients.
3. : (A) The client is disruptive.
  : Group activity provides too much stimulation, which the client will not be able to handle (harmful to
self) and as a result will be disruptive to others.
4. : (C) Agree to talk with the mother and the father together.
  : By agreeing to talk with both parents, the nurse can provide emotional support and further assess and
validate the family¶s needs.
5. : (A) Perceptual disorders.
  : Frightening visual hallucinations are especially common in clients experiencing alcohol withdrawal.
6. : (D) Suggest that it takes awhile before seeing the results.
  : The client needs a specific response; that it takes 2 to 3 weeks (a delayed effect) until the therapeutic
blood level is reached.
7. : (C) Superego
  : This behavior shows a weak sense of moral consciousness. According to Freudian theory, personality
disorders stem from a weak superego.
V. : (C) Skeletal muscle paralysis.
  : Anectine is a depolarizing muscle relaxant causing paralysis. It is used to reduce the intensity of muscle
contractions during the convulsive stage, thereby reducing the risk of bone fractures or dislocation.
9. : (D) Increase calories, carbohydrates, and protein.
  : This client increased protein for tissue building and increased calories to replace what is burned up
(usually via carbohydrates).
10. : (C) Acting overly solicitous toward the child.
  : This behavior is an example of reaction formation, a coping mechanism.
11. : (A) By designating times during which the client can focus on the behavior.
  : The nurse should designate times during which the client can focus on the compulsive behavior or
obsessive thoughts. The nurse should urge the client to reduce the frequency of the compulsive behavior gradually,
not rapidly. She shouldn't call attention to or try to prevent the behavior. Trying to prevent the behavior may cause
pain and terror in the client. The nurse should encourage the client to verbalize anxieties to help distract attention
from the compulsive behavior.
12. : (D) Exploring the meaning of the traumatic event with the client.
  : The client with PTSD needs encouragement to examine and understand the meaning of the traumatic
event and consequent losses. Otherwise, symptoms may worsen and the client may become depressed or engage in
self-destructive behavior such as substance abuse. The client must explore the meaning of the event and won't heal
without this, no matter how much time passes. Behavioral techniques, such as relaxation therapy, may help decrease
the client's anxiety and induce sleep. The physician may prescribe antianxiety agents or antidepressants cautiously to
avoid dependence; sleep medication is rarely appropriate. A special diet isn't indicated unless the client also has an
eating disorder or a nutritional problem.
13. : (C) "Your problem is real but there is no physical basis for it.
We'll work on what is going on in your life to find out why it's happened."
  : The nurse must be honest with the client by telling her that the paralysis has no physiologic cause while
also conveying empathy and acknowledging that her symptoms are real. The client will benefit from psychiatric
treatment, which will help her understand the underlying cause of her symptoms. After the psychological conflict is
resolved, her symptoms will disappear. Saying that it must be awful not to be able to move her legs wouldn't answer
the client's question; knowing that the cause is psychological wouldn't necessarily make her feel better. Telling her
that she has developed paralysis to avoid leaving her parents or that her personality caused her disorder wouldn't
help her understand and resolve the underlying conflict.
14. : (C) fluvoxamine (Luvox) and clomipramine (Anafranil)
  : The antidepressants fluvoxamine and clomipramine have been effective in the treatment of OCD.
Librium and Valium may be helpful in treating anxiety related to OCD but aren't drugs of choice to treat the illness.
The other medications mentioned aren't effective in the treatment of OCD.
15. : (A) A warning about the drugs delayed therapeutic effect, which is from 14 to 30 days.
  : The client should be informed that the drug's therapeutic effect might not be reached for 14 to 30 days.
The client must be instructed to continue taking the drug as directed. Blood level checks aren't necessary. NMS
hasn't been reported with this drug, but tachycardia is frequently reported.
16. : (B) Severe anxiety and fear.
  : Phobias cause severe anxiety (such as a panic attack) that is out of proportion to the threat of the feared
object or situation. Physical signs and symptoms of phobias include profuse sweating, poor moto control,
tachycardia, and elevated blood pressure. Insomnia, an inability to concentrate, and weight loss are common in
depression. Withdrawal and failure to distinguish reality from fantasy occur in schizophrenia.
17. : (A) Antidepressants
  : Tricyclic and monoamine oxidase (MAO) inhibitor antidepressants have been found to be effective in
treating clients with panic attacks. Why these drugs help control panic attacks isn't clearly understood.
Anticholinergic agents, which are smooth-muscle relaxants, relieve physical symptoms of anxiety but don't relieve
the anxiety itself. Antipsychotic drugs are inappropriate because clients who experience panic attacks aren't
psychotic. Mood stabilizers aren't indicated because panic attacks are rarely associated with mood changes.
1V. : (B) 3 to 5 days
  : Monoamine oxidase inhibitors, such as tranylcypromine, have an onset of action of approximately 3 to 5
days. A full clinical response may be delayed for 3 to 4 weeks. The therapeutic effects may continue for 1 to 2
weeks after discontinuation.
19. : (B) Providing emotional support and individual counseling.
  : Clients in the first stage of Alzheimer's disease are aware that something is happening to them and may
become overwhelmed and frightened. Therefore, nursing care typically focuses on providing emotional support and
individual counseling. The other options are appropriate during the second stage of Alzheimer's disease, when the
client needs continuous monitoring to prevent minor illnesses from progressing into major problems and when
maintaining adequate nutrition may become a challenge. During this stage, offering nourishing finger foods helps
clients to feed themselves and maintain adequate nutrition.
20. : (C) Emotional lability, euphoria, and impaired memory
  : Signs of antianxiety agent overdose include emotional lability, euphoria, and impaired memory.
Phencyclidine overdose can cause combativeness, sweating, and confusion. Amphetamine overdose can result in
agitation, hyperactivity, and grandiose ideation. Hallucinogen overdose can produce suspiciousness, dilated pupils,
and increased blood pressure.
21. : (D) A low tolerance for frustration
  : Clients with an antisocial personality disorder exhibit a low tolerance for frustration, emotional
immaturity, and a lack of impulse control. They commonly have a history of unemployment, miss work repeatedly,
and quit work without other plans for employment. They don't feel guilt about their behavior and commonly
perceive themselves as victims. They also display a lack of responsibility for the outcome of their actions. Because
of a lack of trust in others, clients with antisocial personality disorder commonly have difficulty developing stable,
close relationships.
22. : (C) Methadone
  : Methadone is used to detoxify opiate users because it binds with opioid receptors at many sites in the
central nervous system but doesn¶t have the same deterious effects as other opiates, such as cocaine, heroin, and
morphine. Barbiturates, amphetamines, and benzodiazepines are highly addictive and would require detoxification
treatment.
23. : (B) Hallucinations
  : Hallucinations are visual, auditory, gustatory, tactile, or olfactory perceptions that have no basis in
reality. Delusions are false beliefs, rather than perceptions, that the client accepts as real. Loose associations are
rapid shifts among unrelated ideas. Neologisms are bizarre words that have meaning only to the client.
24. : (C) Set up a strict eating plan for the client.
  : Establishing a consistent eating plan and monitoring the client¶s weight are very important in this
disorder. The family and friends should be included in the client¶s care. The client should be monitored during
meals-not given privacy. Exercise must be limited and supervised.
25. : (A) Highly important or famous.
  : A delusion of grandeur is a false belief that one is highly important or famous. A delusion of persecution
is a false belief that one is being persecuted. A delusion of reference is a false belief that one is connected to events
unrelated to oneself or a belief that one is responsible for the evil in the world.
26. : (D) Listening attentively with a neutral attitude and avoiding power struggles.
  : The nurse should listen to the client¶s requests, express willingness to seriously consider the request, and
respond later. The nurse should encourage the client to take short daytime naps because he expends so much energy.
The nurse shouldn¶t try to restrain the client when he feels the need to move around as long as his activity isn¶t
harmful. High calorie finger foods should be offered to supplement the client¶s diet, if he can¶t remain seated long
enough to eat a complete meal. The nurse shouldn¶t be forced to stay seated at the table to finid=sh a meal. The
nurse should set limits in a calm, clear, and self-confident tone of voice.
27. : (D) Denial
  : Denial is unconscious defense mechanism in which emotional conflict and anxiety is avoided by refusing
to acknowledge feelings, desires, impulses, or external facts that are consciously intolerable. Withdrawal is a
common response to stress, characterized by apathy. Logical thinking is the ability to think rationally and make
responsible decisions, which would lead the client admitting the problem and seeking help. Repression is
suppressing past events from the consciousness because of guilty association.
2V. : (B) Paranoid thoughts
  : Clients with schizotypal personality disorder experience excessive social anxiety that can lead to
paranoid thoughts. Aggressive behavior is uncommon, although these clients may experience agitation with anxiety.
Their behavior is emotionally cold with a flattened affect, regardless of the situation. These clients demonstrate a
reduced capacity for close or dependent relationships.
29. : (C) Identify anxiety-causing situations
  : Bulimic behavior is generally a maladaptive coping response to stress and underlying issues. The client
must identify anxiety-causing situations that stimulate the bulimic behavior and then learn new ways of coping with
the anxiety.
30. : (A) Tension and irritability
  : An amphetamine is a nervous system stimulant that is subject to abuse because of its ability to produce
wakefulness and euphoria. An overdose increases tension and irritability. Options B and C are incorrect because
amphetamines stimulate norepinephrine, which increase th heart rate and blood flow. Diarrhea is a common adverse
effect so option D in is incorrect.
31. : (B) ³No, I do not hear your voices, but I believe you can hear them´.
  : The nurse, demonstrating knowledge and understanding, accepts the client¶s perceptions even though
they are hallucinatory.
32. : (C) Confusion for a time after treatment
  : The electrical energy passing through the cerebral cortex during ECT results in a temporary state of
confusion after treatment.
33. : (D) Acceptance stage
  : Communication and intervention during this stage are mainly nonverbal, as when the client gestures to
hold the nurse¶s hand.
34. : (D) A higher level of anxiety continuing for more than 3 months.
  : This is not an expected outcome of a crisis because by definition a crisis would be resolved in 6 weeks.
35. : (B) Staying in the sun
  : Haldol causes photosensitivity. Severe sunburn can occur on exposure to the sun.
36. : (D) Moderate-level anxiety
  : A moderately anxious person can ignore peripheral events and focuses on central concerns.
37. : (C) Diverse interest
  : Before onset of depression, these clients usually have very narrow, limited interest.
3V. : (A) As their depression begins to improve
  : At this point the client may have enough energy to plan and execute an attempt.
39. : (D) Disturbance in recalling recent events related to cerebral hypoxia.
  : Cell damage seems to interfere with registering input stimuli, which affects the ability to register and
recall recent events; vascular dementia is related to multiple vascular lesions of the cerebral cortex and subcortical
structure.
40. : (D) Encouraging the client to have blood levels checked as ordered.
  : Blood levels must be checked monthly or bimonthly when the client is on maintenance therapy because
there is only a small range between therapeutic and toxic levels.
41. : (B) Fine hand tremors or slurred speech
  : These are common side effects of lithium carbonate.
42. : (D) Presence
  : The constant presence of a nurse provides emotional support because the client knows that someone is
attentive and available in case of an emergency.
43. : (A) Client¶s perception of the presenting problem.
  : The nurse can be most therapeutic by starting where the client is, because it is the client¶s concept of the
problem that serves as the starting point of the relationship.
44. : (B) Chocolate milk, aged cheese, and yogurt¶´
  : These high-tyramine foods, when ingested in the presence of an MAO inhibitor, cause a severe
hypertensive response.
45. : (B) 4 to 6 weeks
  : Crisis is self-limiting and lasts from 4 to 6 weeks.
46. : (D) Males are more likely to use lethal methods than are females
  : This finding is supported by research; females account for 90% of suicide attempts but males are three
times more successful because of methods used.
47. : (C) "Your cursing is interrupting the activity. Take time out in your room for 10 minutes."
  : The nurse should set limits on client behavior to ensure a comfortable environment for all clients. The
nurse should accept hostile or quarrelsome client outbursts within limits without becoming personally offended, as
in option A. Option B is incorrect because it implies that the client's actions reflect feelings toward the staff instead
of the client's own misery. Judgmental remarks, such as option D, may decrease the client's self-esteem.
4V. : (C) lithium carbonate (Lithane)
  : Lithium carbonate, an antimania drug, is used to treat clients with cyclical schizoaffective disorder, a
psychotic disorder once classified under schizophrenia that causes affective symptoms, including maniclike activity.
Lithium helps control the affective component of this disorder. Phenelzine is a monoamine oxidase inhibitor
prescribed for clients who don't respond to other antidepressant drugs such as imipramine. Chlordiazepoxide, an
antianxiety agent, generally is contraindicated in psychotic clients. Imipramine, primarily considered an
antidepressant agent, is also used to treat clients with agoraphobia and that undergoing cocaine detoxification.
49. : (B) Report a sore throat or fever to the physician immediately.
  : A sore throat and fever are indications of an infection caused by agranulocytosis, a potentially life-
threatening complication of clozapine. Because of the risk of agranulocytosis, white blood cell (WBC) counts are
necessary weekly, not monthly. If the WBC count drops below 3,000/ȝl, the medication must be stopped.
Hypotension may occur in clients taking this medication. Warn the client to stand up slowly to avoid dizziness from
orthostatic hypotension. The medication should be continued, even when symptoms have been controlled. If the
medication must be stopped, it should be slowly tapered over 1 to 2 weeks and only under the supervision of a
physician.
50. : (C) Neuroleptic malignant syndrome.
  : The client's signs and symptoms suggest neuroleptic malignant syndrome, a life-threatening reaction to
neuroleptic medication that requires immediate treatment. Tardive dyskinesia causes involuntary movements of the
tongue, mouth, facial muscles, and arm and leg muscles. Dystonia is characterized by cramps and rigidity of the
tongue, face, neck, and back muscles. Akathisia causes restlessness, anxiety, and jitteriness.
51. : (B) Advising the client to sit up for 1 minute before getting out of bed.
  : To minimize the effects of amitriptyline-induced orthostatic hypotension, the nurse should advise the
client to sit up for 1 minute before getting out of bed. Orthostatic hypotension commonly occurs with tricyclic
antidepressant therapy. In these cases, the dosage may be reduced or the physician may prescribe nortriptyline,
another tricyclic antidepressant. Orthostatic hypotension disappears only when the drug is discontinued.
52. : (D) Dysthymic disorder.
  : Dysthymic disorder is marked by feelings of depression lasting at least 2 years, accompanied by at least
two of the following symptoms: sleep disturbance, appetite disturbance, low energy or fatigue, low selfesteem, poor
concentration, difficulty making decisions, and hopelessness. These symptoms may be relatively continuous or
separated by intervening periods of normal mood that last a few days to a few weeks. Cyclothymic disorder is a
chronic mood disturbance of at least 2 years' duration marked by numerous periods of depression and hypomania.
Atypical affective disorder is characterized by manic signs and symptoms. Major depression is a recurring,
persistent sadness or loss of interest or pleasure in almost all activities, with signs and symptoms recurring for at
least 2 weeks.
53. : (C) 30 g mixed in 250 ml of water
  : The usual adult dosage of activated charcoal is 5 to 10 times the estimated weight of the drug or
chemical ingested, or a minimum dose of 30 g, mixed in 250 ml of water. Doses less than this will be ineffective;
doses greater than this can increase the risk of adverse reactions, although toxicity doesn't occur with activated
charcoal, even at the maximum dose.
54. : (C) St. John's wort
  : St. John's wort has been found to have serotonin-elevating properties, similar to prescription
antidepressants. Ginkgo biloba is prescribed to enhance mental acuity. Echinacea has immune-stimulating
properties. Ephedra is a naturally occurring stimulant that is similar to ephedrine.
55. : (B) Sodium
  : Lithium is chemically similar to sodium. If sodium levels are reduced, such as from sweating or diuresis, lithium
will be reabsorbed by the kidneys, increasing the risk of toxicity. Clients taking lithium shouldn't restrict their intake of sodium
and should drink adequate amounts of fluid each day. The other electrolytes are important for normal body functions but sodium
is most important to the absorption of lithium.
56. : (D) It's characterized by an acute onset and lasts hours to a number of days
  : Delirium has an acute onset and typically can last from several hours to several days.
57. : (B) Impaired communication.
  : Initially, memory impairment may be the only cognitive deficit in a client with Alzheimer's disease. During the early
stage of this disease, subtle personality changes may also be present. However, other than occasional irritable outbursts and lack
of spontaneity, the client is usually cooperative and exhibits socially appropriate behavior. Signs of advancement to the middle
stage of Alzheimer's disease include exacerbated cognitive impairment with obvious personality changes and impaired
communication, such as inappropriate conversation, actions, and responses. During the late stage, the client can't perform self-
care activities and may become mute.
5V. : (D) This medication may initially cause tiredness, which should become less bothersome over time.
  : Sedation is a common early adverse effect of imipramine, a tricyclic antidepressant, and usually
decreases as tolerance develops. Antidepressants aren't habit forming and don't cause physical or psychological
dependence. However, after a long course of high-dose therapy, the dosage should be decreased gradually to avoid
mild withdrawal symptoms. Serious adverse effects, although rare, include myocardial infarction, heart failure, and
tachycardia. Dietary restrictions, such as avoiding aged cheeses, yogurt, and chicken livers, are necessary for a client
taking a monoamine oxidase inhibitor, not a tricyclic antidepressant.
59. : (C) Monitor vital signs, serum electrolyte levels, and acid-base balance.
  : An anorexic client who requires hospitalization is in poor physical condition from starvation and may die
as a result of arrhythmias, hypothermia, malnutrition, infection, or cardiac abnormalities secondary to electrolyte
imbalances. Therefore, monitoring the client's vital signs, serum electrolyte level, and acid base balance is crucial.
Option A may worsen anxiety. Option B is incorrect because a weight obtained after breakfast is more accurate than
one obtained after the evening meal. Option D would reward the client with attention for not eating and reinforce the
control issues that are central to the underlying psychological problem; also, the client may record food and fluid
intake inaccurately.
60. : (D) Opioid withdrawal
  : The symptoms listed are specific to opioid withdrawal. Alcohol withdrawal would show elevated vital
signs. There is no real withdrawal from cannibis. Symptoms of cocaine withdrawal include depression, anxiety, and
agitation.
61. : (A) Regression
  : An adult who throws temper tantrums, such as this one, is displaying regressive behavior, or behavior
that is appropriate at a younger age. In projection, the client blames someone or something other than the source. In
reaction formation, the client acts in opposition to his feelings. In intellectualization, the client overuses rational
explanations or abstract thinking to decrease the significance of a feeling or event.
62. : (A) Abnormal movements and involuntary movements of the mouth, tongue, and face.
  : Tardive dyskinesia is a severe reaction associated with long term use of antipsychotic medication. The
clinical manifestations include abnormal movements (dyskinesia) and involuntary movements of the mouth, tongue
(fly catcher tongue), and face.
63. : (C) Blurred vision
  : At lithium levels of 2 to 2.5 mEq/L the client will experienced blurred vision, muscle twitching, severe
hypotension, and persistent nausea and vomiting. With levels between 1.5 and 2 mEq/L the client experiencing
vomiting, diarrhea, muscle weakness, ataxia, dizziness, slurred speech, and confusion. At lithium levels of 2.5 to 3
mEq/L or higher, urinary and fecal incontinence occurs, as well as seizures, cardiac dysrythmias, peripheral vascular
collapse, and death.
64. : (C) No acts of aggression have been observed within 1 hour after the release of two of the extremity
restraints.
  : The best indicator that the behavior is controlled, if the client exhibits no signs of aggression after partial
release of restraints. Options A, B, and D do not ensure that the client has controlled the behavior.
65. (A) increased attention span and concentration
  : The medication has a paradoxic effect that decrease hyperactivity and impulsivity among children with
ADHD. B, C, D. Side effects of Ritalin include anorexia, insomnia, diarrhea and irritability.
66. (C) Moderate
  : The child with moderate mental retardation has an I.Q. of 35- 50 Profound Mental retardation has an I.Q.
of below 20; Mild mental retardation 50-70 and Severe mental retardation has an I.Q. of 20-35.
67. (D) Rearrange the environment to activate the child
  : The child with autistic disorder does not want change. Maintaining a consistent environment is
therapeutic. A. Angry outburst can be re-channeling through safe activities. B. Acceptance enhances a trusting
relationship. C. Ensure safety from self-destructive behaviors like head banging and hair pulling.
6V. (B) cocaine
  : The manifestations indicate intoxication with cocaine, a CNS stimulant. A. Intoxication with heroine is
manifested by euphoria then impairment in judgment, attention and the presence of papillary constriction. C.
Intoxication with hallucinogen like LSD is manifested by grandiosity, hallucinations, synesthesia and increase in
vital signs D. Intoxication with Marijuana, a cannabinoid is manifested by sensation of slowed time, conjunctival
redness, social withdrawal, impaired judgment and hallucinations.
69. : (B) insidious onset
  : Dementia has a gradual onset and progressive deterioration. It causes pronounced memory and cognitive
disturbances. A,C and D are all characteristics of delirium.
70. (C) Claustrophobia
  : Claustrophobia is fear of closed space. A. Agoraphobia is fear of open space or being a situation where
escape is difficult. B. Social phobia is fear of performing in the presence of others in a way that will be humiliating
or embarrassing. D. Xenophobia is fear of strangers.
71. (A) Revealing personal information to the client
  : Counter-transference is an emotional reaction of the nurse on the client based on her unconscious needs
and conflicts. B and C. These are therapeutic approaches. D. This is transference reaction where a client has an
emotional reaction towards the nurse based on her past.
72. (D) Hold the next dose and obtain an order for a stat serum lithium level
  : Diarrhea and vomiting are manifestations of Lithium toxicity. The next dose of lithium should be
withheld and test is done to validate the observation. A. The manifestations are not due to drug interaction. B
Cogentin is used to manage the extra pyramidal symptom side effects of antipsychotics. C. The common side effects
of Lithium are fine hand tremors, nausea, polyuria and polydipsia.
73. (C) A living, learning or working environment.
  : A therapeutic milieu refers to a broad conceptual approach in which all aspects of the environment are
channeled to provide a therapeutic environment for the client. The six environmental elements include structure,
safety, norms; limit setting, balance and unit modification. A. Behavioral approach in psychiatric care is based on
the premise that behavior can be learned or unlearned through the use of reward and punishment. B. Cognitive
approach to change behavior is done by correcting distorted perceptions and irrational beliefs to correct maladaptive
behaviors. D. This is not congruent with therapeutic milieu.
74. (B) Transference
  : Transference is a positive or negative feeling associated with a significant person in the client¶s past that
are unconsciously assigned to another A. Splitting is a defense mechanism commonly seen in a client with
personality disorder in which the world is perceived as all good or all bad C. Countert-transference is a phenomenon
where the nurse shifts feelings assigned to someone in her past to the patient D. Resistance is the client¶s refusal to
submit himself to the care of the nurse
75. (B) Adventitious
  : Adventitious crisis is a crisis involving a traumatic event. It is not part of everyday life. A. Situational
crisis is from an external source that upset ones psychological equilibrium C and D. Are the same. They are
transitional or developmental periods in life
76. : (C) Major depression
  : The DSM-IV-TR classifies major depression as an Axis I disorder. Borderline personality disorder as an
Axis II; obesity and hypertension, Axis III.
77. : (B) Transference
  : Transference is the unconscious assignment of negative or positive feelings evoked by a significant
person in the client¶s past to another person. Intellectualization is a defense mechanism in which the client avoids
dealing with emotions by focusing on facts. Triangulation refers to conflicts involving three family members.
Splitting is a defense mechanism commonly seen in clients with personality disorder in which the world is perceived
as all good or all bad.
7V. : (B) Hypochondriasis
  : Complains of vague physical symptoms that have no apparent medical causes are characteristic of clients
with hypochondriasis. In many cases, the GI system is affected. Conversion disorders are characterized by one or
more neurologic symptoms. The client¶s symptoms don¶t suggest severe anxiety. A client experiencing sublimation
channels maladaptive feelings or impulses into socially acceptable behavior
79. : (C) Hypochondriasis
  : Hypochodriasis in this case is shown by the client¶s belief that she has a serious illness, although
pathologic causes have been eliminated. The disturbance usually lasts at lease 6 with identifiable life stressor such
as, in this case, course examinations. Conversion disorder s are characterized by one or more neurologic symptoms.
Depersonalization refers to persistent recurrent episodes of feeling detached from one¶s self or body. Somatoform
disorders generally have a chronic course with few remissions.
V0. : (A) Triazolam (Halcion)
  : Triazolam is one of a group of sedative hypnotic medication that can be used for a limited time because
of the risk of dependence. Paroxetine is a scrotonin-specific reutake inhibitor used for treatment of depression panic
disorder, and obsessive-compulsive disorder. Fluoxetine is a scrotonin-specific reuptake inhibitor used for
depressive disorders and obsessive-compulsive disorders. Risperidome is indicated for psychotic disorders.
V1. : (D) It promotes emotional support or attention for the client
  : Secondary gain refers to the benefits of the illness that allow the client to receive emotional support or
attention. Primary gain enables the client to avoid some unpleasant activity. A dysfunctional family may disregard
the real issue, although some conflict is relieved. Somatoform pain disorder is a preoccupation with pain in the
absence of physical disease.
V2. : (A) ³I went to the mall with my friends last Saturday´
  : Clients with panic disorder tent to be socially withdrawn. Going to the mall is a sign of working on
avoidance behaviors. Hyperventilating is a key symptom of panic disorder. Teaching breathing control is a major
intervention for clients with panic disorder. The client taking medications for panic disorder; such as tricylic
antidepressants and benzodiazepines, must be weaned off these drugs. Most clients with panic disorder with
agoraphobia don¶t have nutritional problems.
V3. : (A) ³I¶m sleeping better and don¶t have nightmares´
  : MAO inhibitors are used to treat sleep problems, nightmares, and intrusive daytime thoughts in
individual with posttraumatic stress disorder. MAO inhibitors aren¶t used to help control flashbacks or phobias or to
decrease the craving for alcohol.
V4. : (D) Stopping the drug can cause withdrawal symptoms
  Stopping antianxiety drugs such as benzodiazepines can cause the client to have withdrawal symptoms.
Stopping a benzodiazepine doesn¶t tend to cause depression, increase cognitive abilities, or decrease sleeping
difficulties.
V5. : (B) Behavioral difficulties
  : Adolescents tend to demonstrate severe irritability and behavioral problems rather than simply a
depressed mood. Anxiety disorder is more commonly associated with small children rather than with adolescents.
Cognitive impairment is typically associated with delirium or dementia. Labile mood is more characteristic of a
client with cognitive impairment or bipolar disorder.
V6. : (D) It¶s a mood disorder similar to major depression but of mild to moderate severity
  : Dysthymic disorder is a mood disorder similar to major depression but it remains mild to moderate in
severity. Cyclothymic disorder is a mood disorder characterized by a mood range from moderate depression to
hypomania. Bipolar I disorder is characterized by a single manic episode with no past major depressive episodes.
Seasonalaffective disorder is a form of depression occurring in the fall and winter.
V7. : (A) Vascular dementia has more abrupt onset
  : Vascular dementia differs from Alzheimer¶s disease in that it has a more abrupt onset and runs a highly
variable course. Personally change is common in Alzheimer¶s disease. The duration of delirium is usually brief. The
inability to carry out motor activities is common in Alzheimer¶s disease.
VV. : (C) Drug intoxication
  : This client was taking several medications that have a propensity for producing delirium; digoxin (a
digitalis glycoxide), furosemide (a thiazide diuretic), and diazepam (a benzodiazepine). Sufficient supporting data
don¶t exist to suspect the other options as causes.
V9. : (D) The client is experiencing visual hallucination
  : The presence of a sensory stimulus correlates with the definition of a hallucination, which is a false
sensory perception. Aphasia refers to a communication problem. Dysarthria is difficulty in speech production. Flight
of ideas is rapid shifting from one topic to another.
90. : (D) The client looks at the shadow on a wall and tells the nurse she sees frightening faces on the wall.
  : Minor memory problems are distinguished from dementia by their minor severity and their lack of significant
interference with the client¶s social or occupational lifestyle. Other options would be included in the history data but don¶t
directly correlate with the client¶s lifestyle.
91. : (D) Loose association
  : Loose associations are conversations that constantly shift in topic. Concrete thinking implies highly definitive
thought processes. Flight of ideas is characterized by conversation that¶s disorganized from the onset. Loose associations don¶t
necessarily start in a cogently, then becomes loose.
92. : (C) Paranoid
  : Because of their suspiciousness, paranoid personalities ascribe malevolent activities to others and tent to be
defensive, becoming quarrelsome and argumentative. Clients with antisocial personality disorder can also be antagonistic and
argumentative but are less suspicious than paranoid personalities. Clients with histrionic personality disorder are dramatic, not
suspicious and argumentative. Clients with schizoid personality disorder are usually detached from other and tend to have
eccentric behavior.
93. : (C) Explain that the drug is less affective if the client smokes
  : Olanzapine (Zyprexa) is less effective for clients who smoke cigarettes. Serotonin syndrome occurs with
clients who take a combination of antidepressant medications. Olanzapine doesn¶t cause euphoria, and
extrapyramidal adverse reactions aren¶t a problem.However, the client should be aware of adverse effects such as
tardive dyskinesia.
94. : (A) Lack of honesty
  : Clients with antisocial personality disorder tent to engage in acts of dishonesty, shown by lying. Clients
with schizotypal personality disorder tend to be superstitious. Clients with histrionic personality disorders tend to
overreact to frustrations and disappointments, have temper tantrums, and seek attention.
95. : (A) ³I¶m not going to look just at the negative things about myself´
  : As the clients makes progress on improving self-esteem, selfblame and negative self evaluation will
decrease. Clients with dependent personality disorder tend to feel fragile and inadequate and would be extremely
unlikely to discuss their level of competence and progress. These clients focus on self and aren¶t envious or jealous.
Individuals with dependent personality disorders don¶t take over situations because they see themselves as inept and
inadequate.
96. : (C) Assess for possible physical problems such as rash
  : Clients with schizophrenia generally have poor visceral recognition because they live so fully in their
fantasy world. They need to have as in-depth assessment of physical complaints that may spill over into their
delusional symptoms. Talking with the client won¶t provide a assessment of his itching, and itching isn¶t as adverse
reaction of antipsychotic drugs, calling the physician to get the client¶s medication increased doesn¶t address his
physical complaints.
97. : (B) Echopraxia
  : Echopraxia is the copying of another¶s behaviors and is the result of the loss of ego boundaries.
Modeling is the conscious copying of someone¶s behaviors. Ego-syntonicity refers to behaviors that correspond with
the individual¶s sense of self. Ritualism behaviors are repetitive and compulsive.
9V. : (C) Hallucination
  : Hallucinations are sensory experiences that are misrepresentations of reality or have no basis in reality.
Delusions are beliefs not based in reality. Disorganized speech is characterized by jumping from one topic to the
next or using unrelated words. An idea of reference is a belief that an unrelated situation holds special meaning for
the client.
99. : (C) Regression
  : Regression, a return to earlier behavior to reduce anxiety, is the basic defense mechanism in
schizophrenia. Projection is a defense mechanism in which one blames others and attempts to justify actions; it¶s
used primarily by people with paranoid schizophrenia and delusional disorder. Rationalization is a defense
mechanism used to justify one¶s action. Repression is the basic defense mechanism in the neuroses; it¶s an
involuntary exclusion of painful thoughts, feelings, or experiences from awareness.
100. : (A) Should report feelings of restlessness or agitation at once
  : Agitation and restlessness are adverse effect of haloperidol and can be treated with antocholinergic
drugs. Haloperidol isn¶t likely to cause photosensitivity or control essential hypertension. Although the client may
experience increased concentration and activity, these effects are due to a decreased in symptoms, not the drug itself.
















c 
c ,-. ,,-/
,-. ,,-/
1. Which element in the circular chain of infection can be eliminated by preserving skin integrity?

a. Host c. Mode of transmission


b. Reservoir d. Portal of entry

2. Which of the following will probably result in a break in sterile technique for respiratory isolation?

a. Opening the patient¶s window to the outside environment


b. Turning on the patient¶s room ventilator
c. Opening the door of the patient¶s room leading into the hospital corridor
d. Failing to wear gloves when administering a bed bath

3. Which of the following patients is at greater risk for contracting an infection?

a. A patient with leukopenia


b. A patient receiving broad-spectrum antibiotics
c. A postoperative patient who has undergone orthopedic surgery
d. A newly diagnosed diabetic patient

4. Effective hand washing requires the use of:

a. Soap or detergent to promote emulsification c. A disinfectant to increase surface tension


b. Hot water to destroy bacteria d. All of the above

5. After routine patient contact, hand washing should last at least:

a. 30 seconds c. 2 minute
b. 1 minute d. 3 minutes

6. Which of the following procedures always requires surgical asepsis?

a. Vaginal instillation of conjugated estrogen c. Nasogastric tube insertion


b. Urinary catheterization d. Colostomy irrigation

7. Sterile technique is used whenever:

a. Strict isolation is required c. Invasive procedures are performed


b. Terminal disinfection is performed d. Protective isolation is necessary

V. Which of the following constitutes a break in sterile technique while preparing a sterile field for a dressing
change?

a. Using sterile forceps, rather than sterile gloves, to handle a sterile item
b. Touching the outside wrapper of sterilized material without sterile gloves
c. Placing a sterile object on the edge of the sterile field
d. Pouring out a small amount of solution (15 to 30 ml) before pouring the solution into a sterile container

9. A natural body defense that plays an active role in preventing infection is:

a. Yawning c. Hiccupping
b. Body hair d. Rapid eye movements
10. All of the following statement are true about donning sterile gloves except:

a. The first glove should be picked up by grasping the inside of the cuff.
b. The second glove should be picked up by inserting the gloved fingers under the cuff outside the glove.
c. The gloves should be adjusted by sliding the gloved fingers under the sterile cuff and pulling the glove over the
wrist
d. The inside of the glove is considered sterile

11.When removing a contaminated gown, the nurse should be careful that the first thing she touches is the:

a. Waist tie and neck tie at the back of the gown c. Cuffs of the gown
b. Waist tie in front of the gown d. Inside of the gown

12.Which of the following nursing interventions is considered the most effective form or universal precautions?

a. Cap all used needles before removing them from their syringes
b. Discard all used uncapped needles and syringes in an impenetrable protective container
c. Wear gloves when administering IM injections
d. Follow enteric precautions

13.All of the following measures are recommended to prevent pressure ulcers except:

a. Massaging the reddened are with lotion c. Adhering to a schedule for positioning and turning
b. Using a water or air mattress d. Providing meticulous skin care

14.Which of the following blood tests should be performed before a blood transfusion?

a. Prothrombin and coagulation time


b. Blood typing and cross-matching
c. Bleeding and clotting time
d. Complete blood count (CBC) and electrolyte levels.

15.The primary purpose of a platelet count is to evaluate the:

a. Potential for clot formation c. Presence of an antigen-antibody response


b. Potential for bleeding d. Presence of cardiac enzymes

16.Which of the following white blood cell (WBC) counts clearly indicates leukocytosis?

a. 4,500/mm3 c. 10,000/mm3
b. 7,000/mm3 d. 25,000/mm3

17. After 5 days of diuretic therapy with 20mg of furosemide (Lasix) daily, a patient begins to exhibit fatigue,
muscle cramping and muscle weakness. These symptoms probably indicate that the patient is experiencing:

a. Hypokalemia c. Anorexia
b. Hyperkalemia d. Dysphagia

1V.Which of the following statements about chest X-ray is false?

a. No contradictions exist for this test


b. Before the procedure, the patient should remove all jewelry, metallic objects, and buttons above the waist
c. A signed consent is not required
d. Eating, drinking, and medications are allowed before this test
19.The most appropriate time for the nurse to obtain a sputum specimen for culture is:

a. Early in the morning c. After aerosol therapy


b. After the patient eats a light breakfast d. After chest physiotherapy

20.A patient with no known allergies is to receive penicillin every 6 hours. When administering the medication, the
nurse observes a fine rash on the patient¶s skin. The most appropriate nursing action would be to:

a. Withhold the moderation and notify the physician c. Administer the medication with an antihistamine
b. Administer the medication and notify the physician d. Apply corn starch soaks to the rash

21.All of the following nursing interventions are correct when using the Ztrack method of drug injection except:

a. Prepare the injection site with alcohol


b. Use a needle that¶s a least 1´ long
c. Aspirate for blood before injection
d. Rub the site vigorously after the injection to promote absorption

22.The correct method for determining the vastus lateralis site for I.M. injection is to:

a. Locate the upper aspect of the upper outer quadrant of the buttock about 5 to V cm below the iliac crest
b. Palpate the lower edge of the acromion process and the midpoint lateral aspect of the arm
c. Palpate a 1´ circular area anterior to the umbilicus
d. Divide the area between the greater femoral trochanter and the lateral femoral condyle into thirds, and select the
middle third on the anterior of the thigh

23.The mid-deltoid injection site is seldom used for I.M. injections because it:

a. Can accommodate only 1 ml or less of medication c. Can be used only when the patient is lying down
b. Bruises too easily d. Does not readily parenteral medication

24.The appropriate needle size for insulin injection is:

a. 1VG, 1 .´ long c. 22G, 1 .´ long


b. 22G, 1´ long d. 25G, 5/V´ long

25.The appropriate needle gauge for intradermal injection is:

a. 20G c. 25G
b. 22G d. 26G

26.Parenteral penicillin can be administered as an:

a. IM injection or an IV solution c. Intradermal or subcutaneous injection


b. IV or an intradermal injection d. IM or a subcutaneous injection

27.The physician orders gr 10 of aspirin for a patient. The equivalent dose in milligrams is:

a. 0.6 mg c. 60 mg
b. 10 mg d. 600 mg

2V.The physician orders an IV solution of dextrose 5% in water at 100ml/hour. What would the flow rate be if the
drop factor is 15 gtt = 1 ml?

a. 5 gtt/minute c. 25 gtt/minute
b. 13 gtt/minute d. 50 gtt/minute
29.Which of the following is a sign or symptom of a hemolytic reaction to blood transfusion?

a. Hemoglobinuria c. Urticaria
b. Chest pain d. Distended neck veins

30.Which of the following conditions may require fluid restriction?

a. Fever c. Renal Failure


b. Chronic Obstructive Pulmonary Disease d. Dehydration

31.All of the following are common signs and symptoms of phlebitis except:

a. Pain or discomfort at the IV insertion site c. A red streak exiting the IV insertion site
b. Edema and warmth at the IV insertion site d. Frank bleeding at the insertion site

32.The best way of determining whether a patient has learned to instill ear medication properly is for the nurse to:

a. Ask the patient if he/she has used ear drops before


b. Have the patient repeat the nurse¶s instructions using her own words
c. Demonstrate the procedure to the patient and encourage to ask questions
d. Ask the patient to demonstrate the procedure

33.Which of the following types of medications can be administered via gastrostomy tube?

a. Any oral medications


b. Capsules whole contents are dissolve in water
c. Enteric-coated tablets that are thoroughly dissolved in water
d. Most tablets designed for oral use, except for extended-duration compounds

34.A patient who develops hives after receiving an antibiotic is exhibiting drug:

a. Tolerance c. Synergism
b. Idiosyncrasy d. Allergy

35.A patient has returned to his room after femoral arteriography. All of the following are appropriate nursing
interventions except:

a. Assess femoral, popliteal, and pedal pulses every 15 minutes for 2 hours
b. Check the pressure dressing for sanguineous drainage
c. Assess a vital signs every 15 minutes for 2 hours
d. Order a hemoglobin and hematocrit count 1 hour after the arteriography

36.The nurse explains to a patient that a cough:

a. Is a protective response to clear the respiratory tract of irritants


b. Is primarily a voluntary action
c. Is induced by the administration of an antitussive drug
d. Can be inhibited by ³splinting´ the abdomen

37.An infected patient has chills and begins shivering. The best nursing intervention is to:

a. Apply iced alcohol sponges c. Provide additional bedclothes


b. Provide increased cool liquids d. Provide increased ventilation
3V.A clinical nurse specialist is a nurse who has:

a. Been certified by the National League for Nursing


b. Received credentials from the Philippine Nurses¶ Association
c. Graduated from an associate degree program and is a registered professional nurse
d. Completed a master¶s degree in the prescribed clinical area and is a registered professional nurse.

39.The purpose of increasing urine acidity through dietary means is to:

a. Decrease burning sensations c. Change the urine¶s concentration


b. Change the urine¶s color d. Inhibit the growth of microorganisms

40.Clay colored stools indicate:

a. Upper GI bleeding c. An effect of medication


b. Impending constipation d. Bile obstruction

41. In which step of the nursing process would the nurse ask a patient if the medication she administered relieved his
pain?

a. Assessmen t c. Planning
b. Analysis d. Evaluation

42.All of the following are good sources of vitamin A except:

a. White potatoes c. Apricots


b. Carrots d. Egg yolks

43.Which of the following is a primary nursing intervention necessary for all patients with a Foley Catheter in
place?

a. Maintain the drainage tubing and collection bag level with the patient¶s bladder
b. Irrigate the patient with 1% Neosporin solution three times a daily
c. Clamp the catheter for 1 hour every 4 hours to maintain the bladder¶s elasticity
d. Maintain the drainage tubing and collection bag below bladder level to facilitate drainage by gravity

44.The ELISA test is used to:

a. Screen blood donors for antibodies to human immunodeficiency virus (HIV)


b. Test blood to be used for transfusion for HIV antibodies
c. Aid in diagnosing a patient with AIDS
d. All of the above

45.The two blood vessels most commonly used for TPN infusion are the:

a. Subclavian and jugular veins c. Femoral and subclavian veins


b. Brachial and subclavian veins d. Brachial and femoral veins

46.Effective skin disinfection before a surgical procedure includes which of the following methods?

a. Shaving the site on the day before surgery


b. Applying a topical antiseptic to the skin on the evening before surgery
c. Having the patient take a tub bath on the morning of surgery
d. Having the patient shower with an antiseptic soap on the evening v=before and the morning of surgery
47.When transferring a patient from a bed to a chair, the nurse should use which muscles to avoid back injury?

a. Abdominal muscles c. Leg muscles


b. Back muscles d. Upper arm muscles

4V.Thrombophlebitis typically develops in patients with which of the following conditions?

a. Increases partial thromboplastin time


b. Acute pulsus paradoxus
c. An impaired or traumatized blood vessel wall
d. Chronic Obstructive Pulmonary Disease (COPD)

49.In a recumbent, immobilized patient, lung ventilation can become altered, leading to such respiratory
complications as:

a. Respiratory acidosis, ateclectasis, and hypostatic pneumonia


b. Appneustic breathing, atypical pneumonia and respiratory alkalosis
c. Cheyne-Strokes respirations and spontaneous pneumothorax
d. Kussmail¶s respirations and hypoventilation

50.Immobility impairs bladder elimination, resulting in such disorders as

a. Increased urine acidity and relaxation of the perineal muscles, causing incontinence
b. Urine retention, bladder distention, and infection
c. Diuresis, natriuresis, and decreased urine specific gravity
d. Decreased calcium and phosphate levels in the urine
Nursing Crib ± Student Nurses¶ Community 200
0 . , $(,-. ,,-/
1. .. In the circular chain of infection, pathogens must be able to leave their reservoir and be transmitted to a
susceptible host through a portal of entry, such as broken skin.
2. . Respiratory isolation, like strict isolation, requires that the door to the door patient¶s room remain closed.
However, the patient¶s room should be well ventilated, so opening the window or turning on the ventricular is
desirable. The nurse does not need to wear gloves for respiratory isolation, but good hand washing is important for
all types of isolation.
3. . Leukopenia is a decreased number of leukocytes (white blood cells), which are important in resisting infection.
None of the other situations would put the patient at risk for contracting an infection; taking broadspectrum
antibiotics might actually reduce the infection risk.
4. . Soaps and detergents are used to help remove bacteria because of their ability to lower the surface tension of
water and act as emulsifying agents. Hot water may lead to skin irritation or burns.
5. . Depending on the degree of exposure to pathogens, hand washing may last from 10 seconds to 4 minutes.
After routine patient contact, hand washing for 30 seconds effectively minimizes the risk of pathogen transmission.
6. +. The urinary system is normally free of microorganisms except at the urinary meatus. Any procedure that
involves entering this system must use surgically aseptic measures to maintain a bacteria-free state.
7. . All invasive procedures, including surgery, catheter insertion, and administration of parenteral therapy, require
sterile technique to maintain a sterile environment. All equipment must be sterile, and the nurse and the physician
must wear sterile gloves and maintain surgical asepsis. In the operating room, the nurse and physician are required
to wear sterile gowns, gloves, masks, hair covers, and shoe covers for all invasive procedures. Strict isolation
requires the use of clean gloves, masks, gowns and equipment to prevent the transmission of highly communicable
diseases by contact or by airborne routes. Terminal disinfection is the disinfection of all contaminated supplies and
equipment after a patient has been discharged to prepare them for reuse by another patient. The purpose of
protective (reverse) isolation is to prevent a person with seriously impaired resistance from coming into contact who
potentially pathogenic organisms.
V. . The edges of a sterile field are considered contaminated. When sterile items are allowed to come in contact
with the edges of the field, the sterile items also become contaminated.
9. +. Hair on or within body areas, such as the nose, traps and holds particles that contain microorganisms. Yawning
and hiccupping do not prevent microorganisms from entering or leaving the body. Rapid eye movement marks the
stage of sleep during which dreaming occurs.
10. .. The inside of the glove is always considered to be clean, but not sterile.
11. . The back of the gown is considered clean, the front is contaminated. So, after removing gloves and washing
hands, the nurse should untie the back of the gown; slowly move backward away from the gown, holding the inside
of the gown and keeping the edges off the floor; turn and fold the gown inside out; discard it in a contaminated linen
container; then wash her hands again.
12. +. According to the Centers for Disease Control (CDC), blood-to-blood contact occurs most commonly when a
health care worker attempts to cap a used needle. Therefore, used needles should never be recapped; instead they
should be inserted in a specially designed puncture resistant, labeled container. Wearing gloves is not always
necessary when administering an I.M. injection. Enteric precautions prevent the transfer of pathogens via feces.
13. . Nurses and other health care professionals previously believed that massaging a reddened area with lotion
would promote venous return and reduce edema to the area. However, research has shown that massage only
increases the likelihood of cellular ischemia and necrosis to the area.
14. +. Before a blood transfusion is performed, the blood of the donor and recipient must be checked for
compatibility. This is done by blood typing (a test that determines a person¶s blood type) and cross-matching (a
procedure that determines the compatibility of the donor¶s and recipient¶s blood after the blood types has been
matched). If the blood specimens are incompatible, hemolysis and antigen-antibody reactions will occur.
15. . Platelets are disk-shaped cells that are essential for blood coagulation. A platelet count determines the number
of thrombocytes in blood available for promoting hemostasis and assisting with blood coagulation after injury. It
also is used to evaluate the patient¶s potential for bleeding; however, this is not its primary purpose. The normal
count ranges from 150,000 to 350,000/mm3. A count of 100,000/mm3 or less indicates a potential for bleeding;
count of less than 20,000/mm3 is associated with spontaneous bleeding.
16. .. Leukocytosis is any transient increase in the number of white blood cells (leukocytes) in the blood. Normal
WBC counts range from 5,000 to 100,000/mm3. Thus, a count of 25,000/mm3 indicates leukocytosis.
17. . Fatigue, muscle cramping, and muscle weaknesses are symptoms of hypokalemia (an inadequate potassium
level), which is a potential side effect of diuretic therapy. The physician usually orders supplemental potassium to
prevent hypokalemia in patients receiving diuretics. Anorexia is another symptom of hypokalemia. Dysphagia
means difficulty swallowing.
1V. . Pregnancy or suspected pregnancy is the only contraindication for a chest X-ray. However, if a chest X-ray is
necessary, the patient can wear a lead apron to protect the pelvic region from radiation. Jewelry, metallic objects,
and buttons would interfere with the X-ray and thus should not be worn above the waist. A signed consent is not
required because a chest X-ray is not an invasive examination. Eating, drinking and medications are allowed
because the X-ray is of the chest, not the abdominal region.
19. . Obtaining a sputum specimen early in this morning ensures an adequate supply of bacteria for culturing and
decreases the risk of contamination from food or medication.
20. . Initial sensitivity to penicillin is commonly manifested by a skin rash, even in individuals who have not been
allergic to it previously. Because of the danger of anaphylactic shock, he nurse should withhold the drug and notify
the physician, who may choose to substitute another drug. Administering an antihistamine is a dependent nursing
intervention that requires a written physician¶s order. Although applying corn starch to the rash may relieve
discomfort, it is not the nurse¶s top priority in such a potentially life-threatening situation.
21. .. The Z-track method is an I.M. injection technique in which the patient¶s skin is pulled in such a way that the
needle track is sealed off after the injection. This procedure seals medication deep into the muscle, thereby
minimizing skin staining and irritation. Rubbing the injection site is contraindicated because it may cause the
medication to extravasate into the skin.
22. .. The vastus lateralis, a long, thick muscle that extends the full length of the thigh, is viewed by many
clinicians as the site of choice for I.M. injections because it has relatively few major nerves and blood vessels. The
middle third of the muscle is recommended as the injection site. The patient can be in a supine or sitting position for
an injection into this site.
23. . The mid-deltoid injection site can accommodate only 1 ml or less of medication because of its size and
location (on the deltoid muscle of the arm, close to the brachial artery and radial nerve).
24. .. A 25G, 5/V´ needle is the recommended size for insulin injection because insulin is administered by the
subcutaneous route. An 1VG, 1 .´ needle is usually used for I.M. injections in children, typically in the vastus
lateralis. A 22G, 1 .´ needle is usually used for adult I.M. injections, which are typically administered in the vastus
lateralis or ventrogluteal site.
25. .. Because an intradermal injection does not penetrate deeply into the skin, a small-bore 25G needle is
recommended. This type of injection is used primarily to administer antigens to evaluate reactions for allergy or
sensitivity studies. A 20G needle is usually used for I.M. injections of oilbased medications; a 22G needle for I.M.
injections; and a 25G needle, for I.M. injections; and a 25G needle, for subcutaneous insulin injections.
26. . Parenteral penicillin can be administered I.M. or added to a solution and given I.V. It cannot be administered
subcutaneously or intradermally.
27. .. gr 10 x 60mg/gr 1 = 600 mg
2V. . 100ml/60 min X 15 gtt/ 1 ml = 25 gtt/minute
29. . Hemoglobinuria, the abnormal presence of hemoglobin in the urine, indicates a hemolytic reaction
(incompatibility of the donor¶s and recipient¶s blood). In this reaction, antibodies in the recipient¶s plasma combine
rapidly with donor RBC¶s; the cells are hemolyzed in either circulatory or reticuloendothelial system. Hemolysis
occurs more rapidly in ABO incompatibilities than in Rh incompatibilities. Chest pain and urticaria may be
symptoms of impending anaphylaxis. Distended neck veins are an indication of hypervolemia.
30. . In real failure, the kidney loses their ability to effectively eliminate wastes and fluids. Because of this,
limiting the patient¶s intake of oral and I.V. fluids may be necessary. Fever, chronic obstructive pulmonary disease,
and dehydration are conditions for which fluids should be encouraged.
31. .. Phlebitis, the inflammation of a vein, can be caused by chemical irritants (I.V. solutions or medications),
mechanical irritants (the needle or catheter used during venipuncture or cannulation), or a localized allergic reaction
to the needle or catheter. Signs and symptoms of phlebitis include pain or discomfort, edema and heat at the I.V.
insertion site, and a red streak going up the arm or leg from the I.V. insertion site.
32. .. Return demonstration provides the most certain evidence for evaluating the effectiveness of patient teaching.
33. .. Capsules, enteric-coated tablets, and most extended duration or sustained release products should not be
dissolved for use in a gastrostomy tube. They are pharmaceutically manufactured in these forms for valid reasons,
and altering them destroys their purpose. The nurse should seek an alternate physician¶s order when an ordered
medication is inappropriate for delivery by tube.
34. .. A drug-allergy is an adverse reaction resulting from an immunologic response following a previous sensitizing exposure to
the drug. The reaction can range from a rash or hives to anaphylactic shock. Ê  to a drug means that the patient
experiences a decreasing physiologic response to repeated administration of the drug in the same dosage.    is an
individual¶s unique hypersensitivity to a drug, food, or other substance; it appears to be genetically determined.   , is a
drug interaction in which the sum of the drug¶s combined effects is greater than that of their separate effects.
35. .. A hemoglobin and hematocrit count would be ordered by the physician if bleeding were suspected. The other answers are
appropriate nursing interventions for a patient who has undergone femoral arteriography.
36. . Coughing, a protective response that clears the respiratory tract of irritants, usually is involuntary; however it can be
voluntary, as when a patient is taught to perform coughing exercises. An antitussive drug inhibits coughing. Splinting the
abdomen supports the abdominal muscles when a patient coughs.
37. . In an infected patient, shivering results from the body¶s attempt to increase heat production and the production of
neutrophils and phagocytotic action through increased skeletal muscle tension and contractions. Initial vasoconstriction may
cause skin to feel cold to the touch. Applying additional bed clothes helps to equalize the body temperature and stop the chills.
Attempts to cool the body result in further shivering, increased metabloism, and thus increased heat production.
3V. .. A clinical nurse specialist must have completed a master¶s degree in a clinical specialty and be a registered professional
nurse. The National League of Nursing accredits educational programs in nursing and provides a testing service to evaluate
student nursing competence but it does not certify nurses. The American Nurses Association identifies requirements for
certification and offers examinations for certification in many areas of nursing., such as medical surgical nursing. These
certification (credentialing) demonstrates that the nurse has the knowledge and the ability to provide high quality nursing care in
the area of her certification. A graduate of an associate degree program is not a clinical nurse specialist: however, she is prepared
to provide bed side nursing with a high degree of knowledge and skill. She must successfully complete the licensing examination
to become a registered professional nurse.
39. .. Microorganisms usually do not grow in an acidic environment.
40. .. Bile colors the stool brown. Any inflammation or obstruction that impairs bile flow will affect the stool pigment, yielding
light, clay-colored stool. Upper GI bleeding results in black or tarry stool. Constipation is characterized by small, hard masses.
Many medications and foods will discolor stool ± for example, drugs containing iron turn stool black.; beets turn stool red.
41. .. In the evaluation step of the nursing process, the nurse must decide whether the patient has achieved the expected outcome
that was identified in the planning phase.
42. . The main sources of vitamin A are yellow and green vegetables (such as carrots, sweet potatoes, squash, spinach, collard
greens, broccoli, and cabbage) and yellow fruits (such as apricots, and cantaloupe). Animal sources include liver, kidneys, cream,
butter, and egg yolks.
43. .. Maintaing the drainage tubing and collection bag level with the patient¶s bladder could result in reflux of urine into the
kidney. Irrigating the bladder with Neosporin and clamping the catheter for 1 hour every 4 hours must be prescribed by a
physician.
44. .. The ELISA test of venous blood is used to assess blood and potential blood donors to human immunodeficiency virus
(HIV). A positive ELISA test combined with various signs and symptoms helps to diagnose acquired immunodeficiency
syndrome (AIDS)
45. .. Tachypnea (an abnormally rapid rate of breathing) would indicate that the patient was still hypoxic (deficient in
oxygen).The partial pressures of arterial oxygen and carbon dioxide listed are within the normal range. Eupnea refers to normal
respiration.
46. .. Studies have shown that showering with an antiseptic soap before surgery is the most effective method of removing
microorganisms from the skin. Shaving the site of the intended surgery might cause breaks in the skin, thereby increasing the risk
of infection; however, if indicated, shaving, should be done immediately before surgery, not the day before. A topical antiseptic
would not remove microorganisms and would be beneficial only after proper cleaning and rinsing. Tub bathing might transfer
organisms to another body site rather than rinse them away.
47. . The leg muscles are the strongest muscles in the body and should bear the greatest stress when lifting. Muscles of the
abdomen, back, and upper arms may be easily injured.
4V. . The factors, known as Virchow¶s triad, collectively predispose a patient to thromboplebitis; impaired venous return to the
heart, blood hypercoagulability, and injury to a blood vessel wall. Increased partial thromboplastin time indicates a prolonged
bleeding time during fibrin clot formation, commonly the result of anticoagulant (heparin) therapy. Arterial blood disorders (such
as pulsus paradoxus) and lung diseases (such as COPD) do not necessarily impede venous return of injure vessel walls.
49. . Because of restricted respiratory movement, a recumbent, immobilize patient is at particular risk for respiratory acidosis
from poor gas exchange; atelectasis from reduced surfactant and accumulated mucus in the bronchioles, and hypostatic
pneumonia from bacterial growth caused by stasis of mucus secretions.
50. +. The immobilized patient commonly suffers from urine retention caused by decreased muscle tone in the perineum. This
leads to bladder distention and urine stagnation, which provide an excellent medium for bacterial growth leading to infection.
Immobility also results in more alkaline urine with excessive amounts of calcium, sodium and phosphate, a gradual decrease in
urine production, and an increased specific gravity.
c      
  $ .$. $
1. For the client who is using oral contraceptives, the nurse informs the client about the need to take the pill at the
same time each day to accomplish which of the following?

a. Decrease the incidence of nausea c. Reduce side effects


b. Maintain hormonal levels d. Prevent drug interactions

2. When teaching a client about contraception. Which of the following would the nurse include as the most effective
method for preventing sexually transmitted infections?

a. Spermicides c. Condoms
b. Diaphragm d. Vasectomy

3. When preparing a woman who is 2 days postpartum for discharge, recommendations for which of the following
contraceptive methods would be avoided?

a. Diaphragm c. Oral contraceptives


b. Female condom d. Rhythm method

4. For which of the following clients would the nurse expect that an intrauterine device would be recommended?

a. Woman over age 35 c. Promiscuous young adult


b. Nulliparous woman d. Postpartum client

5. A client in her third trimester tells the nurse, ³I¶m constipated all the time!´ Which of the following should the
nurse recommend?

a. Daily enemas c. Increased fiber intake


b. Laxatives d. Decreased fluid intake

6. Which of the following would the nurse use as the basis for the teaching plan when caring for a pregnant teenager
concerned about gaining too much weight during pregnancy?

a. 10 pounds per trimester c. . pound per week for 40 weeks


b. 1 pound per week for 40 weeks d. A total gain of 25 to 30 pounds

7. The client tells the nurse that her last menstrual period started on January 14 and ended on January 20. Using
Nagele¶s rule, the nurse determines her EDD to be which of the following?

a. September 27 c. November 7
b. October 21 d. December 27

V. When taking an obstetrical history on a pregnant client who states, ³I had a son born at 3V weeks gestation, a
daughter born at 30 weeks gestation and I lost a baby at about V weeks,´ the nurse should record her obstetrical
history as which of the following?

a. G2 T2 P0 A0 L2 c. G3 T2 P0 A0 L2
b. G3 T1 P1 A0 L2 d. G4 T1 P1 A1 L2

9. When preparing to listen to the fetal heart rate at 12 weeks¶ gestation, the nurse would use which of the
following?

a. Stethoscope placed midline at the umbilicus


b. Doppler placed midline at the suprapubic region
c. Fetoscope placed midway between the umbilicus and the xiphoid process
d. External electronic fetal monitor placed at the umbilicus
10.When developing a plan of care for a client newly diagnosed with gestational diabetes, which of the following
instructions would be the priority?

a. Dietary intake c. Exercise


b. Medication d. Glucose monitoring

11.A client at 24 weeks gestation has gained 6 pounds in 4 weeks. Which of the following would be the priority
when assessing the client?

a. Glucosuria c. Hand/face edema


b. Depression d. Dietary intake

12.A client 12 weeks¶ pregnant come to the emergency department with abdominal cramping and moderate vaginal
bleeding. Speculum examination reveals 2 to 3 cms cervical dilation. The nurse would document these findings as
which of the following?

a. Threatened abortion c. Complete abortion


b. Imminent abortion d. Missed abortion

13.Which of the following would be the priority nursing diagnosis for a client with an ectopic pregnancy?

a. Risk for infection c. Knowledge Deficit


b. Pain d. Anticipatory Grieving

14.Before assessing the postpartum client¶s uterus for firmness and position in relation to the umbilicus and midline,
which of the following should the nurse do first?

a. Assess the vital signs c. Ambulate her in the hall


b. Administer analgesia d. Assist her to urinate

15.Which of the following should the nurse do when a primipara who is lactating tells the nurse that she has sore
nipples?

a. Tell her to breast feed more frequently


b. Administer a narcotic before breast feeding
c. Encourage her to wear a nursing brassiere
d. Use soap and water to clean the nipples

16.The nurse assesses the vital signs of a client, 4 hours¶ postpartum that are as follows: BP 90/60; temperature
100.4oF; pulse 100 weak, thready; R 20 per minute. Which of the following should the nurse do first?

a. Report the temperature to the physician c. Assess the uterus for firmness and position
b. Recheck the blood pressure with another cuff d. Determine the amount of lochia

17.The nurse assesses the postpartum vaginal discharge (lochia) on four clients. Which of the following assessments
would warrant notification of the physician?

a. A dark red discharge on a 2-day postpartum client


b. A pink to brownish discharge on a client who is 5 days postpartum
c. Almost colorless to creamy discharge on a client 2 weeks after delivery
d. A bright red discharge 5 days after delivery
1V.A postpartum client has a temperature of 101.4oF, with a uterus that is tender when palpated, remains unusually
large, and not descending as normally expected. Which of the following should the nurse assess next?

a. Lochia c. Incision
b. Breasts d. Urine

19.Which of the following is the priority focus of nursing practice with the current early postpartum discharge?

a. Promoting comfort and restoration of health


b. Exploring the emotional status of the family
c. Facilitating safe and effective self-and newborn care
d. Teaching about the importance of family planning

20. Which of the following actions would be l effective in maintaining a neutral thermal environment for the
newborn?

a. Placing infant under radiant warmer after bathing


b. Covering the scale with a warmed blanket prior to weighing
c. Placing crib close to nursery window for family viewing
d. Covering the infant¶s head with a knit stockinette

21.A newborn who has an asymmetrical Moro reflex response should be further assessed for which of the
following?

a. Talipes equinovarus c. Congenital hypothyroidism


b. Fractured clavicle d. Increased intracranial pressure

22.During the first 4 hours after a male circumcision, assessing for which of the following is the priority?

a. Infection c. Discomfort
b. Hemorrhage d. Dehydration

23.The mother asks the nurse. ³What¶s wrong with my son¶s breasts? Why are they so enlarged?´ Whish of the
following would be the best response by the nurse?

a. ³The breast tissue is inflamed from the trauma experienced with birth´
b. ³A decrease in material hormones present before birth causes enlargement,´
c. ³You should discuss this with your doctor. It could be a malignancy´
d. ³The tissue has hypertrophied while the baby was in the uterus´

24.Immediately after birth the nurse notes the following on a male newborn: respirations 7V; apical hearth rate 160
BPM, nostril flaring; mild intercostals retractions; and grunting at the end of expiration. Which of the following
should the nurse do?

a. Call the assessment data to the physician¶s attention


b. Start oxygen per nasal cannula at 2 L/min.
c. Suction the infant¶s mouth and nares
d. Recognize this as normal first period of reactivity

25.The nurse hears a mother telling a friend on the telephone about umbilical cord care. Which of the following
statements by the mother indicates effective teaching?

a. ³Daily soap and water cleansing is best´


b. µAlcohol helps it dry and kills germs´
c. ³An antibiotic ointment applied daily prevents infection´
d. ³He can have a tub bath each day´
26.A newborn weighing 3000 grams and feeding every 4 hours needs 120 calories/kg of body weight every 24 hours
for proper growth and development. How many ounces of 20 cal/oz formula should this newborn receive at each
feeding to meet nutritional needs?

a. 2 ounces c. 4 ounces
b. 3 ounces d. 6 ounces

27.The postterm neonate with meconium-stained amniotic fluid needs care designed to especially monitor for which
of the following?

a. Respiratory problems c. Integumentary problems


b. Gastrointestinal problems d. Elimination problems

2V.When measuring a client¶s fundal height, which of the following techniques denotes the correct method of
measurement used by the nurse?

a. From the xiphoid process to the umbilicus c. From the symphysis pubis to the fundus
b. From the symphysis pubis to the xiphoid process d. From the fundus to the umbilicus

29.A client with severe preeclampsia is admitted with of BP 160/110, proteinuria, and severe pitting edema. Which
of the following would be most important to include in the client¶s plan of care?

a. Daily weights c. Right lateral positioning


b. Seizure precautions d. Stress reduction

30.A postpartum primipara asks the nurse, ³When can we have sexual intercourse again?´ Which of the following
would be the nurse¶s best response?

a. ³Anytime you both want to.´


b. ³As soon as choose a contraceptive method.´
c. ³When the discharge has stopped and the incision is healed.´
d. ³After your 6 weeks examination.´

31.When preparing to administer the vitamin K injection to a neonate, the nurse would select which of the following
sites as appropriate for the injection?

a. Deltoid muscle c. Vastus lateralis muscle


b. Anterior femoris muscle d. Gluteus maximus muscle

32.When performing a pelvic examination, the nurse observes a red swollen area on the right side of the vaginal
orifice. The nurse would document this as enlargement of which of the following?

a. Clitoris c. Skene¶s gland


b. Parotid gland d. Bartholin¶s gland

33.To differentiate as a female, the hormonal stimulation of the embryo that must occur involves which of the
following?

a. Increase in maternal estrogen secretion c. Secretion of androgen by the fetal gonad


b. Decrease in maternal androgen secretion d. Secretion of estrogen by the fetal gonad
34.A client at V weeks¶ gestation calls complaining of slight nausea in the morning hours. Which of the following
client interventions should the nurse question?

a. Taking 1 teaspoon of bicarbonate of soda in an V-ounce glass of water


b. Eating a few low-sodium crackers before getting out of bed
c. Avoiding the intake of liquids in the morning hours
d. Eating six small meals a day instead of thee large meals

35.The nurse documents positive ballottement in the client¶s prenatal record. The nurse understands that this
indicates which of the following?

a. Palpable contractions on the abdomen c. Fetal kicking felt by the client


b. Passive movement of the unengaged fetus d. Enlargement and softening of the uterus

36.During a pelvic exam the nurse notes a purple-blue tinge of the cervix. The nurse documents this as which of the
following?

a. Braxton-Hicks sign c. Goodell¶s sign


b. Chadwick¶s sign d. McDonald¶s sign

37.During a prenatal class, the nurse explains the rationale for breathing techniques during preparation for labor
based on the understanding that breathing techniques are most important in achieving which of the following?

a. Eliminate pain and give the expectant parents something to do


b. Reduce the risk of fetal distress by increasing uteroplacental perfusion
c. Facilitate relaxation, possibly reducing the perception of pain
d. Eliminate pain so that less analgesia and anesthesia are needed

3V.After 4 hours of active labor, the nurse notes that the contractions of a primigravida client are not strong enough
to dilate the cervix. Which of the following would the nurse anticipate doing?

a. Obtaining an order to begin IV oxytocin infusion


b. Administering a light sedative to allow the patient to rest for several hour
c. Preparing for a cesarean section for failure to progress
d. Increasing the encouragement to the patient when pushing begins

39.A multigravida at 3V weeks¶ gestation is admitted with painless, bright red bleeding and mild contractions every
7 to 10 minutes. Which of the following assessments should be avoided?

a. Maternal vital sign c. Contraction monitoring


b. Fetal heart rate d. Cervical dilation

40.Which of the following would be the nurse¶s most appropriate response to a client who asks why she must have a
cesarean delivery if she has a complete placenta previa?

a. ³You will have to ask your physician when he returns.´


b. ³You need a cesarean to prevent hemorrhage.´
c. ³The placenta is covering most of your cervix.´
d. ³The placenta is covering the opening of the uterus and blocking your baby.´

41.The nurse understands that the fetal head is in which of the following positions with a face presentation?

a. Completely flexed
b. Completely extended
c. Partially extended
d. Partially flexed
42.With a fetus in the left-anterior breech presentation, the nurse would expect the fetal heart rate would be most
audible in which of the following areas?

a. Above the maternal umbilicus and to the right of midline


b. In the lower-left maternal abdominal quadrant
c. In the lower-right maternal abdominal quadrant
d. Above the maternal umbilicus and to the left of midline

43.The amniotic fluid of a client has a greenish tint. The nurse interprets this to be the result of which of the
following?

a. Lanugo c. Meconium
b. Hydramnio d. Vernix

44.A patient is in labor and has just been told she has a breech presentation. The nurse should be particularly alert
for which of the following?

a. Quickening c. Pica
b. Ophthalmia neonatorum d. Prolapsed umbilical cord

45.When describing dizygotic twins to a couple, on which of the following would the nurse base the explanation?

a. Two ova fertilized by separate sperm c. Each ova with the same genotype
b. Sharing of a common placenta d. Sharing of a common chorion

46.Which of the following refers to the single cell that reproduces itself after conception?

a. Chromosome c. Zygote
b. Blastocyst d. Trophoblast

47.In the late 1950s, consumers and health care professionals began challenging the routine use of analgesics and
anesthetics during childbirth. Which of the following was an outgrowth of this concept?

a. Labor, delivery, recovery, postpartum (LDRP) c. Clinical nurse specialist


b. Nurse-midwifery d. Prepared childbirth

4V.A client has a midpelvic contracture from a previous pelvic injury due to a motor vehicle accident as a teenager.
The nurse is aware that this could prevent a fetus from passing through or around which structure during childbirth?

a. Symphysis pubis c. Ischial spines


b. Sacral promontory d. Pubic arch

49.When teaching a group of adolescents about variations in the length of the menstrual cycle, the nurse understands
that the underlying mechanism is due to variations in which of the following phases?

a. Menstrual phase c. Secretory phase


b. Proliferative phase d. Ischemic phase

50.When teaching a group of adolescents about male hormone production, which of the following would the nurse
include as being produced by the Leydig cells?

a. Follicle-stimulating hormone c. Leuteinizing hormone


b. Testosterone d. Gonadotropin releasing hormone
0 . , $(  $ .$. $
1. +. Regular timely ingestion of oral contraceptives is necessary to maintain hormonal levels of the drugs to
suppress the action of the hypothalamus and anterior pituitary leading to inappropriate secretion of FSH and LH.
Therefore, follicles do not mature, ovulation is inhibited, and pregnancy is prevented. The estrogen content of the
oral site contraceptive may cause the nausea, regardless of when the pill is taken. Side effects and drug interactions
may occur with oral contraceptives regardless of the time the pill is taken.
2. . Condoms, when used correctly and consistently, are the most effective contraceptive method or barrier against
bacterial and viral sexually transmitted infections. Although spermicides kill sperm, they do not provide reliable
protection against the spread of sexually transmitted infections, especially intracellular organisms such as HIV.
Insertion and removal of the diaphragm along with the use of the spermicides may cause vaginal irritations, which
could place the client at risk for infection transmission. Male sterilization eliminates spermatozoa from the ejaculate,
but it does not eliminate bacterial and/or viral microorganisms that can cause sexually transmitted infections.
3. . The diaphragm must be fitted individually to ensure effectiveness. Because of the changes to the reproductive
structures during pregnancy and following delivery, the diaphragm must be refitted, usually at the 6 weeks¶
examination following childbirth or after a weight loss of 15 lbs or more. In addition, for maximum effectiveness,
spermicidal jelly should be placed in the dome and around the rim. However, spermicidal jelly should not be
inserted into the vagina until involution is completed at approximately 6 weeks. Use of a female condom protects the
reproductive system from the introduction of semen or spermicides into the vagina and may be used after childbirth.
Oral contraceptives may be started within the first postpartum week to ensure suppression of ovulation. For the
couple who has determined the female¶s fertile period, using the rhythm method, avoidance of intercourse during
this period, is safe and effective.
4. . An IUD may increase the risk of pelvic inflammatory disease, especially in women with more than one sexual
partner, because of the increased risk of sexually transmitted infections. An UID should not be used if the woman
has an active or chronic pelvic infection, postpartum infection, endometrial hyperplasia or carcinoma, or uterine
abnormalities. Age is not a factor in determining the risks associated with IUD use. Most IUD users are over the age
of 30. Although there is a slightly higher risk for infertility in women who have never been pregnant, the IUD is an
acceptable option as long as the risk-benefit ratio is discussed. IUDs may be inserted immediately after delivery, but
this is not recommended because of the increased risk and rate of expulsion at this time.
5. . During the third trimester, the enlarging uterus places pressure on the intestines. This coupled with the effect
of hormones on smooth muscle relaxation causes decreased intestinal motility (peristalsis). Increasing fiber in the
diet will help fecal matter pass more quickly through the intestinal tract, thus decreasing the amount of water that is
absorbed. As a result, stool is softer and easier to pass. Enemas could precipitate preterm labor and/or electrolyte
loss and should be avoided. Laxatives may cause preterm labor by stimulating peristalsis and may interfere with the
absorption of nutrients. Use for more than 1 week can also lead to laxative dependency. Liquid in the diet helps
provide a semisolid, soft consistency to the stool. Eight to ten glasses of fluid per day are essential to maintain
hydration and promote stool evacuation.
6. .. To ensure adequate fetal growth and development during the 40 weeks of a pregnancy, a total weight gain 25
to 30 pounds is recommended: 1.5 pounds in the first 10 weeks; 9 pounds by 30 weeks; and 27.5 pounds by 40
weeks. The pregnant woman should gain less weight in the first and second trimester than in the third. During the
first trimester, the client should only gain 1.5 pounds in the first 10 weeks, not 1 pound per week. A weight gain of .
pound per week would be 20 pounds for the total pregnancy, less than the recommended amount.
7. +. To calculate the EDD by Nagele¶s rule, add 7 days to the first day of the last menstrual period and count back 3 months,
changing the year appropriately. To obtain a date of September 27, 7 days have been added to the last day of the LMP (rather
than the first day of the LMP), plus 4 months (instead of 3 months) were counted back. To obtain the date of November 7, 7 days
have been subtracted (instead of added) from the first day of LMP plus November indicates counting back 2 months (instead of 3
months) from January. To obtain the date of December 27, 7 days were added to the last day of the LMP (rather than the first day
of the LMP) and December indicates counting back only 1 month (instead of 3 months) from January.
V. .&The client has been pregnant four times, including current pregnancy (G). Birth at 3V weeks¶ gestation is considered full
term (T), while birth form 20 weeks to 3V weeks is considered preterm (P). A spontaneous abortion occurred at V weeks (A). She
has two living children (L).
9. +&At 12 weeks gestation, the uterus rises out of the pelvis and is palpable above the symphysis pubis. The Doppler intensifies
the sound of the fetal pulse rate so it is audible. The uterus has merely risen out of the pelvis into the abdominal cavity and is not
at the level of the umbilicus. The fetal heart rate at this age is not audible with a stethoscope. The uterus at 12 weeks is just above
the symphysis pubis in the abdominal cavity, not midway between the umbilicus and the xiphoid process. At 12 weeks the FHR
would be difficult to auscultate with a fetoscope. Although the external electronic fetal monitor would project the FHR, the uterus
has not risen to the umbilicus at 12 weeks.
10. &Although all of the choices are important in the management of diabetes, diet therapy is the mainstay of the
treatment plan and should always be the priority. Women diagnosed with gestational diabetes generally need only
diet therapy without medication to control their blood sugar levels. Exercise, is important for all pregnant women
and especially for diabetic women, because it burns up glucose, thus decreasing blood sugar. However, dietary
intake, not exercise, is the priority. All pregnant women with diabetes should have periodic monitoring of serum
glucose. However, those with gestational diabetes generally do not need daily glucose monitoring. The standard of
care recommends a fasting and 2- hour postprandial blood sugar level every 2 weeks.
11. &After 20 weeks¶ gestation, when there is a rapid weight gain, preeclampsia should be suspected, which may
be caused by fluid retention manifested by edema, especially of the hands and face. The three classic signs of
preeclampsia are hypertension, edema, and proteinuria. Although urine is checked for glucose at each clinic visit,
this is not the priority. Depression may cause either anorexia or excessive food intake, leading to excessive weight
gain or loss. This is not, however, the priority consideration at this time. Weight gain thought to be caused by
excessive food intake would require a 24-hour diet recall. However, excessive intake would not be the primary
consideration for this client at this time.
12. +&Cramping and vaginal bleeding coupled with cervical dilation signifies that termination of the pregnancy is
inevitable and cannot be prevented. Thus, the nurse would document an imminent abortion. In a threatened abortion,
cramping and vaginal bleeding are present, but there is no cervical dilation. The symptoms may subside or progress
to abortion. In a complete abortion all the products of conception are expelled. A missed abortion is early fetal
intrauterine death without expulsion of the products of conception.
13. +&For the client with an ectopic pregnancy, lower abdominal pain, usually unilateral, is the primary symptom.
Thus, pain is the priority. Although the potential for infection is always present, the risk is low in ectopic pregnancy
because pathogenic microorganisms have not been introduced from external sources. The client may have a limited
knowledge of the pathology and treatment of the condition and will most likely experience grieving, but this is not
the priority at this time.
14. .&Before uterine assessment is performed, it is essential that the woman empty her bladder. A full bladder will
interfere with the accuracy of the assessment by elevating the uterus and displacing to the side of the midline. Vital
sign assessment is not necessary unless an abnormality in uterine assessment is identified. Uterine assessment
should not cause acute pain that requires administration of analgesia. Ambulating the client is an essential
component of postpartum care, but is not necessary prior to assessment of the uterus.
15. &Feeding more frequently, about every 2 hours, will decrease the infant¶s frantic, vigorous sucking from
hunger and will decrease breast engorgement, soften the breast, and promote ease of correct latching-on for feeding.
Narcotics administered prior to breast feeding are passed through the breast milk to the infant, causing excessive
sleepiness. Nipple soreness is not severe enough to warrant narcotic analgesia. All postpartum clients, especially
lactating mothers, should wear a supportive brassiere with wide cotton straps. This does not, however, prevent or
reduce nipple soreness. Soaps are drying to the skin of the nipples and should not be used on the breasts of lactating
mothers. Dry nipple skin predisposes to cracks and fissures, which can become sore and painful.
16. .&A weak, thready pulse elevated to 100 BPM may indicate impending hemorrhagic shock. An increased pulse is a
compensatory mechanism of the body in response to decreased fluid volume. Thus, the nurse should check the amount of lochia
present. Temperatures up to 100.4VF in the first 24 hours after birth are related to the dehydrating effects of labor and are
considered normal. Although rechecking the blood pressure may be a correct choice of action, it is not the first action that should
be implemented in light of the other data. The data indicate a potential impending hemorrhage. Assessing the uterus for firmness
and position in relation to the umbilicus and midline is important, but the nurse should check the extent of vaginal bleeding first.
Then it would be appropriate to check the uterus, which may be a possible cause of the hemorrhage.
17. .&Any bright red vaginal discharge would be considered abnormal, but especially 5 days after delivery, when the lochia is
typically pink to brownish. Lochia rubra, a dark red discharge, is present for 2 to 3 days after delivery. Bright red vaginal
bleeding at this time suggests late postpartum hemorrhage, which occurs after the first 24 hours following delivery and is
generally caused by retained placental fragments or bleeding disorders. Lochia rubra is the normal dark red discharge occurring
in the first 2 to 3 days after delivery, containing epithelial cells, erythrocyes, leukocytes and decidua. Lochia serosa is a pink to
brownish serosanguineous discharge occurring from 3 to 10 days after delivery that contains decidua, erythrocytes, leukocytes,
cervical mucus, and microorganisms. Lochia alba is an almost colorless to yellowish discharge occurring from 10 days to 3
weeks after delivery and containing leukocytes, decidua, epithelial cells, fat, cervical mucus, cholesterol crystals, and bacteria.
1V. &The data suggests an infection of the endometrial lining of the uterus. The lochia may be decreased or copious, dark brown
in appearance, and foul smelling, providing further evidence of a possible infection. All the client¶s data indicate a uterine
problem, not a breast problem. Typically, transient fever, usually 101oF, may be present with breast engorgement. Symptoms of
mastitis include influenza-like manifestations. Localized infection of an episiotomy or C-section incision rarely causes systemic
symptoms, and uterine involution would not be affected. The client data do not include dysuria, frequency, or urgency, symptoms
of urinary tract infections, which would necessitate assessing the client¶s urine.
19. &Because of early postpartum discharge and limited time for teaching, the nurse¶s priority is to facilitate the
safe and effective care of the client and newborn. Although promoting comfort and restoration of health, exploring
the family¶s emotional status, and teaching about family planning are important in postpartum/newborn nursing
care, they are not the priority focus in the limited time presented by early post-partum discharge.
20. &Heat loss by radiation occurs when the infant¶s crib is placed too near cold walls or windows. Thus placing
the newborn¶s crib close to the viewing window would be least effective. Body heat is lost through evaporation
during bathing. Placing the infant under the radiant warmer after bathing will assist the infant to be rewarmed.
Covering the scale with a warmed blanket prior to weighing prevents heat loss through conduction. A knit cap
prevents heat loss from the head a large head, a large body surface area of the newborn¶s body.
21. +&A fractured clavicle would prevent the normal Moro response of symmetrical sequential extension and
abduction of the arms followed by flexion and adduction. In talipes equinovarus (clubfoot) the foot is turned
medially, and in plantar flexion, with the heel elevated. The feet are not involved with the Moro reflex.
Hypothyroiddism has no effect on the primitive reflexes. Absence of the Moror reflex is the most significant single
indicator of central nervous system status, but it is not a sign of increased intracranial pressure.
22. +&Hemorrhage is a potential risk following any surgical procedure. Although the infant has been given vitamin
K to facilitate clotting, the prophylactic dose is often not sufficient to prevent bleeding. Although infection is a
possibility, signs will not appear within 4 hours after the surgical procedure. The primary discomfort of circumcision
occurs during the surgical procedure, not afterward. Although feedings are withheld prior to the circumcision, the
chances of dehydration are minimal.
23. +&The presence of excessive estrogen and progesterone in the maternalfetal blood followed by prompt
withdrawal at birth precipitates breast engorgement, which will spontaneously resolve in 4 to 5 days after birth. The
trauma of the birth process does not cause inflammation of the newborn¶s breast tissue. Newborns do not have breast
malignancy. This reply by the nurse would cause the mother to have undue anxiety. Breast tissue does not
hypertrophy in the fetus or newborns.
24. .&The first 15 minutes to 1 hour after birth is the first period of reactivity involving respiratory and circulatory
adaptation to extrauterine life. The data given reflect the normal changes during this time period. The infant¶s
assessment data reflect normal adaptation. Thus, the physician does not need to be notified and oxygen is not
needed. The data do not indicate the presence of choking, gagging or coughing, which are signs of excessive
secretions. Suctioning is not necessary.
25. +&Application of 70% isopropyl alcohol to the cord minimizes microorganisms (germicidal) and promotes
drying. The cord should be kept dry until it falls off and the stump has healed. Antibiotic ointment should only be
used to treat an infection, not as a prophylaxis. Infants should not be submerged in a tub of water until the cord falls
off and the stump has completely healed.
26. +&To determine the amount of formula needed, do the following mathematical calculation. 3 kg x 120 cal/kg per
day = 360 calories/day feeding q 4 hours = 6 feedings per day = 60 calories per feeding: 60 calories per feeding; 60
calories per feeding with formula 20 cal/oz = 3 ounces per feeding. Based on the calculation. 2, 4 or 6 ounces are
incorrect.
27. &Intrauterine anoxia may cause relaxation of the anal sphincter and emptying of meconium into the amniotic
fluid. At birth some of the meconium fluid may be aspirated, causing mechanical obstruction or chemical
pneumonitis. The infant is not at increased risk for gastrointestinal problems. Even though the skin is stained with
meconium, it is noninfectious (sterile) and nonirritating. The postterm meconiumstained infant is not at additional
risk for bowel or urinary problems.
2V. &The nurse should use a nonelastic, flexible, paper measuring tape, placing the zero point on the superior
border of the symphysis pubis and stretching the tape across the abdomen at the midline to the top of the fundus.
The xiphoid and umbilicus are not appropriate landmarks to use when measuring the height of the fundus
(McDonald¶s measurement).
29. +&Women hospitalized with severe preeclampsia need decreased CNS stimulation to prevent a seizure. Seizure precautions
provide environmental safety should a seizure occur. Because of edema, daily weight is important but not the priority.
Preclampsia causes vasospasm and therefore can reduce utero-placental perfusion. The client should be placed on her left side to
maximize blood flow, reduce blood pressure, and promote diuresis. Interventions to reduce stress and anxiety are very important
to facilitate coping and a sense of control, but seizure precautions are the priority.
30. &Cessation of the lochial discharge signifies healing of the endometrium. Risk of hemorrhage and infection are minimal 3
weeks after a normal vaginal delivery. Telling the client anytime is inappropriate because this response does not provide the
client with the specific information she is requesting. Choice of a contraceptive method is important, but not the specific criteria
for safe resumption of sexual activity. Culturally, the 6- weeks¶ examination has been used as the time frame for resuming sexual
activity, but it may be resumed earlier.
31. . The middle third of the vastus lateralis is the preferred injection site for vitamin K administration because it is free of
blood vessels and nerves and is large enough to absorb the medication. The deltoid muscle of a newborn is not large enough for a
newborn IM injection. Injections into this muscle in a small child might cause damage to the radial nerve. The anterior femoris
muscle is the next safest muscle to use in a newborn but is not the safest. Because of the proximity of the sciatic nerve, the
gluteus maximus muscle should not be until the child has been walking 2 years.
32. .. Bartholin¶s glands are the glands on either side of the vaginal orifice. The clitoris is female erectile tissue found in the
perineal area above the urethra. The parotid glands are open into the mouth. Skene¶s glands open into the posterior wall of the
female urinary meatus.
33. .. The fetal gonad must secrete estrogen for the embryo to differentiate as a female. An increase in maternal estrogen
secretion does not effect differentiation of the embryo, and maternal estrogen secretion occurs in every pregnancy. Maternal
androgen secretion remains the same as before pregnancy and does not effect differentiation. Secretion of androgen by the fetal
gonad would produce a male fetus.
34. . Using bicarbonate would increase the amount of sodium ingested, which can cause complications. Eating low-sodium
crackers would be appropriate. Since liquids can increase nausea avoiding them in the morning hours when nausea is usually the
strongest is appropriate. Eating six small meals a day would keep the stomach full, which often decrease nausea.
35. +. Ballottement indicates passive movement of the unengaged fetus. Ballottement is not a contraction. Fetal kicking felt by
the client represents quickening. Enlargement and softening of the uterus is known as Piskacek¶s sign.
36. +. Chadwick¶s sign refers to the purple-blue tinge of the cervix. Braxton Hicks contractions are painless contractions
beginning around the 4th month. Goodell¶s sign indicates softening of the cervix. Flexibility of the uterus against the cervix is
known as McDonald¶s sign.
37. . Breathing techniques can raise the pain threshold and reduce the perception of pain. They also promote relaxation.
Breathing techniques do not eliminate pain, but they can reduce it. Positioning, not breathing, increases uteroplacental perfusion.
3V. . The client¶s labor is hypotonic. The nurse should call the physical and obtain an order for an infusion of oxytocin, which
will assist the uterus to contact more forcefully in an attempt to dilate the cervix. Administering light sedative would be done for
hypertonic uterine contractions. Preparing for cesarean section is unnecessary at this time. Oxytocin would increase the uterine
contractions and hopefully progress labor before a cesarean would be necessary. It is too early to anticipate client pushing with
contractions.
39. .. The signs indicate placenta previa and vaginal exam to determine cervical dilation would not be done because it could
cause hemorrhage. Assessing maternal vital signs can help determine maternal physiologic status. Fetal heart rate is important to
assess fetal well-being and should be done. Monitoring the contractions will help evaluate the progress of labor.
40. .. A complete placenta previa occurs when the placenta covers the opening of the uterus, thus blocking the passageway for
the baby. This response explains what a complete previa is and the reason the baby cannot come out except by cesarean delivery.
Telling the client to ask the physician is a poor response and would increase the patient¶s anxiety. Although a cesarean would
help to prevent hemorrhage, the statement does not explain why the hemorrhage could occur. With a complete previa, the
placenta is covering all the cervix, not just most of it.
41. +. With a face presentation, the head is completely extended. With a vertex presentation, the head is completely or partially
flexed. With a brow (forehead) presentation, the head would be partially extended.
42. .. With this presentation, the fetal upper torso and back face the left upper maternal abdominal wall. The fetal heart rate
would be most audible above the maternal umbilicus and to the left of the middle. The other positions would be incorrect.
43. &The greenish tint is due to the presence of meconium. Lanugo is the soft, downy hair on the shoulders and back of the
fetus. Hydramnios represents excessive amniotic fluid. Vernix is the white, cheesy substance covering the fetus.
44. .&In a breech position, because of the space between the presenting part and the cervix, prolapse of the umbilical cord is
common. Quickening is the woman¶s first perception of fetal movement. Ophthalmia neonatorum usually results from maternal
gonorrhea and is conjunctivitis. Pica refers to the oral intake of nonfood substances.
45. &Dizygotic (fraternal) twins involve two ova fertilized by separate sperm. Monozygotic (identical) twins involve a common
placenta, same genotype, and common chorion.
46. &The zygote is the single cell that reproduces itself after conception. The chromosome is the material that makes up the cell
and is gained from each parent. Blastocyst and trophoblast are later terms for the embryo after zygote.
47. .&Prepared childbirth was the direct result of the 1950¶s challenging of the routine use of analgesic and anesthetics during
childbirth. The LDRP was a much later concept and was not a direct result of the challenging of routine use of analgesics and
anesthetics during childbirth. Roles for nurse midwives and clinical nurse specialists did not develop from this challenge.
4V. &The ischial spines are located in the mid-pelvic region and could be narrowed due to the previous pelvic injury. The
symphysis pubis, sacral promontory, and pubic arch are not part of the mid-pelvis.
49. +&Variations in the length of the menstrual cycle are due to variations in the proliferative phase. The menstrual, secretory
and ischemic phases do not contribute to this variation.
50. +. Testosterone is produced by the Leyding cells in the seminiferous tubules. Follicle-stimulating hormone and leuteinzing
hormone are released by the anterior pituitary gland. The hypothalamus is responsible for releasing gonadotropin-releasing
hormone.
. $-/ $-/
1. Marco who was diagnosed with brain tumor was scheduled for craniotomy. In preventing the development of
cerebral edema after surgery, the nurse should expect the use of:

a. Diuretics c. Steroids
b. Antihypertensive d. Anticonvulsants

2. Halfway through the administration of blood, the female client complains of lumbar pain. After stopping the
infusion Nurse Hazel should:

a. Increase the flow of normal saline c. Notify the blood bank


b. Assess the pain further d. Obtain vital signs.

3. Nurse Maureen knows that the positive diagnosis for HIV infection is made based on which of the following:

a. A history of high risk sexual behaviors.


b. Positive ELISA and western blot tests
c. Identification of an associated opportunistic infection
d. Evidence of extreme weight loss and high fever

4. Nurse Maureen is aware that a client who has been diagnosed with chronic renal failure recognizes an adequate
amount of high-biologic-value protein when the food the client selected from the menu was:

a. Raw carrots c. Whole wheat bread


b. Apple juice d. Cottage cheese

5. Kenneth who has diagnosed with uremic syndrome has the potential to develop complications. Which among the
following complications should the nurse anticipates:

a. Flapping hand tremors c. Hypotension


b. An elevated hematocrit level d. Hypokalemia

6. A client is admitted to the hospital with benign prostatic hyperplasia, the nurse most relevant assessment would
be:

a. Flank pain radiating in the groin c. Perineal edema


b. Distention of the lower abdomen d. Urethral discharge

7. A client has undergone with penile implant. After 24 hrs of surgery, the client¶s scrotum was edematous and
painful. The nurse should:

a. Assist the client with sitz bath c. Elevate the scrotum using a soft support
b. Apply war soaks in the scrotum d. Prepare for a possible incision and drainage.

V. Nurse hazel receives emergency laboratory results for a client with chest pain and immediately informs the
physician. An increased myoglobin level suggests which of the following?

a. Liver disease c. Hypertension


b. Myocardial damage d. Cancer

9. Nurse Maureen would expect the a client with mitral stenosis would demonstrate symptoms associated with
congestion in the:

a. Right atrium c. Aorta


b. Superior vena cava d. Pulmonary
10. A client has been diagnosed with hypertension. The nurse priority nursing diagnosis would be:

a. Ineffective health maintenance c. Deficient fluid volume


b. Impaired skin integrity d. Pain

11. Nurse Hazel teaches the client with angina about common expected side effects of nitroglycerin including:

a. high blood pressure c. headache


b. stomach cramps d. shortness of breath

12. The following are lipid abnormalities. Which of the following is a risk factor for the development of
atherosclerosis and PVD?

a. High levels of low density lipid (LDL) cholesterol c. Low concentration triglycerides
b. High levels of high density lipid (HDL) cholesterol d. Low levels of LDL cholesterol.

13. Which of the following represents a significant risk immediately after surgery for repair of aortic aneurysm?

a. Potential wound infection c. Potential electrolyte balance


b. Potential ineffective coping d. Potential alteration in renal perfusion

14. Nurse Josie should instruct the client to eat which of the following foods to obtain the best supply of Vitamin
B12?

a. dairy products c. Grains


b. vegetables d. Broccoli

15. Karen has been diagnosed with aplastic anemia. The nurse monitors for changes in which of the following
physiologic functions?

a. Bowel function c. Bleeding tendencies


b. Peripheral sensation d. Intake and out put

16. Lydia is scheduled for elective splenectomy. Before the clients goes to surgery, the nurse in charge final
assessment would be:

a. signed consent c. name band


b. vital signs d. empty bladder

17. What is the peak age range in acquiring acute lymphocytic leukemia (ALL)?

a. 4 to 12 years. c. 40 to 50 years
b. 20 to 30 years d. 60 60 70 years

1V. Marie with acute lymphocytic leukemia suffers from nausea and headache. These clinical manifestations may
indicate all of the following except

a. effects of radiation c. meningeal irritation


b. chemotherapy side effects d. gastric distension

19. A client has been diagnosed with Disseminated Intravascular Coagulation (DIC). Which of the following is
contraindicated with the client?

a. Administering Heparin c. Treating the underlying cause


b. Administering Coumadin d. Replacing depleted blood products
20. Which of the following findings is the best indication that fluid replacement for the client with hypovolemic
shock is adequate?

a. Urine output greater than 30ml/hr c. Diastolic blood pressure greater than 90 mmhg
b. Respiratory rate of 21 breaths/minute d. Systolic blood pressure greater than 110 mmhg

21. Which of the following signs and symptoms would Nurse Maureen include in teaching plan as an early
manifestation of laryngeal cancer?

a. Stomatitis c. Hoarseness
b. Airway obstruction d. Dysphagia

22. Karina a client with myasthenia gravis is to receive immunosuppressive therapy. The nurse understands that this
therapy is effective because it:

a. Promotes the removal of antibodies that impair the transmission of impulses


b. Stimulates the production of acetylcholine at the neuromuscular junction.
c. Decreases the production of autoantibodies that attack the acetylcholine receptors.
d. Inhibits the breakdown of acetylcholine at the neuromuscular junction.

23. A female client is receiving IV Mannitol. An assessment specific to safe administration of the said drug is:

a. Vital signs q4h c. Urine output hourly


b. Weighing daily d. Level of consciousness q4h

24. Patricia a 20 year old college student with diabetes mellitus requests additional information about the advantages
of using a pen like insulin delivery devices. The nurse explains that the advantages of these devices over syringes
includes:

a. Accurate dose delivery c. Lower cost with reusable insulin cartridges


b. Shorter injection time d. Use of smaller gauge needle.

25. A male client¶s left tibia is fractures in an automobile accident, and a cast is applied. To assess for damage to
major blood vessels from the fracture tibia, the nurse in charge should monitor the client for:

a. Swelling of the left thigh c. Prolonged reperfusion of the toes after blanching
b. Increased skin temperature of the foot d. Increased blood pressure

26. After a long leg cast is removed, the male client should:

a. Cleanse the leg by scrubbing with a brisk motion


b. Put leg through full range of motion twice daily
c. Report any discomfort or stiffness to the physician
d. Elevate the leg when sitting for long periods of time.

27. While performing a physical assessment of a male client with gout of the great toe, NurseVivian should assess
for additional tophi (urate deposits) on the:

a. Buttocks c. Face
b. Ears d. Abdomen

2V. Nurse Katrina would recognize that the demonstration of crutch walking with tripod gait was understood when
the client places weight on the:

a. Palms of the hands and axillary regions c. Axillary regions


b. Palms of the hand d. Feet, which are set apart
29. Mang Jose with rheumatoid arthritis states, ³the only time I am without pain is when I lie in bed perfectly still´.
During the convalescent stage, the nurse in charge with Mang Jose should encourage:

a. Active joint flexion and extension c. Range of motion exercises twice daily
b. Continued immobility until pain subsides d. Flexion exercises three times daily

30. A male client has undergone spinal surgery, the nurse should:

a. Observe the client¶s bowel movement and voiding patterns


b. Log-roll the client to prone position
c. Assess the client¶s feet for sensation and circulation
d. Encourage client to drink plenty of fluids

31. Marina with acute renal failure moves into the diuretic phase after one week of therapy. During this phase the
client must be assessed for signs of developing:

a. Hypovolemia c. metabolic acidosis


b. renal failure d. hyperkalemia

32. Nurse Judith obtains a specimen of clear nasal drainage from a client with a head injury. Which of the following
tests differentiates mucus from cerebrospinal fluid (CSF)?

a. Protein c. Glucose
b. Specific gravity d. Microorganism

33. A 22 year old client suffered from his first tonic-clonic seizure. Upon awakening the client asks the nurse, ³What
caused me to have a seizure? Which of the following would the nurse include in the primary cause of tonic clonic
seizures in adults more the 20 years?

a. Electrolyte imbalance c. Epilepsy


b. Head trauma d. Congenital defect

34. What is the priority nursing assessment in the first 24 hours after admission of the client with thrombotic CVA?

a. Pupil size and papillary response c. Echocardiogram


b. cholesterol level d. Bowel sounds

35. Nurse Linda is preparing a client with multiple sclerosis for discharge from the hospital to home. Which of the
following instruction is most appropriate?

a. ³Practice using the mechanical aids that you will need when future disabilities arise´.
b. ³Follow good health habits to change the course of the disease´.
c. ³Keep active, use stress reduction strategies, and avoid fatigue.
d. ³You will need to accept the necessity for a quiet and inactive lifestyle´.

36. The nurse is aware the early indicator of hypoxia in the unconscious client is:

a. Cyanosis c. Hypertension
b. Increased respirations d. Restlessness

37. A client is experiencing spinal shock. Nurse Myrna should expect the function of the bladder to be which of the
following?

a. Normal c. Spastic
b. Atonic d. Uncontrolled
3V. Which of the following stage the carcinogen is irreversible?

a. Progression stage c. Regression stage


b. Initiation stage d. Promotion stage

39. Among the following components thorough pain assessment, which is the most significant?

a. Effect c. Causing factors


b. Cause d. Intensity

40. A 65 year old female is experiencing flare up of pruritus. Which of the client¶s action could aggravate the cause
of flare ups?

a. Sleeping in cool and humidified environment c. Using clothes made from 100% cotton
b. Daily baths with fragrant soap d. Increasing fluid intake

41. Atropine sulfate (Atropine) is contraindicated in all but one of the following client?

a. A client with high blood c. A client with glaucoma


b. A client with bowel obstruction d. A client with U.T.I

42. Among the following clients, which among them is high risk for potential hazards from the surgical experience?

a. 67-year-old client c. 33-year-old client


b. 49-year-old client d. 15-year-old client

43. Nurse Jon assesses vital signs on a client undergone epidural anesthesia. Which of the following would the nurse
assess next?

a. Headache c. Dizziness
b. Bladder distension d. Ability to move legs

44. Nurse Katrina should anticipate that all of the following drugs may be used in the attempt to control the
symptoms of Meniere's disease except:

a. Antiemetics c. Antihistamines
b. Diuretics d. Glucocorticoids

45. Which of the following complications associated with tracheostomy tube?

a. Increased cardiac output c. Increased blood pressure


b. Acute respiratory distress syndrome (ARDS) d. Damage to laryngeal nerves

46. Nurse Faith should recognize that fluid shift in an client with burn injury results from increase in the:

a. Total volume of circulating whole blood c. Permeability of capillary walls


b. Total volume of intravascular plasma d. Permeability of kidney tubules

47. An V3-year-old woman has several ecchymotic areas on her right arm. The bruises are probably caused by:

a. increased capillary fragility and permeability


b. increased blood supply to the skin
c. self inflicted injury
d. elder abuse
4V. Nurse Anna is aware that early adaptation of client with renal carcinoma is:

a. Nausea and vomiting


b. flank pain
c. weight gain
d. intermittent hematuria

49. A male client with tuberculosis asks Nurse Brian how long the chemotherapy must be continued. Nurse Brian¶s
accurate reply would be:

a. 1 to 3 weeks
b. 6 to 12 months
c. 3 to 5 months
d. 3 years and more

50. A client has undergone laryngectomy. The immediate nursing priority would be:

a. Keep trachea free of secretions


b. Monitor for signs of infection
c. Provide emotional support
d. Promote means of communication
Nursing Crib ± Student Nurses¶ Community 231
0 . , $(. $-/ $-/
1. . Glucocorticoids (steroids) are used for their anti-inflammatory action, which decreases the development of
edema.
2. . The blood must be stopped at once, and then normal saline should be infused to keep the line patent and
maintain blood volume.
3. +. These tests confirm the presence of HIV antibodies that occur in response to the presence of the human
immunodeficiency virus (HIV).
4. .. One cup of cottage cheese contains approximately 225 calories, 27 g of protein, 9 g of fat, 30 mg cholesterol,
and 6 g of carbohydrate. Proteins of high biologic value (HBV) contain optimal levels of amino acids essential for
life.
5. . Elevation of uremic waste products causes irritation of the nerves, resulting in flapping hand tremors.
6. +. This indicates that the bladder is distended with urine, therefore palpable.
7. . Elevation increases lymphatic drainage, reducing edema and pain.
V. +. Detection of myoglobin is a diagnostic tool to determine whether myocardial damage has occurred.
9. .. When mitral stenosis is present, the left atrium has difficulty emptying its contents into the left ventricle
because there is no valve to prevent back ward flow into the pulmonary vein, the pulmonary circulation is under
pressure.
10. . Managing hypertension is the priority for the client with hypertension. Clients with hypertension frequently
do not experience pain, deficient volume, or impaired skin integrity. It is the asymptomatic nature of hypertension
that makes it so difficult to treat.
11. . Because of its widespread vasodilating effects, nitroglycerin often produces side effects such as headache,
hypotension and dizziness.
12.A. An increased in LDL cholesterol concentration has been documented at risk factor for the development of
atherosclerosis. LDL cholesterol is not broken down into the liver but is deposited into the wall of the blood vessels.
13. .. There is a potential alteration in renal perfusion manifested by decreased urine output. The altered renal
perfusion may be related to renal artery embolism, prolonged hypotension, or prolonged aortic cross-clamping
during the surgery.
14. . Good source of vitamin B12 are dairy products and meats.
15. . Aplastic anemia decreases the bone marrow production of RBC¶s, white blood cells, and platelets. The client
is at risk for bruising and bleeding tendencies.
16. +&An elective procedure is scheduled in advance so that all preparations can be completed ahead of time. The
vital signs are the final check that must be completed before the client leaves the room so that continuity of care and
assessment is provided for.
17. . The peak incidence of Acute Lymphocytic Leukemia (ALL) is 4 years of age. It is uncommon after 15 years
of age.
1V. .. Acute Lymphocytic Leukemia (ALL) does not cause gastric distention. It does invade the central nervous
system, and clients experience headaches and vomiting from meningeal irritation.
19. +. Disseminated Intravascular Coagulation (DIC) has not been found to respond to oral anticoagulants such as
Coumadin.
20. . Urine output provides the most sensitive indication of the client¶s response to therapy for hypovolemic
shock. Urine output should be consistently greater than 30 to 35 mL/hr.
21. . Early warning signs of laryngeal cancer can vary depending on tumor location. Hoarseness lasting 2 weeks
should be evaluated because it is one of the most common warning signs.
22. . Steroids decrease the body¶s immune response thus decreasing the production of antibodies that attack the
acetylcholine receptors at the neuromuscular junction
23. . The osmotic diuretic mannitol is contraindicated in the presence of inadequate renal function or heart failure
because it increases the intravascular volume that must be filtered and excreted by the kidney.
24. . These devices are more accurate because they are easily to used and have improved adherence in insulin
regimens by young people because the medication can be administered discreetly.
25. . Damage to blood vessels may decrease the circulatory perfusion of the toes, this would indicate the lack of
blood supply to the extremity.
26. .&Elevation will help control the edema that usually occurs.
27. +&Uric acid has a low solubility, it tends to precipitate and form deposits at various sites where blood flow is
least active, including cartilaginous tissue such as the ears.
2V. +. The palms should bear the client¶s weight to avoid damage to the nerves in the axilla.
29. . Active exercises, alternating extension, flexion, abduction, and adduction, mobilize exudates in the joints
relieves stiffness and pain.
30. &Alteration in sensation and circulation indicates damage to the spinal cord, if these occurs notify physician
immediately.
31. . In the diuretic phase fluid retained during the oliguric phase is excreted and may reach 3 to 5 liters daily,
hypovolemia may occur and fluids should be replaced.
32. . The constituents of CSF are similar to those of blood plasma. An examination for glucose content is done to
determine whether a body fluid is a mucus or a CSF. A CSF normally contains glucose.
33. +. Trauma is one of the primary cause of brain damage and seizure activity in adults. Other common causes of
seizure activity in adults include neoplasms, withdrawal from drugs and alcohol, and vascular disease.
34. . It is crucial to monitor the pupil size and papillary response to indicate changes around the cranial nerves.
35. . The nurse most positive approach is to encourage the client with multiple sclerosis to stay active, use stress
reduction techniques and avoid fatigue because it is important to support the immune system while remaining active.
36. .. Restlessness is an early indicator of hypoxia. The nurse should suspect hypoxia in unconscious client who
suddenly becomes restless.
37. +. In spinal shock, the bladder becomes completely atonic and will continue to fill unless the client is
catheterized.
3V. . Progression stage is the change of tumor from the preneoplastic state or low degree of malignancy to a fast
growing tumor that cannot be reversed.
39. .. Intensity is the major indicative of severity of pain and it is important for the evaluation of the treatment.
40. +&The use of fragrant soap is very drying to skin hence causing the pruritus.
41. &Atropine sulfate is contraindicated with glaucoma patients because it increases intraocular pressure.
42. . A 67 year old client is greater risk because the older adult client is more likely to have a less-effective
immune system.
43. +. The last area to return sensation is in the perineal area, and the nurse in charge should monitor the client for
distended bladder.
44. .. Glucocorticoids play no significant role in disease treatment.
45. .. Tracheostomy tube has several potential complications including bleeding, infection and laryngeal nerve
damage.
46. . In burn, the capillaries and small vessels dilate, and cell damage cause the release of a histamine-like
substance. The substance causes the capillary walls to become more permeable and significant quantities of fluid are
lost.
47. . Aging process involves increased capillary fragility and permeability. Older adults have a decreased amount
of subcutaneous fat and cause an increased incidence of bruise like lesions caused by collection of extravascular
blood in loosely structured dermis.
4V. .. Intermittent pain is the classic sign of renal carcinoma. It is primarily due to capillary erosion by the
cancerous growth.
49. +&Tubercle bacillus is a drug resistant organism and takes a long time to be eradicated. Usually a combination
of three drugs is used for minimum of 6 months and at least six months beyond culture conversion.
50. &Patent airway is the most priority; therefore removal of secretions is necessary.
c1 -/
1. Marco approached Nurse Trish asking for advice on how to deal with his alcohol addiction. Nurse Trish should
tell the client that the only effective treatment for alcoholism is:

a. Psychotherapy c. Total abstinence


b. Alcoholics anonymous (A.A.) d. Aversion Therapy

2. Nurse Hazel is caring for a male client who experience false sensory perceptions with no basis in reality. This
perception is known as:

a. Hallucinations c. Loose associations


b. Delusions d. Neologisms

3. Nurse Monet is caring for a female client who has suicidal tendency. When accompanying the client to the
restroom, Nurse Monet should«

a. Give her privacy


b. Allow her to urinate
c. Open the window and allow her to get some fresh air
d. Observe her

4. Nurse Maureen is developing a plan of care for a female client with anorexia nervosa. Which action should the
nurse include in the plan?

a. Provide privacy during meals c. Encourage client to exercise to reduce anxiety


b. Set-up a strict eating plan for the client d. Restrict visits with the family

5. A client is experiencing anxiety attack. The most appropriate nursing intervention should include?

a. Turning on the television


b. Leaving the client alone
c. Staying with the client and speaking in short sentences
d. Ask the client to play with other clients

6. A female client is admitted with a diagnosis of delusions of GRANDEUR. This diagnosis reflects a belief that one
is:

a. Being Killed c. Responsible for evil world


b. Highly famous and important d. Connected to client unrelated to oneself

7. A 20 year old client was diagnosed with dependent personality disorder. Which behavior is not likely to be
evidence of ineffective individual coping?

a. Recurrent self-destructive behavior


b. Avoiding relationship
c. Showing interest in solitary activities
d. Inability to make choices and decision without advise

V. A male client is diagnosed with schizotypal personality disorder. Which signs would this client exhibit during
social situation?

a. Paranoid thoughts c. Independence need


b. Emotional affect d. Aggressive behavior
9. Nurse Claire is caring for a client diagnosed with bulimia. The most appropriate initial goal for a client diagnosed
with bulimia is?

a. Encourage to avoid foods c. Eat only three meals a day


b. Identify anxiety causing situations d. Avoid shopping plenty of groceries

10. Nurse Tony was caring for a 41 year old female client. Which behavior by the client indicates adult cognitive
development?

a. Generates new levels of awareness


b. Assumes responsibility for her actions
c. Has maximum ability to solve problems and learn new skills
d. Her perception are based on reality

11.A neuromuscular blocking agent is administered to a client before ECT therapy. The Nurse should carefully
observe the client for?

a. Respiratory difficulties c. Dizziness


b. Nausea and vomiting d. Seizures

12.A 75 year old client is admitted to the hospital with the diagnosis of dementia of the Alzheimer¶s type and
depression. The symptom that is unrelated to depression would be?

a. Apathetic response to the environment c. Shallow of labile effect


b. ³I don¶t know´ answer to questions d. Neglect of personal hygiene

13.Nurse Trish is working in a mental health facility; the nurse priority nursing intervention for a newly admitted
client with bulimia nervosa would be to?

a. Teach client to measure I & O c. Observe client during meals


b. Involve client in planning daily meal d. Monitor client continuously

14.Nurse Patricia is aware that the major health complication associated with intractable anorexia nervosa would be?

a. Cardiac dysrhythmias resulting to cardiac arrest


b. Glucose intolerance resulting in protracted hypoglycemia
c. Endocrine imbalance causing cold amenorrhea
d. Decreased metabolism causing cold intolerance

15.Nurse Anna can minimize agitation in a disturbed client by?

a. Increasing stimulation c. increasing appropriate sensory perception


b. limiting unnecessary interaction d. ensuring constant client and staff contact

16.A 39 year old mother with obsessive-compulsive disorder has become immobilized by her elaborate hand
washing and walking rituals. Nurse Trish recognizes that the basis of O.C. disorder is often:

a. Problems with being too conscientious c. Feelings of guilt and inadequacy


b. Problems with anger and remorse d. Feeling of unworthiness and hopelessness

17.Mario is complaining to other clients about not being allowed by staff to keep food in his room. Which of the
following interventions would be most appropriate?

a. Allowing a snack to be kept in his room c. Ignoring the clients behavior


b. Reprimanding the client d. Setting limits on the behavior
1V.Conney with borderline personality disorder who is to be discharge soon threatens to ³do something´ to herself if
discharged. Which of the following actions by the nurse would be most important?

a. Ask a family member to stay with the client at home temporarily


b. Discuss the meaning of the client¶s statement with her
c. Request an immediate extension for the client
d. Ignore the clients statement because it¶s a sign of manipulation

19.Joey a client with antisocial personality disorder belches loudly. A staff member asks Joey, ³Do you know why
people find you repulsive?´ this statement most likely would elicit which of the following client reaction?

a. Depensiveness c. Shame
b. Embarrassment d. Remorsefulness

20.Which of the following approaches would be most appropriate to use with a client suffering from narcissistic
personality disorder when discrepancies exist between what the client states and what actually exist?

a. Rationalization c. Limit setting


b. Supportive confrontation d. Consistency

21.Cely is experiencing alcohol withdrawal exhibits tremors, diaphoresis and hyperactivity. Blood pressure is
190/V7 mmhg and pulse is 92 bpm. Which of the medications would the nurse expect to administer?

a. Naloxone (Narcan) c. Lorazepam (Ativan)


b. Benzlropine (Cogentin) d. Haloperidol (Haldol)

22.Which of the following foods would the nurse Trish eliminate from the diet of a client in alcohol withdrawal?

a. Milk c. Soda
b. Orange Juice d. Regular Coffee

23.Which of the following would Nurse Hazel expect to assess for a client who is exhibiting late signs of heroin
withdrawal?

a. Yawning & diaphoresis c. Constipation & steatorrhea


b. Restlessness & Irritability d. Vomiting and Diarrhea

24.To establish open and trusting relationship with a female client who has been hospitalized with severe anxiety,
the nurse in charge should?

a. Encourage the staff to have frequent interaction with the client


b. Share an activity with the client
c. Give client feedback about behavior
d. Respect client¶s need for personal space

25. Nurse Monette recognizes that the focus of environmental (MILIEU) therapy is to:

a. Manipulate the environment to bring about positive changes in behavior


b. Allow the client¶s freedom to determine whether or not they will be involved in activities
c. Role play life events to meet individual needs
d. Use natural remedies rather than drugs to control behavior
26.Nurse Trish would expect a child with a diagnosis of reactive attachment disorder to:

a. Have more positive relation with the father than the mother
b. Cling to mother & cry on separation
c. Be able to develop only superficial relation with the others
d. Have been physically abuse

27.When teaching parents about childhood depression Nurse Trina should say?

a. It may appear acting out behavior c. Is short in duration & resolves easily
b. Does not respond to conventional treatment d. Looks almost identical to adult depression

2V.Nurse Perry is aware that language development in autistic child resembles:

a. Scanning speech c. Shuttering


b. Speech lag d. Echolalia

29.A 60 year old female client who lives alone tells the nurse at the community health center ³I really don¶t need
anyone to talk to´. The TV is my best friend. The nurse recognizes that the client is using the defense mechanism
known as?

a. Displacement c. Sublimation
b. Projection d. Denial

30.When working with a male client suffering phobia about black cats, Nurse Trish should anticipate that a problem
for this client would be?

a. Anxiety when discussing phobia


b. Anger toward the feared object
c. Denying that the phobia exist
d. Distortion of reality when completing daily routines

31.Linda is pacing the floor and appears extremely anxious. The duty nurse approaches in an attempt to alleviate
Linda¶s anxiety. The most therapeutic question by the nurse would be?

a. Would you like to watch TV? c. Are you feeling upset now?
b. Would you like me to talk with you? d. Ignore the client

32.Nurse Penny is aware that the symptoms that distinguish post traumatic stress disorder from other anxiety
disorder would be:

a. Avoidance of situation & certain activities that resemble the stress


b. Depression and a blunted affect when discussing the traumatic situation
c. Lack of interest in family & others
d. Re-experiencing the trauma in dreams or flashback

33.Nurse Benjie is communicating with a male client with substance-induced persisting dementia; the client cannot
remember facts and fills in the gaps with imaginary information. Nurse Benjie is aware that this is typical of?

a. Flight of ideas c. Confabulation


b. Associative looseness d. Concretism
34.Nurse Joey is aware that the signs & symptoms that would be most specific for diagnosis anorexia are?

a. Excessive weight loss, amenorrhea & abdominal distension


b. Slow pulse, 10% weight loss & alopecia
c. Compulsive behavior, excessive fears & nausea
d. Excessive activity, memory lapses & an increased pulse

35.A characteristic that would suggest to Nurse Anne that an adolescent may have bulimia would be:

a. Frequent regurgitation & re-swallowing of food c. Badly stained teeth


b. Previous history of gastritis d. Positive body image

36.Nurse Monette is aware that extremely depressed clients seem to do best in settings where they have:

a. Multiple stimuli c. Minimal decision making


b. Routine Activities d. Varied Activities

37.To further assess a client¶s suicidal potential. Nurse Katrina should be especially alert to the client expression of:

a. Frustration & fear of death c. Anxiety & loneliness


b. Anger & resentment d. Helplessness & hopelessness

3V.A nursing care plan for a male client with bipolar I disorder should include:

a. Providing a structured environment


b. Designing activities that will require the client to maintain contact with reality
c. Engaging the client in conversing about current affairs
d. Touching the client provide assurance

39.When planning care for a female client using ritualistic behavior, Nurse Gina must recognize that the ritual:

a. Helps the client focus on the inability to deal with reality


b. Helps the client control the anxiety
c. Is under the client¶s conscious control
d. Is used by the client primarily for secondary gains

40.A 32 year old male graduate student, who has become increasingly withdrawn and neglectful of his work and
personal hygiene, is brought to the psychiatric hospital by his parents. After detailed assessment, a diagnosis of
schizophrenia is made. It is unlikely that the client will demonstrate:

a. Low self esteem c. Effective self boundaries


b. Concrete thinking d. Weak ego

41.A 23 year old client has been admitted with a diagnosis of schizophrenia says to the nurse ³Yes, its march, March
is little woman´. That¶s literal you know´. These statement illustrate:

a. Neologisms c. Flight of ideas


b. Echolalia d. Loosening of association

42.A long term goal for a paranoid male client who has unjustifiably accused his wife of having many extramarital
affairs would be to help the client develop:

a. Insight into his behavior c. Feeling of self worth


b. Better self control d. Faith in his wife
43.A male client who is experiencing disordered thinking about food being poisoned is admitted to the mental health
unit. The nurse uses which communication technique to encourage the client to eat dinner?

a. Focusing on self-disclosure of own food preference


b. Using open ended question and silence
c. Offering opinion about the need to eat
d. Verbalizing reasons that the client may not choose to eat

44.Nurse Nina is assigned to care for a client diagnosed with Catatonic Stupor. When Nurse Nina enters the client¶s
room, the client is found lying on the bed with a body pulled into a fetal position. Nurse Nina should?

a. Ask the client direct questions to encourage talking


b. Rake the client into the dayroom to be with other clients
c. Sit beside the client in silence and occasionally ask open-ended question
d. Leave the client alone and continue with providing care to the other clients

45.Nurse Tina is caring for a client with delirium and states that ³look at the spiders on the wall´. What should the
nurse respond to the client?

a. ³You¶re having hallucination, there are no spiders in this room at all´


b. ³I can see the spiders on the wall, but they are not going to hurt you´
c. ³Would you like me to kill the spiders´
d. ³I know you are frightened, but I do not see spiders on the wall´

46.Nurse Jonel is providing information to a community group about violence in the family. Which statement by a
group member would indicate a need to provide additional information?

a. ³Abuse occurs more in low-income families´ c. ³Abuser use fear and intimidation´
b. ³Abuser Are often jealous or self-centered´ d. ³Abuser usually have poor self-esteem´

47.During electroconvulsive therapy (ECT) the client receives oxygen by mask via positive pressure ventilation. The nurse
assisting with this procedure knows that positive pressure ventilation is necessary because?

a. Anesthesia is administered during the procedure


b. Decrease oxygen to the brain increases confusion and disorientation
c. Grand mal seizure activity depresses respirations
d. Muscle relaxations given to prevent injury during seizure activity depress respirations.

4V.When planning the discharge of a client with chronic anxiety, Nurse Chris evaluates achievement of the discharge
maintenance goals. Which goal would be most appropriately having been included in the plan of care requiring evaluation?

a. The client eliminates all anxiety from daily situations


b. The client ignores feelings of anxiety
c. The client identifies anxiety producing situations
d. The client maintains contact with a crisis counselor

49.Nurse Tina is caring for a client with depression who has not responded to antidepressant medication. The nurse anticipates
that what treatment procedure may be prescribed.

a. Neuroleptic medication c. Psychosurgery


b. Short term seclusion d. Electroconvulsive therapy

50.Mario is admitted to the emergency room with drug-included anxiety related to over ingestion of prescribed antipsychotic
medication. The most important piece of information the nurse in charge should obtain initially is the:

a. Length of time on the med.


b. Name of the ingested medication & the amount ingested
c. Reason for the suicide attempt
d. Name of the nearest relative & their phone number
0 . , $(c1 -/
1. . Total abstinence is the only effective treatment for alcoholism
2. . Hallucinations are visual, auditory, gustatory, tactile or olfactory
perceptions that have no basis in reality.
3. .. The Nurse has a responsibility to observe continuously the acutely suicidal
client. The Nurse should watch for clues, such as communicating suicidal
thoughts, and messages; hoarding medications and talking about death.
4. +&Establishing a consistent eating plan and monitoring client¶s weight are
important to this disorder.
5. . Appropriate nursing interventions for an anxiety attack include using short
sentences, staying with the client, decreasing stimuli, remaining calm and
medicating as needed.
6. +. Delusion of grandeur is a false belief that one is highly famous and
important.
7. .. Individual with dependent personality disorder typically shows
indecisiveness submissiveness and clinging behavior so that others will
make decisions with them.
V. . Clients with schizotypal personality disorder experience excessive social
anxiety that can lead to paranoid thoughts
9. +. Bulimia disorder generally is a maladaptive coping response to stress and
underlying issues. The client should identify anxiety causing situation that
stimulate the bulimic behavior and then learn new ways of coping with the
anxiety.
10. . An adult age 31 to 45 generates new level of awareness.
11. . Neuromuscular Blocker, such as SUCCINYLCHOLINE (Anectine)
produces respiratory depression because it inhibits contractions of respiratory
muscles.
12. . With depression, there is little or no emotional involvement therefore little
alteration in affect.
13. .. These clients often hide food or force vomiting; therefore they must be carefully monitored.
14. . These clients have severely depleted levels of sodium and potassium
because of their starvation diet and energy expenditure, these electrolytes are
necessary for cardiac functioning.
15. +. Limiting unnecessary interaction will decrease stimulation and agitation.
16. . Ritualistic behavior seen in this disorder is aimed at controlling guilt and
inadequacy by maintaining an absolute set pattern of behavior.
17. .. The nurse needs to set limits in the client¶s manipulative behavior to help
the client control dysfunctional behavior. A consistent approach by the staff is
necessary to decrease manipulation.
1V. +. Any suicidal statement must be assessed by the nurse. The nurse should
discuss the client¶s statement with her to determine its meaning in terms of
suicide.
19. . When the staff member ask the client if he wonders why others find him
repulsive, the client is likely to feel defensive because the question is
belittling. The natural tendency is to counterattack the threat to self image.
Nursing Crib ± Student Nurses¶ Community 243
20. +. The nurse would specifically use supportive confrontation with the client to
point out discrepancies between what the client states and what actually
exists to increase responsibility for self.
21. . The nurse would most likely administer benzodiazepine, such as lorazepan
(ativan) to the client who is experiencing symptom: The client¶s experiences
symptoms of withdrawal because of the rebound phenomenon when the
sedation of the CNS from alcohol begins to decrease.
22. .. Regular coffee contains caffeine which acts as psychomotor stimulants
and leads to feelings of anxiety and agitation. Serving coffee top the client
may add to tremors or wakefulness.
23. .. Vomiting and diarrhea are usually the late signs of heroin withdrawal,
along with muscle spasm, fever, nausea, repetitive, abdominal cramps and
backache.
24. .. Moving to a client¶s personal space increases the feeling of threat, which
increases anxiety.
25. . Environmental (MILIEU) therapy aims at having everything in the client¶s
surrounding area toward helping the client.
26. . Children who have experienced attachment difficulties with primary
caregiver are not able to trust others and therefore relate superficially
27. . Children have difficulty verbally expressing their feelings, acting out
behavior, such as temper tantrums, may indicate underlying depression.
2V. .. The autistic child repeat sounds or words spoken by others.
29. .. The client statement is an example of the use of denial, a defense that
blocks problem by unconscious refusing to admit they exist
30. . Discussion of the feared object triggers an emotional response to the
object.
31. +. The nurse presence may provide the client with support & feeling of
control.
32. .. Experiencing the actual trauma in dreams or flashback is the major
symptom that distinguishes post traumatic stress disorder from other anxiety
disorder.
33. . Confabulation or the filling in of memory gaps with imaginary facts is a
defense mechanism used by people experiencing memory deficits.
34. . These are the major signs of anorexia nervosa. Weight loss is excessive
(15% of expected weight)
35. . Dental enamel erosion occurs from repeated self-induced vomiting.
36. +. Depression usually is both emotional & physical. A simple daily routine is
the best, least stressful and least anxiety producing.
37. .. The expression of these feeling may indicate that this client is unable to
continue the struggle of life.
3V. . Structure tends to decrease agitation and anxiety and to increase the
client¶s feeling of security.
39. +. The rituals used by a client with obsessive compulsive disorder help
control the anxiety level by maintaining a set pattern of action.
40. . A person with this disorder would not have adequate self-boundaries
Nursing Crib ± Student Nurses¶ Community 244
41. .. Loose associations are thoughts that are presented without the logical
connections usually necessary for the listening to interpret the message.
42. . Helping the client to develop feeling of self worth would reduce the client¶s
need to use pathologic defenses.
43. +. Open ended questions and silence are strategies used to encourage
clients to discuss their problem in descriptive manner.
44. . Clients who are withdrawn may be immobile and mute, and require
consistent, repeated interventions. Communication with withdrawn clients
requires much patience from the nurse. The nurse facilitates communication
with the client by sitting in silence, asking open-ended question and pausing
to provide opportunities for the client to respond.
45. .. When hallucination is present, the nurse should reinforce reality with the
client.
46. . Personal characteristics of abuser include low self-esteem, immaturity,
dependence, insecurity and jealousy.
47. .. A short acting skeletal muscle relaxant such as succinylcholine (Anectine)
is administered during this procedure to prevent injuries during seizure.
4V. . Recognizing situations that produce anxiety allows the client to prepare to
cope with anxiety or avoid specific stimulus.
49. .. Electroconvulsive therapy is an effective treatment for depression that has
not responded to medication
50. +. In an emergency, lives saving facts are obtained first. The name and the
amount of medication ingested are of outmost important in treating this
potentially life threatening situation.

Potrebbero piacerti anche